MBE KAPLAN--CONTRACTS Flashcards

1
Q
  1. A man who owned a haberdashery placed an order by telephone from a wholesale supplier of cashmere and wool clothing, for “triple-dozen purple cashmere socks, size 10—13 at current resale price.” The supplier’s sales agent orally accepted the order at the agreed price of $250 per dozen. In accordance with the supplier’s customary business practice, the sales agent then mailed the following confirmation letter, which he signed and dated:
    “As per your telephone order, this letter serves to confirm the purchase of 36 dozen cashmere socks, color purple, size 10—13, at the agreed price of $250 per dozen. Total sales price: $9,000.”
    This letter was received by the man, who briefly glanced at it but failed to notice the “36 dozen” wording or the total price. The man placed the letter in his files and did not respond to it. Three weeks later, the supplier tendered 36 dozen purple cashmere socks, which the man rejected on the grounds that he had ordered only three dozen. The supplier resold the same purple cashmere socks to another buyer for a total price of $8,000.
    The supplier sued the man for the $1,000 difference. If the man pleads the statute of frauds as a defense, will such a defense be successful?
    (A) Yes, because the statute was not satisfied by the supplier’s tender of the goods, which were rejected by the man, and the supplier did not rely on the oral agreement other than by attempting delivery.
    (B) Yes, because the agreed price for three dozen socks was over $500, and the supplier’s written memo incorrectly stated the quantity of goods ordered.
    (C) No, because the supplier’s written memo was sufficient to satisf’ the statute as against the supplier, and the man, having reason to know of the memo’s contents, failed to give notice of objection within 10 days of receipt.
    (D) No, because the supplier’s written memo operated as an acceptance, with proposals for additional terms that became part of the contract after the man failed to object to such additional terms within a reasonable time.
A


1. (C) According to UCC 2-201(2), when a contract is between merchants (which would be the case here), if one merchant, within a reasonable time, sends a writing that is sufficient against the sender confirming the contract, and the other merchant receives it, has reason to know of its contents but fails to object to it within 10 days after receipt, the confirmation is deemed to satisfy the Statute of Frauds. Therefore, the supplier’s letter sent by its sales agent would satisfy the Statute of Frauds, since the man failed to respond to it. Choice (A) is wrong, because the letter sent by the supplier’s sales agent satisfied the statute before the tender of the goods by the supplier. Choice (B) is incorrect, because UCC 2-201 states that a writing is not insufficient to satisfy the statute because it incorrectly states a term. Choice (D) is incorrect, because, in accord with UCC Section 2-207, additional terms that materially alter the original bargain wilL not be included unless expressly agreed to by the other party. Since the man who owned the shop did not expressly agree to the new terms, they did not become part of the contract. However, this would have nothing to do with the interrogatory, which asked only if the Statute of Frauds would be a successful defense. Whether the additional terms are or are not a part of the contract does not have an impact on this question.

How well did you know this?
1
Not at all
2
3
4
5
Perfectly
2
Q
  1. A woman hired a builder to build a house according to certain plans and specifications prepared by the woman’s architect. The agreed upon price was $250,000, with construction to be completed within four months. Two weeks after the building contract was formed, the builder contacted a lumber yard to purchase wood necessary for the construction of the house. The builder and the owner of the lumber yard entered into a valid written agreement whereby the lumber yard was to supply the necessary lumber in exchange for $10,000, payable by the builder within 30 days. One week later, a fire destroyed a good portion of the lumber yard’s supply of lumber. As a result, the lumber yard refused to supply lumber to the builder. The builder was unable to find another supplier of lumber and therefore notified the woman that he would be unable to complete her building on time.
    If the woman sues the owner of the lumber yard for breach of contract, will she prevail?
    (A) Yes, because by operation of law the woman is an equitable assignee of the builder’s claim against the owner of the lumber yard for breach of contract.
    (B) Yes, but only if the builder’s contract with the owner of the lumber yard was not discharged by the fire.
    (C) No, because privity of contract does not exist between the woman and the owner of the lumber yard.
    (D) No, because the woman is only an incidental beneficiary of the contract between the builder and the owner of the lumber yard.
A
  1. (D) With respect to third-party beneficiary contracts, remember the following rule:
    intention to confer a benefit on the third party has always been declared by the courts as essential to the right of the third party to enforce the promise. A useful test to determine the necessary intention is to askyourself “To whom is performance to be rendered?” If performance is to the third party, she is a protected beneficiary and thus entitled to sue. But if the promised performance is to be rendered to the promisee, the contract is for the benefit alone of the parties thereto, and any third party is an incidental beneficiary. In this particular fact pattern, performance was to be rendered to the builder, the promisee. In other words, the builder’s motive in ordering the lumber was to make a profit on the construction contract. Since his primary intent was not to benefit the woman, she is viewed as an incidental beneficiary. Choice (A) is incorrect, because there would be no such creation of an equitable assignment.There is no indication thatthe ownerof the Lumberyard knewthatthe lumber in question was being supplied for use in the woman’s house. The woman, at best, is only incidentally involved in the contract between the builder and the owner of the lumber yard. Choice (B) is wrong, because it does not matter whether the builder’s contract was discharged by the fire. The woman, as an incidental beneficiary, has no right to maintain the suit. Choice (C) is not as good an answer as Choice (D). Even though the woman was not in direct privity with the owner of the lumberyard, she could stiLl sue if she was an intended beneficiary. However, since she was only an incidental beneficiary, she is prevented from suing. Choice (D) better expresses this.
How well did you know this?
1
Not at all
2
3
4
5
Perfectly
3
Q
  1. A creditor loaned his friend $15,000 to help pay for the friend’s daughter’s college tuition. Six months later, the friend lost his job and was unable to repay the loan to the creditor. After learning of his friend’s situation, the creditor sent his friend the following letter on June 1:
    “I promise to discharge the $15,000 debt which you owe me upon delivery of your autographed baseball bat if you promise to deliver the bat to me by August 1 .“
    After receiving this letter, the friend telephoned the creditor and accepted the offer.
    The friend’s verbal acceptance of the creditor’s offer most likely effectuated
    (A) a bilateral executory accord.
    (B) an accord and satisfaction.
    (C) a substituted compromise agreement.
    (D) a novation.
A
  1. (A) A bilateral executory accord is “an agreement that an existing claim shalL be discharged in the future by the rendition of a substituted performance.” For example, C (creditor) writes D (debtor), “I promise to discharge the debtyou owe me upon delivery of your black mare if you promise to deliverthe horse to me within a reasonable time.” D promises. Their agreement is a bilateral executory accord. Note, too, that if D delivers the horse and C accepts it, there is an accord and satisfaction. The agreement is the accord. Its performance is the satisfaction. In this fact pattern, choice (B) is wrong, because the accord and satisfaction will not occur until the friend actuay de’ivers the baseball bat. Choice (C) is incorrect, because a substituted compromise agreement usually involves a disputed or unliquidated cLaim, in which case the creditor enters into a new or substituted agreement to discharge the uncertainty on an unliquidated claim. However, an executory bilateral accord generally covers a liquidated and undisputed obligation. Choice (D) is wrong, because a novation is synonymous with a “substituted contract” usually involving at least one obligor or obligee who was not a party to the original contract. There is no substituted party in this question.
How well did you know this?
1
Not at all
2
3
4
5
Perfectly
4
Q
  1. A man was the owner of the newly constructed hotel in a city. On March 15, the man received a telephone call from a salesperson who was a distributor of hotel equipment. The salesperson offered to sell the man 1,000 fire extinguishers for his hotel. The salesperson told the man that the cost of the fire extinguishers would be $35,000 (or $35 apiece), payable 90 days after delivery. The salesperson promised to have the fire extinguishers installed no later than April 15.
    On March 16, the man telephoned the salesperson and accepted the offer. The following day, the man mailed the following memo to the salesperson:
    “Please be advised that I shall take a 15 percent discount for cash payment seven days after installation.” The salesperson received the man’s correspondence on March 20. On April 1, the salesperson sent a telegram to the man, stating: “It’s apparent we don’t have an enforceable contract in effect. I will not be delivering the fire extinguishers on April 15 or any other time.”
    The man brings suit against the salesperson for breach of contract. The salesperson asserts the defense of the statute of frauds under the UCC.
    Which of the following is the most accurate statement regarding the salesperson’s defenses?
    (A) The salesperson’s defense is valid, because the man’s memo was not sufficient to indicate that a contract was formed.
    (B) The salesperson’s defense is valid, because the man’s memo was inconsistent with the terms of the salesperson’s oral offer.
    (C) The salesperson’s defense is not valid, because the salesperson failed to respond to the man’s memo within a reasonable period of time.
    (D) The salesperson’s defense is not valid, because under the UCC the statute of frauds is not applicable in agreements between merchants.
A
  1. (A) Under the UCC, the only term that must be included in a writing sufficient to satisfy the Statute of Frauds is the quantity term. The price, time and place of payment or delivery, the general quality of the goods, or any particular warranties may all be omitted. (Comment 1, Section 2-201.) Since the man’s March 17 memo did not mention a quantity, it does not satisfy the Statute of Frauds. Choice (B) is incorrect, because the fact that the memo is inconsistent with the offer does not invalidate the memo for purposes of the Statute of Frauds. Since both parties were considered to be merchants, the salesperson could have objected to the different terms in the memo within 10 days to prevent them from being controlling. Choice (C) is not the best choice for a couple of reasons. First, while it is true that a merchant can object to a memo sent by another merchant, the objection must be made within 10 days of receipt, not a reasonable time. Second, since the memo was not sufficient to satisfy the Statute of Frauds, no objection to the memo was needed here. Choice (D) is incorrect. The Statute of Frauds as discussed above is applicable to agreements between merchants.
How well did you know this?
1
Not at all
2
3
4
5
Perfectly
5
Q
  1. On June 1, an owner of a business that built mobile homes was visited by a representative of a company that manufactured propane tanks. Propane tanks were an essential component of the mobile homes produced by the builder. The representative told the owner that the company could supply propane tanks at $50 per tank, a substantial savings over what the owner currently paid for propane tanks. The owner asked if the company could supply 1,000 propane tanks by the end of the month, and the representative assured him that they could. The owner stated that he would think about it and decide what to do within a week.
    On June 3, the owner sent the following memo to the company headquarters at the address provided by the representative: “I am happy to confirm my order of 1,000 propane tanks, to be delivered by June 30. I have always received a 10 percent discount for cash payment, so I will assume you will grant me the same discount. I will have $45,000 cash ready to give to your representative at the time of delivery.”
    On June 30, the company delivered 1,000 propane tanks to the owner. The representative accompanied the delivery and presented the owner with a bill for $50,000. The owner refused to pay any more than
    $45,000.
    Which of the following accurately states the legal rights of the parties?
    (A) The contract price was $45,000, because the June 3 memo was an effective integration of their agreement.
    (B) The contract price was $45,000, because the company did not specifically object to the 10 percent discount stipulated by the owner in his June 3 memo.
    (C) The contract price was $50,000 if the discount term in the owner’s June 3 memo materially altered the terms of the company’s offer.
    (D) The contract price was $50,000 even though the company’s offer did not expressly limit acceptance to the terms contained therein.
A
  1. (C) In accordance with UCC Section 2-207, when the parties to a contract are both merchants (as is the case here), additional or different terms contained in an acceptance will become part of the contract unless 1) the offer is expressly limited to acceptance on its own terms, 2) the offeror objects to the additional or different terms within a reasonable time after receiving notice of them, or 3) the additional or different terms materially alter the contract. Therefore, if the discount term proposed by the owner is deemed to be a material variation of the contract (a very likely occurrence under these facts), the terms will revert back to those in the offer, making the contract price $50,000. Choice (A) is incorrect, because the June 3 memo would not be deemed an effective integration if it proposed terms that materially alter the contract. Choice (B) is wrong, because the company does not have to specifically object to the additional or different terms if they constitute a material alteration. The additional or different terms would be knocked out by the simple fact that they are a material variation, even in the absence of an objection. Choice (D) is not the best choice, because it does not address the possibility of the additional or different terms being a material variation of the contract.
How well did you know this?
1
Not at all
2
3
4
5
Perfectly
6
Q
  1. On September 1, a buyer contracted to purchase
    10,000 widgets from a seller for $1 per widget, delivery to be made no later than September 30. On September 15, a worldwide shortage of widgets caused a steep increase in the market price for widgets, so the seller decided not to deliver the widgets to the buyer. The seller sent a letter to the buyer, stating that the widgets would not be delivered. The buyer received the seller’s letter on September 20. On October 15, the buyer filed suit against the seller for breach of contract.
    In determining the damages to which the buyer is entitled, which of the following would be taken as the market price of widgets?
    (A) The market price on September 1.
    (B) The market price on September 15.
    (C) The market price on September 20.
    (D) The market price on September 30.
A
  1. (C) According to UCC Section 2-713, the measure of damages for non-delivery or repudiation by the seller is the difference between the market price at the time when the buyer learned of the breach and the contract price together with incidental (and! or consequential) damages. Choice (A) is wrong, because the date of the making of the contract is not the controlling date for determining damages. Choice (B) is wrong, because it is not the date the seller decided to breach the contract that determines damages but the date the buyer learned of the breach. Choice (D) is incorrect, because the market value on the date delivery was to be made is not the determining factor.
How well did you know this?
1
Not at all
2
3
4
5
Perfectly
7
Q
  1. A purchasing agent for a women’s clothing store negotiated a contract with dressmaking company to purchase a specified quantity of khaki garments at a price of $75,000. One week later, the purchasing agent received a telephone call from the vice president of the dressmaking company, who informed her that the dressmaking company’s sales representative had made an error in calculating the contract price. As a result, the vice president said that unless the women’s clothing store agreed to pay an additional $15,000, the garments would not be delivered.
    If the purchasing agent should have known that the dressmaking company’s original price term of $75,000 was in error, but agreed to the contract anyway, which of the following is the most accurate statement?
    (A) There was an enforceable contract at the original price term, because the mistake resulted from an error in computation, not in judgment.
    (B) There was an enforceable contract at the original price term, because the mistake was unilateral.
    (C) There was no valid contract formed, because there was no mutuality of assent.
    (D) There was a voidable contract, because the purchasing agent should have known of the error.
A
  1. (D) As a general rule, in all unilateral mistake situations, if the non-mistaken party is aware of the other party’s mistake and takes advantage of the innocent party’s mistake, the contract is voidabLe at the discretion of the mistaken party. Choice (A) is incorrect, because relief is rareLy granted for an error in judgment, but can be granted for a computation error. Choice (B) is wrong, because, as stated above, relief can be granted for a unilateraL mistake when the non-mistaking party is aware of the existence of the mistake. Choice (C) is not the best choice, because mutuaL assent is held to exist; the unilateral mistake could make the contract voidable by the mistaken party.
How well did you know this?
1
Not at all
2
3
4
5
Perfectly
8
Q
  1. A law bookstore entered into a written contract to purchase from the publisher 100 copies of the latest edition of a certain casebook for $10 per book. Three days after the contract was formed, but prior to delivery of the casebooks, the publisher called the owner of the law bookstore and informed him that, because of a calculation error, the price for the casebooks should have been $11 per book, and the shipment could not be delivered unless the owner promised to pay that amount. The owner reluctantly agreed.
    The owner’s agreement to pay $11 per book is
    (A) enforceable, because it was not supported by any new consideration.
    (B) enforceable, under the principle of promissory estoppel.
    (C) unenforceable, because it is violative of the statute of frauds.
    (D) unenforceable, because the error resulted from the publisher’s computational error.
A
  1. (C) A modification regarding the sale of goods of $500 or more comes within the Statute of Frauds and must be in writing to be enforceable. Since the owner’s agreement was oral and never memorialized in writing, it would be unenforceable. Choice (A) is incorrect. While it is true that underthe Uniform Commercial Code, modifications do not require new consideration, the agreement is nevertheless unenforceable, as it violates the Statute of Frauds. Choice (B) is incorrect, because promissory estoppel is a substitute for missing consideration. Since consideration is not needed here, promissory estoppeL does not apply. Choice (D) is not the best choice. Unless the owner had reason to know of the computational error at the time the contract was formed, no relief would be granted for the pubLisher’s unilateral mistake.
How well did you know this?
1
Not at all
2
3
4
5
Perfectly
9
Q
  1. An employee successfully negotiated a lucrative contract for her employer. As a result, her employer orally promised her a $10,000 bonus payable at the end of the year because of the employee’s “good work.” At the end of the year, the employer informed the employee that the company’s profits were not as large as he expected, so the promised bonus would not be paid.
    Which of the following is the legal effect of the employer’s promise to pay the bonus to the employee?
    (A) It is enforceable, because the employee conferred a material benefit on the employer by negotiating the lucrative contract.
    (B) It is enforceable, because the employer was morally obligated to pay the bonus.
    (C) It is unenforceable, because it was not supported by legally sufficient consideration.
    (D) It is unenforceable, because it was not in writing.
A
  1. (C) Donative promises generally are not enforceable unless supported by consideration or a consideration substitute (like promissory estoppel). Here, the employer’s promise was made in exchange for work already performed by the employee. Past consideration is not considered to be good consideration, so the empLoyer’s promise to pay the bonus would be unenforceable. Choice (A) is therefore wrong, because no present material benefit was conferred on the employer in exchange for the bonus. Choice (B) is incorrect. While moral obligation can (in some jurisdictions) make a promise supported by past consideration enforceable, this is generally only done when necessary to prevent unjust enrichment or undue hardship, neither of which would be the case here. Choice (D) is wrong, because there would be no reason why the promise would need to be in writing. The duration of the promise would not be for longer than one year, and the only time the amount of the promise can trigger the need for a writing is when the contract involves a sale of goods.
How well did you know this?
1
Not at all
2
3
4
5
Perfectly
10
Q
  1. A store published the following advertisement in a local newspaper on Monday, March 12:
    “8 Brand New COWBOY HATS Beaver Felt,
    selling for $72.50 … out they go … Sat. March 17,
    Each… $5.
    1 Navajo Turquoise Necklace … worth $125, now selling for $40.
    “FIRST COME, FIRST SERVED”
    On the following Saturday, a man was the first person to arrive at Store and demanded the necklace. The store clerk refused to sell it to him, because it was a “house rule” that the necklace was intended for women only.
    If the man brings suit against the store for its refusal to sell him the necklace, the man will
    (A) lose, because the advertisement was intended only as an invitation to make an offer.
    (B) lose, because the man did not notii’ the store in writing that he intended to accept the offer.
    (C) win, because the advertisement should be construed as a binding offer.
    (D) win, because it is immaterial whether the man was the first customer to appear at the store to purchase the necklace.
A
  1. (C) As a general rule, advertisements for the sale of goods, circular letters, price Lists, and articles displayed on a shelf with a price tag are construed as preliminary proposals inviting offers. However, in certain situations an advertisement for the sale of goods may constitute an offer. In the case of Lefkowitzv. Great Minneapolis Surplus Store [86 N.W.2d 689 (1957)], the court held that an advertisement in a newspaper proposing the sale of a coat “FIRST COME, FIRST SERVED” did in fact constitute an offer because the language in the ad indicated a promise to sell and indicated a quantity of one. Therefore, the store’s advertisement would be considered an offer that was accepted by the man by being the first person to arrive and request to purchase the necklace. Choice (A) is therefore wrong. Choice (B) is wrong, because there would be no reason why the man would have to provide written notice of an intent to accept. The advertisement said nothing about providing notification in writing. Choice (D) is incorrect, because being the first to arrive was a condition of acceptance indicated by the language “First Come, First Served.” Therefore, if the man was not the first customer to appear at the store to purchase the necklace, he would have been unable to validly accept the offer.
How well did you know this?
1
Not at all
2
3
4
5
Perfectly
11
Q
  1. A man knew that his neighbor frequently earned extra money by mowing lawns in the area. On Wednesday, the man slipped a note under his neighbor’s door, which said:
    “If you will mow my lawn by Saturday I will pay you $25.”
    The neighbor mowed the lawn Friday afternoon, but the man refused to pay the $25.
    The court, in evaluating the relationship between the man and his neighbor, would most probably find that
    (A) the neighbor’s mowing of the lawn created a bilateral contract.
    (B) the neighbor’s mowing of the lawn created a unilateral contract.
    (C) the note slipped under the door was an acceptance of a standing offer by the neighbor.
    (D) the neighbor is only entitled to recover in quasi-contract for the reasonable value of the mowing of the lawn.
A
  1. (B) The note slipped under the door created an offer for a unilateral contract, one that is accepted by performance of the requested act. The neighbor’s mowing of the lawn therefore created a unilateral contract. Choice (A) is wrong, because a bilateral contract involves an exchange of promises. The note the man slipped under the door did not request a return promise but rather the performance of an act, the mowing of the lawn. Choice (C) is incorrect, because there is no indication that the neighbor made such a standing offer. The facts state only that the neighbor had mowed lawns in the past. This does not mean that he made a standing offer to mow any lawn in the future. Choice (D) is incorrect, because the neighbor would not need to resort to quasi-contract to recover. Quasi-contract is a contract implied at law to prevent unjust enrichment when there is not an enforceable contract between the parties. Here, an enforceable contract was formed when the neighbor mowed the lawn, so quasi-contract would not be needed.
How well did you know this?
1
Not at all
2
3
4
5
Perfectly
12
Q
  1. On Wednesday morning the following conversation took place:
    A man: “My stereo speakers haven’t been sounding good lately. The owner of a stereo store promised to give me $15 for them, and I think I’ll take him up on the offer.”
    A woman: “Don’t do that. In my spare time, I repair stereo speakers. If you promise to pay me $20, I promise to repair them by next Tuesday and they’ll be in tip-top condition.”
    The man then handed his speakers and $20 to the woman.
    The conversation and events on Wednesday resulted in
    (A) a contract for the sale of services governed by the UCC.
    (B) a unilateral contract.
    (C) a bilateral contract.
    (D) an unconscionable contract.
A
  1. (C) A bilateral contract is a contract in which mutual promises are given as the agreed exchange for each other. In a bilateral, or two-sided contract, each party promises a performance, so each party is both a promisor as to his own promise and a prom isee as to the other’s promise. On the other hand, a unilateral contract is a contract in which a promise is given in exchange for an actual performance by the other party. Here, the woman asked for a return promise from the man, thereby calling for a bilateral contract. The man did not expressly issue a promise to the woman, but instead performed his end of the deal. Performance by the offeree in the presence of the offeror is deemed to create an implied promise that accepts the offer for a bilateral contract. Therefore, a bilateral contract was formed between the parties. Choice (B) is therefore wrong. Choice (A) is wrong, because the Uniform Commercial Code governs sale of goods contracts, not contracts for services. Choice (D) is incorrect, because there is nothing in the facts to even suggest that the deal between the parties should be considered unconscionable.
How well did you know this?
1
Not at all
2
3
4
5
Perfectly
13
Q
  1. A man needed to have the oil changed on his car. On Friday, he decided to take his car to the local dealership to have the oil changed and asked his neighbor if she would give him a ride home from the dealership. The neighbor said, “Why pay the high prices a dealership will charge you? I can change the oil in your car for you. If you will agree to pay me $50, I’ll change the oil in your car over the weekend.” The man readily agreed.
    On Sunday afternoon, the man noticed that his neighbor still had not started working on the car. He asked his neighbor if the car would be ready for him to drive to work Monday morning. The neighbor replied, “I thought about it and realized $50 is too low a price for the work involved. I don’t think I’m going to change the oil in your car.” The man then said, “Look, I realize $50 is low for the work involved. If you can change the oil in my car by tomorrow morning, I’ll pay you an additional $25. And I won’t sue you in small claims court for your failure to perform your promise.” The neighbor then changed the oil late Sunday afternoon, but the man refused to pay to the neighbor anything more than $50.
    In a suit by the neighbor to recover the additional $25 promised by the man, the neighbor will
    (A) win, because she performed her part of the bargain.
    (B) win, because the second contract for $75 superseded the original $50 contract.
    (C) lose, because the $75 contract did not supersede the $50 contract.
    (D) lose, because the neighbor had a pre-existing duty to change the oil in the car for $50.
A
  1. (D) Underthe pre-existing duty rule, neither doing nor promising to do that which one is already legally bound to the promisorto do can furnish consideration for a promise. In such case, neither benefit to the promisor nor detriment to the promisee exists. Thus, the neighbor was under a pre-existing duty to change the oil in the man’s car. Consequently, the man’s subsequent promise to pay higher compensation would be unenforceable. Choices (A) and (B) are incorrect, because there was therefore no consideration to enforce the revised agreement. And note that the man’s promise to forbear to sue would not still furnish the necessary consideration, because it was not given for a bargained-for exchange. Choice (C) is not the best answer. While it is technically a correct statement, it is not as specific as Choice (D), which states why the new agreement did not supersede the first. A more specific answer is always a better choice over a more general answer.
How well did you know this?
1
Not at all
2
3
4
5
Perfectly
14
Q
  1. A woman needed to have her microwave repaired. She contacted the local handyman, who said he could repair the microwave for $100. The woman readily agreed and delivered the microwave to the handyman, who promised to have it ready in two weeks.
    One week later, the handyman realized that he had so much work to do that he would not be able to repair the microwave on time. He then took the microwave to a repair store. The repair store agreed to repair the microwave for $80 within one week. The owner of the repair store that contracted with the handyman was unaware that the woman actually owned the microwave.
    If the repair store fails to repair the microwave, which of the following is the most accurate statement?
    (A) The woman has a cause of action against the repair store only.
    (B) The woman has a cause of action against the handyman only.
    (C) The woman has a cause of action against both the repair store and the handyman.
    (D) The woman has no cause of action against either the repair store or the handyman.
A
  1. (B) It should be emphasized that a delegation of duties will not discharge the delegator’s, herein the handyman’s, Liability under the terms of the original contract. Only a novation assented to by the obligee will have this effect. In failing to make the necessary repairs on the microwave, the handyman breached his duty of performance. Therefore, he would be liable for breach. Choices (A) and (C) are wrong, because the woman would not have a cause of action against the repair store, since the repair store was unaware of the existence of the woman and never assumed the obligation to perform to the woman. A detegatee is not liable to the obligee unless the delegatee has assumed the obligation to perform. Choice (D) is incorrect, because the woman would have a cause of action against the handyman.
How well did you know this?
1
Not at all
2
3
4
5
Perfectly
15
Q
  1. A debtor owed a creditor $750 on an old debt. On July 1, the debt was barred by the statute of limitations. On August 1, the debtor ran into the creditor at a party and overheard him telling mutual friends that the debtor “is a deadbeat who weiches on his debts.” Feeling pangs of guilt, the debtor approached the creditor and orally agreed to pay him the $750 debt on September 1. The debtor refused to pay the creditor the $750 as promised on September 1St.
    If the creditor sues the debtor to recover the $750 debt, which would provide the strongest grounds that the debtor’s oral promise was unenforceable?
    (A) It was not supported by new consideration.
    (B) It was violative of the statute of frauds.
    (C) The debt was already barred by the statute of limitations.
    (D) There was no mutuality of obligation.
A
  1. (B) An express promise by a debtor to pay a debt barred by the statute of limitations or by a decree in bankruptcy is legally enforceable without new consideration. The promise is supported by the past consideration of the unpaid debt, which is still operative to give validity to the new promise. Although the debtor’s promise is enforceable, here the testmaker wants to know what is the strongest grounds that the promise is unenforceable. This is typical of the mental gymnastics employed on the Multistate. By process of elimination, Choices (A) and (D) are incorrect, because it is well established that a debt created in the past is sufficient consideration for a subsequent promise to pay it. Choice (C) is wrong, because an express promise by a debtor to pay a debt barred by the statute of limitations is legally enforceable without new consideration. Thus, Choice (B) is correct, because most states require the promise (to pay a contractual debt barred by the statute of limitations) to be in a signed writing.
How well did you know this?
1
Not at all
2
3
4
5
Perfectly
16
Q
  1. A builder wanted to have security systems installed in a series of homes he was building. He contacted several companies and asked them to submit bids for the installation work. An alarm company decided to submit a bid and, in turn, requested bids from several wholesalers for the burglar alarms it planned to use if it was awarded the job. A supplier submitted a bid to the alarm company that the latter used in computing the bid that it was preparing for the builder.
    On September 1, the alarm company sent the builder its bid, in which it proposed to install the security systems for $100,000. On September 9, the supplier notified the alarm company that it would be unable to supply any burglar alarms to them. On September 11, the builder sent the following fax to the alarm company: “I hereby accept your offer to install the security systems for $100,000.”
    The alarm company had to pay another wholesaler $10,000 above the price quoted by the supplier for the burglar alarms. As a result, the alarm company advised the builder that the total price for the job would have to be increased to $10,000. The builder replied that he would hold the alarm company to the initially agreed price of $100,000. The alarm company installed the security systems, but the builder has not yet paid them anything.
    In an action by the alarm company against the builder for services rendered, the alarm company will probably be able to recover
    (A) $100,000, because that was the contract price. (B) $110,000 because of an unanticipated change
    of circumstances after the parties had entered into their contract.
    (C) only in quantum meruit, because of the doctrine of commercial frustration.
    (D) only in quantum meruit, because by demanding $110,000 the alarm company repudiated its contract with the builder.
A
  1. (A) Here, students must recognize that the alarm company is obligated under the terms of its contract with the builder to install the security system for $100,000. The mere fact that the supplier refused to perform would not excuse the alarm company from its duties under the agreement with the builder. By agreeing to perform the installation work, the alarm company assumed the risks attendant with producing that result. Choice (B) is therefore wrong, because the alarm company has no right to raise the price they were charging the builder to do the requested work. Choices (C)and (D) are incorrect, because there is still an enforceable contract between the parties. Quantum meruit is a remedy that is awarded onLy when there is not an enforceabLe contract and the court needs to imply the existence of a contract at law to avoid unjust enrichment.
How well did you know this?
1
Not at all
2
3
4
5
Perfectly
17
Q
  1. A contractor learned that a city intended to open a new grammar school and was going to ask for bids to construct the school. The contractor decided to submit a bid to do the construction. The contractor contacted all of the subcontractors she had worked with in the past, informed them of the specifics of the school construction project, and asked each to submit a bid for the work they would be requested to perform. An insulation company submitted a bid of $25,000 to do the required insulation work in the new school. Based on that and other subcontract bids, the contractor prepared a general bid and submitted it to the city.
    Three days after the contractor submitted the bid to the city, the insulation company notified the contractor that it had overbooked its workforce and would be unable to perform the insulation work. The next day, the city notified the contractor that she had won the bid to build the school. The contractor was forced to find another company to do the insulation work. The other company charged the contractor $30,000 to do the insulation.
    Which of the following arguments best supports the claim for $5,000 by the contractor against the insulation company?
    (A) The contractor had made an offer to the insulation company that the latter accepted when it submitted its bid.
    (B) The insulation company had made an offer that the contractor accepted by using the insulation company’s bid in computing the bid it submitted to the city.
    (C) The insulation company’s bid was an offer that it was obligated to hold open, because the insulation company and the contractor were merchants.
    (D) An option contract was created, because the contractor used the insulation company’s bid in computing the bid it submitted to the city and notified the insulation company of that fact.
A
  1. (D) When a general contractor, about to submit a bid on a construction project, secures a bid from a subcontractor for a definite part of the proposed work, and uses the bid to determine that part of her cost, she often finds after the principal contract is awarded to her that the subcontractor refuses to go through with the job. She must then find another to do the job, usually at a price much higher than the promised figure. Can she recoup her loss from the defaulting subcontractor? Yes, the subcontractor is bound under the doctrine of promissory estoppel. Thus, a promisor who induces substantial change of position by the promisee in reliance on the promise is estopped to deny its enforceability as lacking consideration. The reason for the doctrine is to avoid an unjust result. Choice (D) is therefore correct, because Restatement of Contracts, 2d, Section 87(2) provides: “An offer which the offeror should reasonably expect to induce action or forbearance of a substantial character on the part of the offeree before acceptance and which does induce such action or forbearance is binding as an option contract to the extent necessary to avoid injustice.” Choice (A) is incorrect, because the initial communication from the contractor was not an offer but just a request for the submission of bids. It was the response by the subcontractor that was the offer. Choice (B) is incorrect, because the act of using the subcontract bid in the general bid was not an acceptance of the offer. Acceptance would take place when the contractor is awarded the bid and then so notifies the subcontractor. Choice (C) is incorrect, because the fact that the parties might be merchants is only relevant when dealing with a sale of goods, since the Uniform Commercial Code has some special rules applicable when one or both of the parties are merchants.
How well did you know this?
1
Not at all
2
3
4
5
Perfectly
18
Q
  1. On February 15, a company that manufactures metal sidings for home exteriors received the following order from a builder: “Please ship 300 sheets of 1/4-inch refabricated aluminum siding. Delivery by April 1.”
    On March 8, the company shipped 300 sheets of 1/2-inch refabricated aluminum siding, which were received by the builder on March 10. The following day, the builder sent the following fax to the company: “Be advised that your shipment is rejected. Order stipulated 1/4-inch sheets.” This fax was received by the company, but the builder did not ship the nonconforming aluminum sheets back to the company.
    Did the builder properly reject the shipment delivered on March 10?
    (A) Yes, because the aluminum sheets were nonconforming goods.
    (B) Yes, because the company did not notify the builder that the 1/2-inch sheets were for accommodation only.
    (C) No, because the builder waived its right to reject the nonconforming goods by not returning them promptly to the company.
    (D) No, because the company could accept the builder’s offer by prompt shipment of either conforming or nonconforming goods.
A
  1. (A) UCC Section 2-601 provides that if the goods or the tender of delivery fail in any respect to conform to the contract, the buyer may 1) reject the whole; or 2) accept the whole; or 3) accept any commercial unit or units and reject the rest. Choice (A) is correct, because the March 10 shipment of 1/2” refabricated aluminum siding was nonconforming, since the contract called for 1/4” siding. Therefore, in accordance with subsection (a) above, the builder properly rejected the nonconforming shipment. Choice (B) is wrong, because the builder still could have rejected the shipment even if a notice of accommodation was included. The notice of accommodation would have prevented the shipment from constituting a breach of the contract. Choice (C) is incorrect, because the UCC does not impose upon the buyer a duty to return nonconforming goods. It is the seller’s obligation to retrieve the goods or provide instructions to the buyer as to how the goods are to be returned. While Choice (D) is a correct statement of the law, it does not address the relevant issue being tested in the question.
How well did you know this?
1
Not at all
2
3
4
5
Perfectly
19
Q
  1. On September 15, a card shop sent the following fax to a printing company: “Please deliver 100 dozen assorted Christmas cards, delivery by November 1.”
    On October 10, the printing company shipped 100 dozen assorted Thanksgiving cards to the card shop, which were received on October 12. The following day, the card shop sent the following fax to the printing company: “Be advised that your shipment is rejected. Order stipulated Christmas cards, not Thanksgiving cards.”
    On October 15, the printing company sent the following fax to the card shop: “Will ship 100 dozen assorted Christmas cards by November 1. Please ship the Thanksgiving cards back to our warehouse and bill us for the shipping charges.” This fax was received by the card shop, but the card shop did not respond to it or ship the Thanksgiving cards back to the printing company. On October 25, the printing company attempted to deliver 100 dozen assorted Christmas cards to the card shop, but the latter refused to accept.
    Did the card shop properly reject the October 25 delivery?
    (A) No, because under the UCC a contract for the sale of goods can be modified without consideration.
    (B) No, because the printing company cured the October 10 defective shipment by its tender of conforming goods on October 25.
    (C) Yes, because the printing company’s shipping of the Thanksgiving cards on October 10 constituted an anticipatory breach.
    (D) Yes, because the printing company’s shipping of the Thanksgiving cards on October 10 constituted a present breach of contract.
A
  1. (B) In accordance with UCC Section 2-508, “where anytenderordeliverybythe seller is rejected because nonconforming and the time for performance has not yet expired, the seller may seasonably notify the buyer of his intention to cure and may then within the contract time make a conforming delivery.” Therefore, choice (B) is correct, because UCC 2-508 permits a seller who has made a nonconforming tender in any case to make a conforming delivery within the contract time upon reasonable notification to the buyer. Choices (C) and (D) are therefore wrong, because the printing company cured any breach by shipping the conforming goods on October 25. Choice (A) is not the best answer. While it is true that a contract for the sale of goods can be modified without consideration, this was not a modification, which is an agreement to change the terms of the contract. No such agreement was ever formed between the parties.
How well did you know this?
1
Not at all
2
3
4
5
Perfectly
20
Q
  1. On February 1, a man dispatched the following letter to a mechanic:
    “My car has not been running very well lately. I’ll pay you $275 if you will change the oil, replace the oil filter, and adjust the carburetors by February 10.”
    The mechanic received the man’s letter on February 3. That same day, he telephoned an auto supply company and ordered the necessary materials to perform the repair work. Two days later, the mechanic met the man at a party and this conversation took place:
    The man: “Disregard the letter I sent you last week.”
    The mechanic: “No way, man, I already ordered the materials on from the auto supply company.”
    The man: “Sorry, but I sold my car yesterday, so forget the repair work.”
    If the mechanic initiates suit for breach of contract, which of the following is the man’s strongest argument that no enforceable contract was formed between the parties?
    (A) The mechanic had not completed performance before the man revoked his offer.
    (B) The man’s offer could only be accepted by a return promise.
    (C) Because the man made his offer by letter, the mechanic could accept only in the same manner.
    (D) Although the mechanic was preparing to perform the repair work, he had not begun the requested acts of acceptance when the man revoked his offer.
A
  1. (D) It is important to note that when dealing with an offer for a unilateral contract (as is the case here), part of the actual performance requested must have been given in order to render the offer irrevocable. Mere preparation for performance, no matter how detrimental to the offeree, will not affect the offeror’s power and privilege to revoke a unilateral offer. Note that an offer that invites performance of an act as acceptance, rather than a return promise, becomes irrevocable as soon as the offeree has started to perform the act. However, students must be aware that where a clearly unilateral offer (as in the present example) calls for several acts, it may be interpreted as inviting acceptance by completion of the initial act, performance of the balance being regarded as conditions merely to the offeror’s duty of performance. As a consequence, Choice (A) is incorrect, because where the unilateral offer calls for several acts (e.g., 1) change the oil, 2) replace the oil filter, and 3) adjust the carburetors), an option contract is completed when the offeree has performed one of the requested acts. Thus, in the latter situation, the offeree need not render completed performance (of all the requested acts) in order to recover for breach of contract (where there is a wrongful revocation on the part of the offeror). Choice (B) is incorrect, because a unilateral offer is accepted by a return performance, not a return promise (as in the case of a bilateral contract). Similarly, Choice (C) is incorrect, because a unilateral offer cannot be accepted by communicating a return promise, but rather by completing performance of the requested act(s).
How well did you know this?
1
Not at all
2
3
4
5
Perfectly
21
Q
  1. A woman leased a condo from the owner for a period of one year. After six months, the owner gave the woman a written option to purchase the condo for $100,000 before the expiration of the lease. With the owner’s approval, the woman spent $10,000 to have the kitchen remodeled to her personal specifications. One month before the end of the lease, the owner notified the woman that he was revoking the option to purchase the condo. One week later, the woman delivered a written notice of acceptance of the option, but the owner refused to sell.
    If the woman initiates suit for breach of contract, which of the following is her strongest argument that an enforceable contract was formed between her and the owner?
    (A) Because the woman had until the expiration of the lease to accept the offer, the owner’s revocation would be ineffective.
    (B) Because the owner was a merchant, the written offer was irrevocable for a period not exceeding three months.
    (C) Because the owner’s offer invited a return promise as acceptance, the woman manifested her intent to accept by remodeling the kitchen.
    (D) After the woman paid to have the kitchen remodeled, an option contract resulted, because the owner knew the woman was relying on the offer to her detriment.
A
  1. (D) Under the Restatement view, a promisor who induces substantial change of position by the promisee in reliance on the promise is estopped to deny its enforceability as lacking consideration. The reason for the doctrine is to avoid an unjust result. Choice (D) is therefore correct, because Restatement of Contracts, 2d, Section 87(2), provides: “An offer which the offeror should reasonably expect to induce action or forbearance of a substantial character on the part of the offeree before acceptance and which does induce such action or forbearance is binding as an option contract to the extent necessary to avoid injustice.” Choice (A) is incorrect, because an offer can be revoked prior to acceptance unless the offer was irrevocable. Simply promising that an offer would be open for a period of time is not enough to make an offer irrevocable. Choice (B) is wrong, because the merchant’s firm offer rule applies only to a sale of goods. Choice (C) is wrong, because the remodeling of the kitchen was not a manifestation of acceptance, but rather evidence of detrimental reliance on the offer.
How well did you know this?
1
Not at all
2
3
4
5
Perfectly
22
Q
  1. On Monday, a man told a gardener, “I am having a party on Sunday and I want my house to look good. If you will promise to mow my lawn by Saturday, I will pay you $50.” On Friday, the gardener arrived at the man’s home just as the man was leaving for work and began to mow the man’s lawn. The man said nothing to the gardener but drove off as he saw the gardener unloading his lawn mower. When the man arrived home from work that evening, he noticed that only half of his lawn had been mowed. He then found a note from the gardener slipped into his mailbox. The note said:
    “Sorry, but I ran out of gas to power the lawn mower and did not have time to buy more gas to finish the job. I’m taking the weekend off, but I will be back Monday morning to finish the job.”
    If the man brings suit against the gardener for breach of contract, who is likely to prevail?
    (A) The gardener, because he never accepted the offer made by the man.
    (B) The gardener, because he offered to cure the defective performance by finishing the job on Monday morning.
    (C) The man, because the gardener’s part performance necessarily implied an acceptance and a promise that he would render complete performance.
    (D) The man, because under the doctrine of equitable estoppel, the gardener’s part performance was evidence of his intent to honor the entire contract.
A
  1. (C) As a general rule, where the offeree (the gardener) begins the performance contemplated, he thereby impliedly promises to complete it. However, in order that the act of part performance may be treated as implying a promise to complete, the following requirements must be present. First, the offer was for an entire contract, and not for a series of separate contracts. Second, that what is begun must be a part of the actual performance bargained for, and not mere preparation for performance. Third, that such implied acceptance is communicated to the offeror, or he had knowledge of it. Since the man saw the gardener beginning to mow his lawn, the gardener will have impliedly promised to mow the lawn by Saturday, and his failure to do so would be a breach. Choice (A) is therefore incorrect, because the implied promise was the acceptance of the offer. Choice (B) is incorrect for two reasons: First, cure is a concept recognized by the UCC and therefore applies only to a sale of goods contract. Second, even if cure was allowed, a cure must take place before the date performance was due under the contract. Since the contract called for the lawn to be mowed by Saturday, waiting until Monday to finish the job would be a breach. Choice (D) is wrong, because estoppel is not needed to make the contract enforceable.
How well did you know this?
1
Not at all
2
3
4
5
Perfectly
23
Q
  1. An art collector attended a party on March 15. At the party, the art collector was describing his collection to a woman in attendance. When the art collector described a painting by a well-known artist, the woman indicated she might like to buy that painting. The art collector said, “I’ll sell you the painting for $10,000. I’ll give you 30 days to decide whether you want the painting.” On March 25, the art collector wrote to the woman and stated that the offer of March 15 was withdrawn. The woman received the March 25 letter on March 26. On March 27, the woman wrote the art collector the following letter:
    “Please be advised that I hereby accept your offer of March 15.” The art collector received this letter on March28.
    Thereafter, the art collector and the woman engaged in several telephone discussions. On April 10, the woman, in a telephone conversation, told the art collector that she would pay $15,000 if the painting was delivered on or before April 15. The art collector agreed to deliver the painting for $15,000.
    On April 15, the art collector tendered the painting, but the woman refused to pay more than $10,000.
    If the art collector asserts a claim against the woman for breach of contract, which of the following is the most accurate statement?
    (A) The art collector is obligated to sell the woman the painting for $10,000, because the woman, as offeree, had the right to accept the initial offer within the 30-day period.
    (B) Since the art collector, as offeror, had the power to revoke the original offer before acceptance, the woman is obligated under the terms of their April 10 agreement.
    (C) Since the parties entered into a subsequent modification, the woman is obligated to pay the art collector $15,000 for the painting.
    (D) An enforceable contract does not exist between the parties, because of mutual mistake of fact.
A
  1. (B) The woman is obligated under the terms of the April 10 agreement. The art collector effectively revoked his original offer to sell the painting to the woman in the written communication of March 25. The general rule provides that the offeror may, at any time before acceptance, terminate his offer by revoking it. This is true even though the offeror has promised not to revoke for a stated time, unless the promise is 1) under seal or 2) fora consideration. Choice (A) is incorrect, since the original offer was not supported by consideration, e.g., an option contract was not created. In addition, even though the contract involved a sale of goods and the art collector was arguably a merchant, the March 15 offer would not be a merchant’s firm offer, since a merchant’s firm offer must be in writing and this offer was oral. Choice (C) is wrong, because the subsequent agreement cannot be a modification, since no contract had previously been formed. Choice (D) is wrong, because there was no mutual mistake ever made.
How well did you know this?
1
Not at all
2
3
4
5
Perfectly
24
Q
  1. A debtor owed a creditor $15,000 on a debt that had been discharged by the debtor’s bankruptcy the previous year. The debtor wrote a letter to the creditor stating that he would pay the creditor $10,000 received from the proceeds of the sale of his house in payment of the discharged debt. One week later, the debtor learned that the person who had contracted to buy his house reneged on the deal. As a result, the debtor refused to pay anything to the creditor.
    If the creditor sues the debtor for breach of contract, he should be entitled to recover
    (A) nothing.
    (B) $10,000.
    (C) $10,000, only if the debtor is successful in suing the person who had contracted to buy his house.
    (D) $15,000.
A
  1. (B) The creditor will be entitled to $10,000 from the debtor as per the debtor’s letter. The Restatement of Contracts 2d, Section 83, states that “an express promise to pay all or part of an indebtedness of the promisor, discharged or dischargeable in a bankruptcy proceeding begun before the promise is made, is binding.” Thus, the debtor’s promise is enforceable without consideration. Choice (A) is therefore wrong. Choice (C) is incorrect, because the debtor’s promise was not expressly conditioned on successfully suing the buyer of his house. A conditional promise is usually prefaced by such words as “provided that,”“if,”“on condition that,” etc. Choice (D) is incorrect, because a new promise is enforceable only as to the extent of the new promise. Therefore, the creditor can only enforce the new promise to pay $10,000, not the discharged obligation to pay $15,000.
How well did you know this?
1
Not at all
2
3
4
5
Perfectly
25
Q
  1. A woman was hired by a restaurant as a cashier under an at-will employment contract. On the woman’s first day at work, she noticed a sign in the kitchen area that read:
    “IMPORTANT NOTICE TO ALL EMPLOYEES
    Employees are not permitted to smoke during working hours. Any employee who is found to be in violation of this policy will be fined $50 for the first offense; $100 for the second offense; and fired for the third violation.”
    The woman, who was a smoker, read the notice, but did not object or make any comment regarding the restaurant’s nonsmoking policy. For six months, the woman worked at the restaurant and never smoked during business hours. One afternoon, the woman was working when an armed robber stormed into the restaurant. He pointed a pistol at the woman and demanded all the money from the cash register. Frightened and panic stricken, the woman handed over the money and then collapsed. Moments later, she regained consciousness and then smoked a cigarette while she regained her composure. Thereafter, the woman resumed her duties for the rest of the day.
    The next week, however, when the woman received her pay check, she noticed that the restaurant had deducted $50 from her check. A note was attached indicating that the woman was being fined for smoking during business hours. Although the woman protested, the restaurant refused to make any waiver and stood by its policy.
    In an action by the woman against the restaurant to recover the $50, which of the following is the best argument in the woman’s favor?
    (A) The restaurant’s nonsmoking policy concerned a collateral matter that was not incorporated within the terms of their employment contract.
    (B) The restaurant impliedly waived the nonsmoking provision by permitting the woman to continue working for the rest of the day.
    (C) The nonsmoking provision constituted a constructive condition subsequent that was excused because of temporary impracticability.
    (D) The nonsmoking provision concerning disciplinary action is unenforceable, because it attempts to impose a penalty instead of reasonably liquidating the damages, if any, sustained by the restaurant.
A
  1. (D) By process of elimination, Choice (A) is incorrect, because the restaurant’s nonsmoking policy concerns an issue that is not a collateral matter. In today’s society, many companies are concerned about the health-related consequences of smoking. As such, the restaurant’s nonsmoking policy deals with an important matter. Choice (B) is incorrect, because permitting the woman to continue working did not necessarily constitute a waiver of the nonsmoking provision. Choice (C) is wrong, because there is no impracticability, which would be the creation of an extreme hardship to complete performance of the contract. As a result, the woman’s best argument is that the disciplinary action taken by the restaurant is in the form of a penalty and is in excess of any actual loss suffered by the restaurant. Such a penalty would be an unenforceable liquidated damages provision.
How well did you know this?
1
Not at all
2
3
4
5
Perfectly
26
Q
  1. A 25-year-old concert pianist lived in a small studio apartment in a city. The pianist could not keep a piano in her apartment because of its small dimensions. In order to practice each day, she had to travel to a school of performing arts to use its musical facilities. Finally, the pianist decided to move out of her apartment and buy a more spacious home, where she could have her own piano. As she was house hunting, the pianist found a quaint home in the county that she wanted. She put a bid down on the home with the seller’s broker. She was informed that the owner was in Europe on a business trip and would not be entertaining any offers until he returned.
    While she was awaiting word on the county property, the pianist’s friend orally agreed to sell his piano to her for $8,000. The pianist explained that she wanted the piano only if she should succeed in her efforts to buy the county home. For this reason, the parties agreed that the piano sale would not take effect unless the pianist bought the county home. The next day, the parties reduced their oral agreement to a signed writing, but did not include in the writing any mention that the sale would not take effect unless the pianist bought the county home. Two weeks later, the owner of the county property returned from a trip and rejected the pianist’s offer.
    The friend now brings an action against the pianist for breach of contract to buy the piano. How should the court rule on the pianist’s offer to prove, over the friend’s objection, that she has not been able to buy the county home?
    (A) The evidence is admissible, to show that the written agreement did not become a contract.
    (B) The evidence is admissible, to show frustration of the purpose of the contract.
    (C) The evidence is barred, because the oral agreement is within the statute of frauds.
    (D) The evidence is barred, because the oral agreement would contradict the written agreement, which is unconditional on its face.
A
  1. (A) Generally speaking, the parol evidence rule provides that when a contract is expressed in a writing that is intended to be the complete and final expression of the rights and duties of the parties, parol evidence of prior oral or written negotiations or agreements of the parties (or of their contemporaneous oral agreements), which varies or contradicts the written contract, is not admissible. At the time the pianist and the friend reduced their agreement to a writing, they understood the piano sale would not take effect unless the pianist purchases the county home. Since the pianist did not buy the home, parol evidence is admissible to show the condition did not occur. This is an exception to the parol evidence rule where such evidence is not being admitted to vary (or contradict) the terms of the writing, but rather to show that there is not an agreement at all. That’s why choice (A) is a better answer than choice (D). Choice (B) is incorrect, because frustration of purpose covers situations where a supervening (or unforeseeable) event frustrates the purpose of the contract. Here, at the time the parties entered into their agreement, they were unsure whether the pianist would purchase the county home. Thus, it was not an unforeseeable event. Choice (C) is wrong, because the parties did reduce their agreement to a writing. However, paroL evidence is admissible to show the writing never became a contract.
How well did you know this?
1
Not at all
2
3
4
5
Perfectly
27
Q
  1. An owner operated a successful retail business. He decided he wanted to retire and listed his business for sale. A buyer saw the listing and visited the retail business. The buyer told the owner that she was very interested in buying the business, but that she needed to get a loan from the bank to finance the purchase. The parties drafted a contract, signed by both parties, which stated that the buyer agreed to purchase the retail business from the owner for $250,000. The written agreement contained a provision wherein the sale would not take effect “unless the buyer is successful in her bid to obtain a loan from a bank to finance the purchase of the business.” Subsequently, the buyer made no effort to attempt to obtain a loan from a bank so that the sale could be finalized. After several months, the owner filed suit against the buyer for breach of contract.
    Which of the following will be the owner’s strongest argument in support of his action against the buyer for breach of contract?
    (A) The obtaining of a loan from a bank was not worded as a “condition” to the buyer’s duty to buy the business.
    (B) Although obtaining a loan from a bank was a condition to the buyer’s duty to buy the business, the condition should be excused because its non-occurrence would cause a forfeiture to the owner.
    (C) Although obtaining a loan from a bank was a condition to the buyer’s duty to buy the business, it should be stricken from the contract because it is an unconscionable term.
    (D) The buyer breached an implied promise to make a reasonable effort to obtain a loan from a bank.
A
  1. (D) Where a party to a contract for an agreed exchange of performances knowingly prevents, hinders, or makes more costly the other’s performance, such conduct is a breach of contract for which an action will lie. The breach is of an implied promise against prevention. Choice (A) is wrong, because the obtaining of a loan from a bank was a condition precedent to the buyer’s duty to buy the business. Choice (B) is incorrect, because the non-occurrence of the condition would not cause a forfeiture to the owner. He could still contract elsewhere and sell the business to another buyer. Similarly, Choice (C) is erroneous, because the condition is not an unconscionable term.
How well did you know this?
1
Not at all
2
3
4
5
Perfectly
28
Q
  1. A buyer contracted to purchase a used car from a seller for $10000. On the date the sale was to take place, the buyer tendered a $10,000 cashier’s check to the seller. The seller rejected the tender and refused to deliver the car to the buyer.
    If the buyer brings an action for breach of contract against the seller, the buyer is entitled to which of the following remedies?
    (A) Damages measured by the difference between the market price and the contract price for the car.
    (B) Recovery of the contract price of the car.
    (C) Specific performance.
    (D) Recovery of the market price of the car.
A
  1. (A) Choice (A) is correct, because according to UCC Section 2-713, the measure of damages for non-delivery or repudiation by the seller is the difference between the market price at the time when the buyer learned of the breach and the contract price together with any incidental and consequential damages as provided under Section 2-715. Choices (B) and (D) are incorrect, because recovery for the contract (or market) price is inappropriate, since the buyer’s proper remedy is either to “cover” or seek damages measured by Section 2-713. Since damages at law are adequate, specific performance will not be decreed; nothing about the car suggests that the car was unique enough to warrant specific performance to the buyer. Therefore, choice (C) is incorrect.
How well did you know this?
1
Not at all
2
3
4
5
Perfectly
29
Q
  1. A third-year law student was the captain of the law school rugby club. One evening, the student and a few of his rugby teammates were drinking beer at a local pub. They were worried that the rugby club would be forced to disband because the law school had withdrawn its sponsorship. While the student was discussing the problem with his teammates, the owner of the tavern approached the players. The owner indicated that he was interested in sponsoring the rugby club because he felt it would help business at the pub. During their ensuing discussion, the owner agreed to sponsor the rugby club. The parties then orally agreed that the owner “would pay for all the usual sponsoring fees” incurred by the club.
    The owner had understood the agreement to mean that he would pay for the rugby shirts and supply the keg of beer following each “home” game that the rugby club played. Conversely, the student thought that the owner would be reimbursing the team for (a) the shirts, (b) the keg of beer (following “home” games), as well as (c) transportation expenses for “away” games and (d) equipment expenses (balls, etc.).
    Assume that it was the customary practice of the rugby teams in the community to have the sponsors pay for (a) the shirts and (b) beer only. Before the rugby club was to play its first game under the owner’s sponsorship, which of the following is the most accurate statement regarding the legal relationship between the parties?
    (A) A contract exists on the terms understood by the owner.
    (B) A contract exists on the terms understood by the student.
    (C) A contract exists on the terms that are customary for the other teams in the community.
    (D) No contract exists.
A
  1. (D) The first requisite of a contract is that the parties manifest to each othertheirmutual assent to the same bargain at the same time. In the present exampLe, since there was no “meeting of the minds” regarding the essential terms of their purported agreement, choice (D) is the best answer. Choices (A) and (B) are therefore incorrect. Choice (C) is wrong, because the customary practices would factor in when there is ambiguity or uncertainty as to the terms agreed upon. However, here there was a clear difference of opinion, so no meeting of the minds ever took place.
How well did you know this?
1
Not at all
2
3
4
5
Perfectly
30
Q
  1. A man entered a local retail store and approached the proprietor. The man explained that he was participating in a local run for charity and wondered if the proprietor would be interested in sponsoring him. The proprietor believed the publicity would be good for his business and agreed, provided the man would wear a shirt with the retail store’s logo on it during the run. The man readily agreed.
    The proprietor thought that sponsorship meant paying a set fee for every mile covered during the charity run. However, the man expected the proprietor, in addition to paying the fee for each mile coverçd during the run, to also pay his entrance fee and be in attendance at the finish line to greet the man when he finished the charity run.
    Assume that the man and the proprietor entered into an enforceable contract. Which of the following, if true, would not help to establish that the manifestations of the parties were operative according to the meaning adjudged by the proprietor?
    (A) The customary practice of the other chrity runs in the community is the same as the proprietor’s understanding, that the sponsor be responsible for paying only a set fee for each mile covered during the charity run.
    (B) A reasonably objective individual would have attached the same meaning to the manifestations of the parties as did the proprietor.
    (C) At the time the agreement was entered into, the man had reason to know the proprietor’s understanding of what the parties meant, although the proprietor did not know the man’s understanding was different from his.
    (D) The proprietor subjectively believed that the man understood that the proprietor would only be obligated to pay a set fee for each mile covered during the charity run.
A
  1. (D) With regard to contract interpretation, when there is no integration (as in the present hypo), the standard is the meaning that the party making the manifestation should reasonably expect the other party to give it. This is commonly referred to as the standard of reasonable expectation, which is based primarily upon the “objective theory of contracts.” Under this rule, the test is to determine what a reasonable (or objective) person in the position of the person hearing the representations would have concluded. So if a reasonable individual in the proprietor’s position would have attached the same meaning that the proprietor did after hearing the man’s representations, then the meaning would be the one that the hearer attached to it. Since Choice (B) therefore does help the proprietor, it is incorrect. Moreover, since the test is one of what a reasonable person would believe the manifestations to mean, the proprietor’s subjective belief of the man’s meaning is irrelevant if it were unreasonable to believe so. For this reason, Choice (D) is the correct choice, because this argument will not help the proprietor. Choice (C) is also incorrect, because where there is an ambiguity, when one party knows or has reason to know of the ambiguity and the other does not, there is a contract based upon the meaning of the party who was unaware of the ambiguity. So if the proprietor is able to prove that the man knew that the proprietor’s understanding did not match the man’s, this will help the proprietor and make Choice (C) incorrect. In addition, in cases of ambiguity, courts may resort to extraneous aids, such as “course of dealing,”“trade usage,” or “customary practice,” in the interpretation of the agreement. Therefore, Choice (A) will be of help to the proprietor and is also incorrect.
How well did you know this?
1
Not at all
2
3
4
5
Perfectly
31
Q
  1. A manufacturer of widgets entered into a written agreement to deliver 500 widgets to a buyer. The contract provided that the widgets would be shipped C.O.D. The manufacturer subsequently delivered 490 widgets, which were accepted and paid for by the buyer.
    If the buyer brings suit for breach of contract against the manufacturer, the buyer will most likely
    (A) not recover, because under the circumstances the manufacturer substantially performed.
    (B) not recover, because the buyer accepted delivery of the 490 widgets.
    (C) recover, because the manufacturer failed to perform his contractual obligation.
    (D) recover, because the manufacturer’s failure to deliver the additional widgets resulted in a material breach.
A
  1. (C) Students should take note that the manufacturerwas under a contractual obligation to deliver 500 widgets to the buyer. Since the manufacturer delivered only 490 widgets, he failed to perform part of what was promised in the contract. In this regard, any unjustified failure to perform when performance is due is a breach of contract, which entitles the injured party to damages. If the breach is slight or insubstantial, it is called a partial breach, for which plaintiff’s damages are restricted to compensation for the defective performance. Choice (A) is wrong, because the doctrine of substantial performance does not apply to contracts for the sale of goods under the UCC. Choice (B) is wrong, because acceptance of the goods obligates the buyer to pay for those goods accepted, but does not prevent the buyer from asserting rights as to the goods that were not delivered. Choice (D) is incorrect, because any tender or deLivery that does not conform exactly to the contract is considered to be a material breach.
How well did you know this?
1
Not at all
2
3
4
5
Perfectly
32
Q
  1. A woman is engaged in the retail sale of widgets throughout the United States. On March 7, the woman sent the following purchase order to a manufacturer of widgets:
    “Promptly ship 1,000 widgets, catalogue #B4-IEU, at the current wholesale price to our warehouse. Thank you for your attention to this matter.”
    This order was received by the manufacturer on March 9. The next day, the manufacturer replied by fax:
    “Your order has been received. Shipment date will be March 12. Price $50 for each widget delivered. Be advised that these are the last widgets, under catalogue #B4-IEU, that we can deliver, since this variety is no longer being manufactured.”
    Upon receipt of the fax, the woman immediately sent the following fax:
    “Cancel our previous order for 1,000 widgets under catalogue #B4-IEU. Price too high.”
    Assume $50 per widget was the current wholesale market price. Was an enforceable contract in effect between the woman and the manufacturer?
    (A) No, because the woman’s order was too indefinite to constitute an offer, since it didn’t contain a price term.
    (B) No, because even if the woman’s order was a valid offer, it was effectively revoked prior to acceptance.
    (C) Yes, because the woman’s order created a valid option contract.
    (D) Yes, because the manufacturer’s fax of March 10 constituted an acceptance of the woman’s offer.
A
  1. (D) Here, the test maker is aware that many students will not choose Choice (D) because
    the woman’s March 7 purchase order called for a “prompt shipment” of goods. As a
    result of this language, students might construe the offer as limiting acceptance by
    performance only. However, according to UCC 2-206(b), “an order or other offer to
    buy goods for prompt or current shipment shall be construed as inviting acceptance
    either by a prompt promise to ship or by prompt or current shipment.” Therefore,
    the manufacturer’s fax promising to ship the widgets will be a valid acceptance.
    Choice (A) is wrong, because the UCC does not view price as an essential term. UCC 2-305 describes how the price is to be determined when the parties fail to designate a price. Choice (B) is incorrect, because the offer was accepted by the March 10 fax prior to the attempted revocation by return fax. Choice (C) is incorrect, because there were no assurances in the woman’s original offer that it would be held open.
How well did you know this?
1
Not at all
2
3
4
5
Perfectly
33
Q
  1. On September 1, a buyer contracted to buy 1000 widgets from a seller at $10 per widget, delivery to take place on or before September 15. On September 5, the buyer discovered that another widget seller was selling widgets for $8 per widget. The buyer then sent the following letter to the seller:
    “Please cancel our order for 1000 widgets. Your price is too high. We have found another supplier at a cheaper price.”
    On receipt of this letter, the seller would be legally justified in pursuing which of the following courses?
    (A) Shipping the widgets to the buyer.
    (B) Selling the widgets to another buyer by means of a public sale.
    (C) Selling the widgets to another buyer by means of either a public or private sale.
    (D) Selling the widgets to another buyer, but only if the seller is successful in whatever claims it has against the buyer.
A
  1. (C) UCC Section 2-703 provides that “where a buyer wrongfully rejects or revokes acceptance of goods on or before delivery…then with respect to the whole undelivered balance, the aggrieved seller may (a) withhold delivery of such goods; (b) stop delivery by any bailee; (c) resell and recover damages; (d) recover damages for non-acceptance; or (e) cancel.” With respect to resale, UCC Section 2-706 entitles the aggrieved seller to sell the goods by either public or private sale. As a result, Choice (C) is the preferred answer. Choice (A) is contrary to UCC Section 2-703(a) and therefore incorrect. Since goods may be resold at either a public or private sale, Choice (B) is not completely accurate. Choice (D) is wrong, because the seller is not required to sell the goods to another buyer only if he is successful in his claim against the buyer. Exam Tip: As a general rule, be leery of any answer choice that is prefaced by such words as “only if,” because it is very restrictive.
How well did you know this?
1
Not at all
2
3
4
5
Perfectly
34
Q
  1. A supplier of ink for printers sent the following letter to all of its customers:
    “Closeout special! We have decided to no longer stock green ink cartridges. We have on hand a limited supply of green ink cartridges for all printers; when they’re gone, they’re gone! Please submit your orders as soon as possible to make sure your order can be filled.”
    One of the regular customers of the supplier sent the following reply by fax:
    “Sorry to hear that you will no longer carry green ink cartridges, since that is one of our favorite colors. Please ship 100 green ink cartridges to our office as soon as possible.”
    The supplier faxed an acknowledgement of the order to the customer with a promise that the cartridges would be shipped out in one week. The next day, the supplier received the following e-mail from the customer:
    “Please cancel our order. We just discovered that we already have plenty of green ink cartridges in inventory.” The supplier proceeded to sell its entire stock of green ink cartridges at its asking price to other customers.
    In an action for breach of contract by the supplier against the customer, what is the maximum amount of damages that the supplier should be entitled to recover?
    (A) Nothing.
    (B) Only incidental damages, if any, that the supplier has incurred in preparing the green ink cartridges for shipment to the customer before receiving the customer’s e-mail.
    (C) $5,000, which was the asking price for the 100 green ink cartridges ordered.
    (D) Consequential damages, since the green ink cartridges were unique because they were the last of their kind to be offered for sale by the supplier.
A
  1. (B) In accordance with UCC Section 2-706, a seller may resell goods after the buyer has repudiated or breached the contract. Where the resale is made in good faith and in a commercially reasonable manner, the seller may recover the difference between the resale price and the contract price together with incidental damages.
    Since the supplier sold its entire stock of green ink cartridges, this would include the 100 that had been ordered by the customer. Therefore, the supplier can recover only incidental damages. Choice (A) is wrong, because it does not include possible recovery for incidental damages. Choice (C) is incorrect, because the supplierwould have no right to recover the full contract price. The supplier covered its losses by selling its entire inventory to other customers. Choice (D) is wrong for a couple of reasons: First, UCC 2-708 allows an aggrieved seller to sue for incidental damages, but not consequential damages. Second, the ink cartridges were not unique simply because this supplier was deciding to discontinue stocking them. There is no indication that green ink cartridges would not be available from other suppliers.
How well did you know this?
1
Not at all
2
3
4
5
Perfectly
35
Q
  1. On September 1, a man mailed a letter to a resort hotel on an island, reserving the “honeymoon suite” for the period from December 24 to January 1. The man explained that he and his fiancée were being married on December 23 and intended to travel by plane to the island on December 24. The resort hotel then sent back a confirmation notice stating that it was reserving the honeymoon suite for the man and his bride from December 24 to January 1 “for the weekly rental of $10,000.”
    On December 23, a blizzard struck, blanketing the city with five feet of snow. As a result, the airports were shut down. The man immediately telephoned the manager of the resort hotel and explained his predicament. When the manager requested assurance that the man and his bride would still be making the trip, the man responded, “The airports are scheduled to re-open tomorrow … if they do we hope to make the trip.” The next morning, the manager sent an e-mail to the man, advising him that the honeymoon suite had been rented to another couple for the period from December 24 to January 1.
    If the man brings suit against the resort hotel for breach of contract, which of the following will provide the hotel with its best defense’?’
    (A) The resort hotel’s duty to hold the honeymoon suite for the man and his bride’s arrival was excused by the apparent impossibility on December 23 of their timely performance.
    (B) The resort hotel’s duty to hold the honeymoon suite for the man and his bride’s arrival was discharged by their failure to give adequate assurances of their own agreed performance.
    (C) The resort hotel’s duty to hold the honeymoon suite for the man and his bride’s arrival was excused by frustration of purpose.
    (D) The man and his bride’s apparent inability on December 23 to make the trip constituted a material breach that excused the resort hotel of any obligation to hold the honeymoon suite for their arrival.
A
  1. (B) Choice (A) is wrong, because the facts do not indicate that it was impossibLe for the man and his bride to make the trip. Since the airports were scheduled to re-open on December 24, the man and his bride clearly could have made the trip and checked into the hotel on their arrival date. Choice (C) is incorrect, because frustration of purpose excuses performance in situations where the purpose of the contract is destroyed bya supervening event. Since the coronation cases, the American courts have excused the promisor’s duty where the purpose of the contract is wholly frustrated by a fortuitous, unforeseeable event on the theory of failure of consideration. Since it was still possible for the man and his bride to make the trip, their performance in renting the hotel room was not totally frustrated. Choice (D) is incorrect, because the man and his bride’s apparent inability to make the trip on December 23 did not constitute a material breach of contract or an anticipatory breach. In order that an action will lie for anticipatory breach, the repudiation (where it is an express refusal to perform) must be positive and unequivocal. A mere expression of doubt or fear by the promisor that he will or may be unable to perform, when in the future his performance is due, is not the positive repudiation that is necessary before an anticipatory breach will lie. Thus, by process of eLimination, Choice (B) is correct. The manager of the hotel asked for assurance of performance, but the man merely said that if the airports re-opened, he and his bride “hoped to make the trip.” This is not an adequate assurance that they will arrive at the specified time.
How well did you know this?
1
Not at all
2
3
4
5
Perfectly
36
Q
  1. For the last 20 years, a husband and wife had taken a vacation during the first week in July. They would fly from their home to the city and spend the week at a hotel in the most luxurious suite. On March 1, the husband mailed a letter to the hotel, reserving the suite for the first week in July. The hotel then sent back a confirmation notice stating that it was reserving the suite for the husband and wife for the first week in July “for the weekly rental of $25,000.”
    When the husband and wife arrived at the hotel on July 1, the hotel, without legal excuse, informed the man and his wife that the hotel had rented the suite to another couple for the first week in July. Quite apologetic, the manager of the hotel offered the husband and wife the hotel’s next best accommodation at a weekly rental of $20,000. The manager informed the husband and wife that the other suite was beautifully furnished, “but not quite as luxurious as the suite they reserved.” Visibly upset, the husband and wife rejected the manager’s offer and relocated to another hotel, where they rented the other hotel’s most luxurious suite for the first week in July at a cost of $25,000.
    If the husband and wife now sue the first hotel for breach of contract, they will most likely
    (A) prevail, because the hotel knew that for the past 20 years the husband and wife always stayed in the most luxurious suite.
    (B) prevail, because the substitute accommodations offered by the hotel were not comparable to the suite they reserved.
    (C) not prevail, because the hotel did offer substitute accommodations at a $5,000 savings.
    (I?) not prevail, because the husband and wife sustained no legal damages in renting a comparable suite at another hotel for the same rental.
A
  1. (B) First, it is important to point out that the hotel was in breach of contract by not renting the agreed upon suite to the husband and wife. As a consequence, Choices (C) and (D) are incorrect. Note that Choice (C) is wrong because the husband and wife’s “expectation interest” was staying in the suite which was the hotel’s most luxurious accommodations. As a result, the husband and wife were not obligated to accept the manager’s offer, since the other suite was not comparable to the value of the promised performance. By the same token, Choice (D) is wrong, because even though the husband and wife may not have suffered any pecuniary injury, they nonetheless would be entitled to any consequential damages (like the cost of moving to the other hotel) or at least be entitled to nominal damages. Choice (A) is incorrect, because the hotel’s knowledge of past performance has no direct bearing on the cause of action. The hotel was in breach when they failed to rent the most luxurious suite to the husband and wife, and this would be true regardless of whether the husband and wife had ever stayed in the hoteL before. Therefore, Choice (B) is the best answer.
How well did you know this?
1
Not at all
2
3
4
5
Perfectly
37
Q
  1. A collector owned a rare 16th-century tapestry. The collector contracted in writing to sell the tapestry to a tapestry dealer for $100,000. The contract stipulated that delivery would be “F.O.B. at the dealer’s shop,” with payment to be made one week after tender.
    When the dealer received the tapestry, he noticed that it had been damaged in transit. The dealer immediately contacted the collector and notified him that he was rejecting the tapestry because it had ripped apart and was becoming unwoven. The collector told the dealer that he would get back to him with re-shipping instructions. The collector did not make any further contact with the dealer. After four weeks, the dealer then sold the tapestry to a buyer for $120,000.
    If the collector sues the dealer for damages, the collector should recover
    (A) $120,000, for conversion.
    (B) $108,000, because the dealer is entitled to a reasonable sum not exceeding 10% on the resale.
    (C) $100,000, which is the contract price.
    (D) $20,000, which covers the difference between the contract price and the sale price.
A
  1. (B) This fact pattern is governed by UCC Section 2-603, which deaLs with a merchant
    buyer’s duties as to rightfully rejected goods. After there has been a rightful rejec
    tion of goods, the buyer is under a duty to (a) follow reasonable instructions received
    from the seller with respect to the goods and, in the absence of such instructions,
    (b) make reasonable efforts to sell them for the seller’s account. When the buyer
    sells the goods, he is entitled to reimbursement out of the proceeds for reasonable
    expenses of caring for and selling them. This amount, however, should not exceed
    10 percent on the gross proceeds. Based upon the facts, the dealer was entitled
    to resell the tapestry, since he did not receive any instructions from the collector.
    Consequently, choice (B) is correct, because the dealer is entitled to keep 10 per
    cent of the resale price ($12,000) for reasonable expenses. Choices (A) and (C) are
    therefore wrong. Choice (D) is incorrect, because it speaks to the buyer’s remedy if
    the buyer has to cover by purchasing replacement goods from another source. That
    is not the situation here.
How well did you know this?
1
Not at all
2
3
4
5
Perfectly
38
Q
  1. A collector regularly bought and sold coins. One day, she saw an advertisement in a coin collectors’ magazine advertising for sale a rare coin from 1898 for $10,000. She immediately contacted the seller and asked about the quality of the coin. The seller assured her that the coin was in mint condition. The collector then agreed to purchase the coin for $10,000. The contract stipulated that delivery would be “F.O.B. at the collector’s establishment,” with payment to be made one week after delivery. The seller stated that the coin would be shipped out at the end of the week.
    When the collector received the coin, she noticed that the coin had a large scratch across the face. Nonetheless, the collector accepted delivery. Two weeks later, the collector sold the coin to another collector for $12,000. The collector has refused to pay anything to the seller. The seller brought a breach of contract action against the collector, who, in turn, has filed a counterclaim against the seller.
    Who is most likely to prevail, and in what amount?
    (A) The collector is entitled to nominal damages, because the coin was received in a damaged condition.
    (B) Neither party should prevail, because the risk of loss was on the seller, but the collector did not incur any loss, since she sold the coin for a profit.
    (C) The seller is entitled to $10,000, because the collector accepted delivery of the coin.
    (D) The seller is entitled to $12,000, because the collector’s resale constituted a conversion.
A
  1. (C) According to UCC 2-607, the buyer must pay at the contract rate for any goods accepted. Acceptance of goods by the buyer precludes rejection of those goods that were accepted and if made with knowledge of a non-conformity, cannot be revoked because of it unless the acceptance was on the reasonable assumption that the non-conformity would be seasonably cured. Here, the facts state that the collector accepted delivery of the coin. Since there was a valid acceptance, she must pay the seller the contract price. Choice (A) is wrong, because the collector accepted the coin knowing that it was damaged, thereby precluding the collector from recovering any damages forthe scratch on the surface of the coin. Choice (B) is wrong, because the seller is entitled to payment, since the collector accepted the coin in its damaged condition. Choice (D) is incorrect, because the seller would have no right to
    the profit the collector made on the resale of the coin.
How well did you know this?
1
Not at all
2
3
4
5
Perfectly
39
Q
  1. A man mailed a letter to a woman promising to sell her his motorcycle for $1,000. After receiving the letter, the woman sent the man a fax that stated, “The price is a little high, I’ll give you $800.” The man responded by fax, “I will not accept $800.” The next day, the woman telephoned the man and said, “I changed my mind, I will pay $1,000 for the motorcycle.” The man refused to sell the woman his motorcycle.
    If the woman sues the man for breach of contract, which of the following defenses would be the man’s best defense?
    (A) Since the woman’s purported acceptance was oral, it constituted a different mode of communication from the written offer.
    (B) The contract was unenforceable under the statute of frauds, because the woman’s purported acceptance was oral.
    (C) The woman’s counter offer terminated her power of acceptance.
    (D) The man’s rejection of the woman’s counter offer terminated the woman’s power of acceptance.
A
  1. (C) This is a very technical Contracts formation question dealing with the issue of when a counter-offer operates as a rejection. An offeree’s power of acceptance is term inated by the making of a counter-offer. Here, when the woman stated that $1,000 was too high but that she would pay $800, she made a counter-offer that rejected the original offer. Therefore, her subsequent attempt to accept that initial offer was not timely. Choice (A) is wrong, because there is no reason why an oral acceptance, if timely, would not be good. An acceptance does not have to be in the same form as the offer, unless specifically stated otherwise. Choice (B) is incorrect, because the dealings between the parties never got that far. The Statute of Frauds does not become an issue until after mutual assent is found to exist. Here, the problem is that mutual assent is lacking, so there is no need to determine whether the contract must be in writing. Choice (D) is incorrect, because it’s not the man’s rejection of the woman’s counter-offer that terminates her power of acceptance; her power to accept the $1,000 price ended much earlier in time when she made the counteroffer, as discussed above.
How well did you know this?
1
Not at all
2
3
4
5
Perfectly
40
Q
  1. A man offered to sell his barbecue to his neighbor for $100. After receiving the man’s offer, the neighbor responded, “Let me think it over.” The man then said, “If you say so.” The next day, the man sold the barbecue to his brother for $100. Thereafter, the neighbor decided to accept the man’s offer, but learned from a reliable source that the barbecue had been sold to the brother.
    If the neighbor sues the man for breach of contract, judgment for
    (A) the man, because the offer to the neighbor terminated when the neighbor learned of the sale to the brother.
    (B) the man, because there was no consideration to keep the offer open for an extended period of time.
    (C) the neighbor, because the offer became irrevocable for a reasonable time when the man allowed the neighbor to “think it over.”
    (D) the neighbor, because, he is a merchant.
A
  1. (A) An offeree’s power of acceptance is terminated when the offeror takes definite action inconsistent with an intention to enter into the proposed contract and the offeree acquires reliable information to that effect. Thus, the man’s offer was effectively revoked when the neighbor learned from a reliable source that the barbecue had been sold to the man’s brother. Choice (B) is not the best answer. The fact that consideration was not paid to keep the offer open speaks to why the offer was not irrevocable, but does not state why the offer was terminated. Choice (A) is therefore a better choice, because it states why the offer was terminated. Choice (C) is incorrect, because simply allowing an offeree to think over an offer for a period of
    time does not make the offer irrevocable. Absent consideration, the offeror is not obligated to hold the offer open for that period. Choice (D) is incorrect, because the neighbor’s status as a merchant is irrelevant. If the man had been a merchant and his offer was in writing, that may make the offer irrevocable as a merchant’s firm offer.
How well did you know this?
1
Not at all
2
3
4
5
Perfectly
41
Q
  1. A landlord rented an apartment to a tenant for $250 per month in accordance with a month-to-month agreement. On September 3, the landlord mailed the following letter to the tenant:
    “September 2 In consideration of one dollar, receipt of which is acknowledged, I hereby give you an option for 20 days from the above date to sign a two-year lease at $225 per month, provided you pay two months’ rent in advance.”
    The tenant received the letter on September 4, but did not read it until September 5. On September 23, the tenant telephoned the landlord and said, “I want to give you the $450 and sign the two-year lease as soon as possible.” The landlord replied, “I’ve change my mind. I do not want to sign a lease with you. Moreover, I want you to vacate the apartment immediately.” At no time after receiving the landlord’s letter on September 4 did the tenant pay him the one-dollar consideration.
    Did the landlord’s letter constitute an effective offer for a two-year lease?
    (A) Yes, because it manifested a willingness to enter into a specific bargain that could be concluded by the tenant’s assent.
    (B) Yes, because consideration for the option can be infeffed from the previous month-to-month lease.
    (C) No, unless the tenant paid or tendered to the landlord the one-dollar consideration.
    (D) No, because it contained a condition precedent to execution of the proposed lease.
A
  1. (A) According to the Restatement of Contracts 2d, Section 24, “An offer is the manifestation of willingness to enter into a bargain, so made as to justify another person in understanding that his assent to that bargain is invited and will conclude it.” Clearly, Choice (A) is correct, because the landlord’s Letter manifested such a present contractual intent. It is crucial to note that the interrogatory asked only if a valid offer was made by the September 2 letter. Choice (B) is wrong, because consideration is not needed to create a valid offer. Choice (C) is incorrect, because the
    offer acknowledged the receipt of the $1 consideration and courts generaLly do not
    inquire into the adequacy of consideration. In addition, the significance of whether consideration was paid would determine whether the offer was irrevocable, not whether an offer was initially made. Choice (D) is incorrect, because there was no condition precedent to the offer taking effect.
How well did you know this?
1
Not at all
2
3
4
5
Perfectly
42
Q
  1. A buyer sent an e-mail to a retailer of camping supplies inquiring about the possibility of purchasing a tent that would sleep four adults. The buyer explained that he was planning for an upcoming camping trip at the end of the month. The retailer responded by e-mail:
    “I can sell you a tent that would sleep four, for $500. This price is good for one week.”
    Three days later, the retailer learned from another seller of camping supplies that the same buyer had come to her store the day before and had inquired about purchasing a tent that would sleep four adults. The other seller told the retailer that she had quoted the buyer a price of $600 for a tent that would sleep four adults. The next day, the buyer telephoned the retailer and stated that he wanted to buy the tent for $500. The retailer replied: “I do not want to sell to you. I tried to deal with you in good faith, but I hear you’ve been comparing my prices with other sellers. Why don’t you deal with them!” The retailer then slammed down the phone.
    Assume that the retailer’s e-mail created in the buyer a valid power of acceptance. Was that power terminated when the retailer learned from the other seller of the buyer’s conversation with the other seller?
    (A) Yes, because the other seller gave factually accurate information to the retailer.
    (B) Yes, because it gave the retailer reasonable grounds to believe that the buyer had rejected his offer.
    (C) No, because the indirect communication to the retailer was oral.
    (D) No, because the buyer’s conversation with the other seller did not constitute a rejection.
A
  1. (D) Rejection of an offer by the offeree terminates the offer. As a general ruLe, a rejection is the offeree’s refusal to accept the offer as made which is communicated to the offeror. Whether expressed in words or implied from the offeree’s conduct, if it is apparent to the offeror that the offeree declines to accept, the offer is rejected. The mere fact that the buyer talked to the other seller about the possible purchase of a tent is not a clear indication that he intended to reject the retailer’s offer. Choices (A) and (B) are wrong, because the information the retailer learned from the other seller, even if factual, does not imply that the buyer does not want to purchase a tent from the retailer. Choice (C) is wrong, because it would not matterwhetherthe communication was written or oral; it is not sufficient to constitute a rejection of the offer.
How well did you know this?
1
Not at all
2
3
4
5
Perfectly
43
Q
  1. On May 2, a woman mailed the following letter to a man:
    “May 1
    I have two tickets to the concert on July 1 at the auditorium in town. I’ll sell them to you for $60 per ticket, which is $10 above face value. Since the concert has been sold out for months, I think that is a good deal. You have 15 days from the above date to decide whether to accept this offer.”
    The man received the letter on May 4, but did not read it until May 6.
    On May 18, the man went to the woman’s home and attempted to accept the offer. The woman replied:
    “Too late! I sold the tickets last week for $75 each.”
    Assume that the woman’s letter created in the man a valid power of acceptance. Was that power terminated by lapse of time before the man went to the woman’s home on May 17?
    (A) Yes, because the letter was mailed on May 2.
    (B) Yes, because the letter was dated May 1.
    (C) No, because the man received the letter on May 4.
    (D) No, because the man did not read the letter until May 6.
A
  1. (B) According to the woman’s letter, the offer was open for a period of 15 days. The question is, from what date do the 15 days run? Since the offer specifically stated that the 15 days run from the date listed on the letter, which was May 1, the man had until May16 to accept the offer. Since the 15 days run from the date on the letter, not the date on which it was sent, Choice (B) is better than Choice (A). Choice (A) focuses on the date the letter was mailed. But since the woman, as offeror, specifically stated that the offer would expire 15 days from the date written on the letter, the last day to accept would have been May 16. Choices (C) and (0) are wrong, because, as explained above, the date of receipt or the date the letter was read do not affect the duration of the offer. The offeror is master of the offer, and the woman expressly stated that the 15 days were to run from the date of the letter.
How well did you know this?
1
Not at all
2
3
4
5
Perfectly
44
Q
  1. Two law students were having lunch one afternoon when the first student said to the second, “I’m thinking of selling my car.” The second student replied, “I might be interested in buying your car if the price is reasonable and I can convince my parents to float me a loan.” The first student then grabbed a piece of paper and wrote the following on it:
    “I will sell you my car for $1,000. In exchange for $1 received this day, I agree that you can have one month from today’s date to decide whether to accept this offer.”
    The first student then signed the paper and handed it to the second student. The second student handed a one dollar bill to the first student and left.
    One week later, the second student succeeded in obtaining a $1,000 loan from his parents. He then telephoned the first student to accept the offer. Upon hearing the second student say hello, and before the second student could accept the offer, the first student said, “I’ve changed my mind. I do not want to sell my car.”
    Can the second student still accept the first student’s offer?
    (A) No, because $1 is inadequate consideration.
    (B) No, because the first student’s statement was unequivocal notice that he no longer wished to contract with the second student.
    (C) Yes, because the first student’s letter formed a valid option.
    (D) Yes, based on the principal of promissory estoppel.
A
  1. (C) The offer is the proposal to sell the car for $1,000; there is a grant of a one-month option on the deal; and the payment of $1 to the offeror is a valid and adequate way to secure enforcement of the subsidiary promise to keep the offer open. Therefore, the facts present a legitimate option contract, whereby the second student may still accept the first student’s offer, regardless of the first student’s attempt to revoke. Therefore, answer choice (C) is correct, and choices (A) and (B) are incorrect. Choice (D) is incorrect, because under promissory estoppel, courts will sometimes enforce a subsidiary promise to keep an offer open where the offeree has foreseeably and reasonably relied on the option, and injustice can only be avoided by enforcing the promise. However, these facts not only fail to demonstrate a reliance and injustice, but moreover do not necessitate the doctrine of promissory estoppel because the second student has the full legal right to accept under the valid option.
How well did you know this?
1
Not at all
2
3
4
5
Perfectly
45
Q
  1. A construction company, in preparing its bid for the construction of a new hospital, received a quotation of $120,000 from a subcontractor who offered to do the kitchen work in the new hospital. This bid was $30,000 lower than the construction company’s next lowest bid for the kitchen work. As a result, the construction company lowered its bid by $20,000 before submitting it to the hospital board. After the construction company was awarded the construction bid and had accepted the subcontractor’s offer, the subcontractor discovered that in his preparation of the quotation, he had overlooked some subsidiary kitchen installments required by the plans.
    Immediately thereafter, the subcontractor brought suit for rescission of the contract. It should
    (A) prevail, because of the unilateral mistake.
    (B) prevail, because the mistake was an essential element of the bargain.
    (C) not prevail, unless the construction company knew or should have known of the subcontractor ‘s error.
    (D) not prevail, because the computation mistake was antecedent to acceptance of the bid.
A
  1. (C) In all unilateral mistake situations, if the offeree knows or has reason to know of the offeror’s mistake when he accepts, then the offeror is not bound. In otherwords, if the non-mistaken party is (or should have been) aware of the mistake, he cannot “snap up” the offer. So, if the construction company neither knew nor should have known of the subcontractor’s error, then the contract was enforceable. Choices (A) and (B) are wrong, because the unilateral mistake will not provide the basis for relief unless it is shown that the non-mistaking knew or should have known of the mistake. This is true even if the mistake went to an essential element of the bargain. Choice (D) is incorrect, because the timing of the mistake is not a determining factor.
How well did you know this?
1
Not at all
2
3
4
5
Perfectly
46
Q
  1. A homeowner wished to have his house painted. He contacted a number of house painters in his area and asked them to submit bids to do the work. The homeowner received 10 bids. The first nine offered to paint the house for amounts ranging from $10,000 to $12,500. The 10th bid was sent by telegram and was supposed to be for $10,000. However, the telegraph company made a mistake and transmitted the bid as $1,000. The homeowner immediately accepted the 1 0th bid, but the 1 0 painter refused to perform.
    The I 0th painter’s best defense in an action for breach of contract by the homeowner would be
    (A) that the homeowner should have been aware of the mistaken transmission, because of the disparity between its bid and the others.
    (B) that the telegraph company should be liable as an independent contractor.
    (C) that the homeowner was under an affirmative duty to investigate all submitted bids.
    (D) that the mistake made the contract unconscionable.
A
  1. (A) If a mistake is made in an offer and the offeree is or should be aware of the mistake, there will be no contract. In the present case, since there was a substantial difference between the tenth painter’s bid and the other bids, the homeowner should have known of the mistake in transmission, thereby precluding him from snapping up the offer. Choice (B) is incorrect, because the telegraph company may have liability for their mistake, but that does not speak to a valid contract defense for the tenth painter. Choice (C) is wrong, because there is no such requirement that an offeree investigate the validity of submitted offers. Unless the offeree has reason to know the offer may be mistake, the offeree is entitled to treat the offer as genuine. Choice (D) is wrong, because the tenth painter chose this form of conveyance of its offer, so the tenth painter cannot now claim the contract is unconscionable because of a mistake by the conveyor. The tenth painter would have to argue that the homeowner should have known of the mistake, to have a valid defense.
How well did you know this?
1
Not at all
2
3
4
5
Perfectly
47
Q
  1. A woman awoke one morning to discover that someone had vandalized her home during the night. The woman then published the following notice in the local newspaper:
    “REWARD
    Any person who supplies information leading to the arrest and conviction of the person who vandalized my home, located at 1223 1st Street, will be paid $5,000.”
    The reward notice in the local newspaper proposed a
    (A) unilateral contract only.
    (B) bilateral contract only.
    (C) unilateral contract or bilateral contract, at the offeree’s option.
    (D) unilateral contract that ripened into a bilateral contract when someone supplied the information leading to the vandal’s conviction.
A
  1. (A) A unilateral contract is a contract in which a promise is given in exchange for an actual performance by the other party. The genuine unilateral contract situation will be rare, as it will operate only where the offer expressly and unambiguousLy requires a performance acceptance, or as in the present case, where the offer is an offer to the public (e.g., a reward offer). Choice (B) is therefore wrong. Choice (C) is wrong, because it is the language used by the offeror in creating the offer that determines whether it is for a unilateral or a bilateral contract. Choice (D) is wrong, because a unilateral contract is formed when the requested act is performed.
How well did you know this?
1
Not at all
2
3
4
5
Perfectly
48
Q
  1. After the murder of his brother, a man published the following notice in the local newspaper:
    “REWARD
    Any person who supplies information leading to the arrest and conviction of the murderer of my brother will be paid $10,000.”
    An amateur detective, without knowledge of the reward notice, began investigating the matter as a result of his own curiosity kindled by the sensationalism surrounding the brother’s murder. One week later, the detective secured information that led to the arrest and later conviction of the murderer. During the murder trial, the detective found out about the reward and demanded the $10,000 from the man.
    In an action by the detective to recover the $10,000 reward, he will
    (A) succeed, because his apprehension of the murderer created a contract implied in law.
    (B) succeed, because he was unaware of the offer. (C) not succeed, because he did not have knowledge of the reward.
    (D) not succeed, because his investigation was not a bargained-for exchange.
A
  1. (C) As a general rule, a contract can only be formed if the offeree knew of the existence of the offer at the time of the alleged acceptance. So, since the detective was unaware of the reward offer when he performed the requested act, he cannot claim a valid contract had been formed. Choices (A) and (B) are therefore wrong. Choice (D), though arguably correct, is less preferred because this question deals with whether an offeree must know of the offer at the time of his alleged acceptance. The key issue is not whether there is valid consideration (i.e., bargained-for exchange) to support the contract. As such, Choice (C) is preferred, because a reward offer (or other unilateral contract) must be accepted by an offeree who knows of the offer.
How well did you know this?
1
Not at all
2
3
4
5
Perfectly
49
Q
  1. A recent law school graduate was hired by a bar review course, under an oral agreement, as an editorial consultant. Her job responsibilities included preparing new course outlines, proofreading and grading student homework assignments. The bar review course agreed to pay the graduate a starting salary of $2,500 a month.
    Three months later, the graduate was approached by the regional director of the bar review course, who handed her a newly published 60-page booklet entitled “Employment Manual.” He instructed the graduate to read the manual and indicated that it contained important information concerning company policy considerations and employee benefits.
    When the graduate returned home that evening, she started to read the manual. After reading about 30 pages, the graduate became tired and went to sleep. She never got around to reading the rest of the manual.
    Six months later, the graduate received a termination notice from the bar review course. The notice indicated that the graduate was being fired for insubordination because she had complained about the poor quality of the bar review course’s materials and refused to work overtime grading papers. Following her dismissal, the graduate brought suit against the bar review course for breach of contract.
    Which of the following, if true and provable, would furnish the bar review course with its best defense?
    (A) All other bar review course employees worked overtime whenever requested to do so.
    (B) When the graduate accepted employment with the bar review course, the company never made any promises regarding job security or duration of employment.
    (C) The materials in the bar review course had recently been reviewed by the American Bar Association’s Committee on Legal Education and had received “high acclaim.”
    (D) The second page of the bar review course manual contained a paragraph stating that all policies, guidelines and employee benefits are “purely gratuitous and not intended to create any ongoing contractual obligation.”
A
  1. (D) The bar review course will want to show that the graduate’s employment was “atwill” and could be terminated at any time. The only thing that could potentially change the graduate’s employment from an at-wiLl employment arrangement would be the contents of the employment manual. However, if the manual specifically stated that it did not create “any ongoing contractual obligation,” the graduate’s employment will remain an “at-wilL” employment that can be terminated by the bar review course. Note that choice (B) is less preferred, because it does not address whether the employment manual may create contractual obLigations that could change the graduate’s employment from an at-will employment. Choices (A) and (C) are irrelevant considerations with respect to whether the graduate’s employment was gratuitous or bargained-for.
How well did you know this?
1
Not at all
2
3
4
5
Perfectly
50
Q
  1. After a lengthy interview with a company vice president, an employee was hired by the company to work in the company’s accounting department. The parties agreed that the employment would be on an at-will basis. At the end of her first week of work, the employee was given a booklet entitled “Employment Manual,” with instructions to read the book in its entirety by the end of the following week. That evening, the employee began reading the manual. The first few pages described the history of the company and provided a personal biography of its president. On page 20, the manual stated that the company treats its employees “as family” and that employees will be discharged “only with good cause.” The employee finished reading the manual as requested. The employee interpreted the statement on page 20 as insuring continued employment unless good cause existed for termination. Over the next two months, the employee continually complained to her supervisor that the lighting in the accounting department was insufficient. Finally the supervisor, fed up with the complaints, fired the employee. The employee then sued the company, seeking to recover on grounds of promissory estoppel.
    Which of the following facts, if true and provable, would be most helpful for the employee’s cause of action?
    (A) At the time when the company hired the employee, the company subjectively intended that the employee be given job security.
    (B) The employee interpreted the clause in the manual stating that company employees would be treated “as family” to mean that she would have job security and could only be fired for good cause.
    (C) Just prior to receiving the manual, the employee seriously considered quitting, but continued to work for the company in reliance on the provisions contained on page 20 of the manual.
    (D) The employee’s complaints regarding the insufficient lighting were factually true and justifIable.
A
  1. (C) Detrimental action or forbearance by the promisee in reliance on a gratuitous promise, within limits, constitutes a substitute for consideration, or a sufficient reason for enforcement of the promise without consideration, under the doctrine of promissory estoppel. To avoid unjust enrichment, the promisor who induces substantial change of position by the promisee in reliance on the promise is estopped to deny its enforceability as lacking consideration. Since the employee is seekingto recover on the basis of promissory estoppel as a substitute for consideration, her best argument must show detrimental reliance. Choice (C), the correct answer, states that instead of quitting her job, she continued to work, “in reliance on the provisions contained on page 20 in the manual.” Choice (B), while factually similar, is not as strong, since it deals with her interpretation of the manual rather than herreliance on it. Choice (A) is wrong, because the subjective intent of a party is not controlling when interpreting the terms of the contract. In addition, it does not address the crucial concern of promissory estoppel. Choice (D) also fails to address the key issue of reliance.
How well did you know this?
1
Not at all
2
3
4
5
Perfectly
51
Q
  1. A man placed the following advertisement in his local newspaper:
    “Public Auction Sale … without reserve December 7, 10:00 a.m.
    110 Walnut St., City, State
    Entire furnishings of home must go: antiques, piano, pool table, appliances, tables, stereo system, etc.”
    On the morning of December 7, a group of approximately 20 people showed up at the man’s home. The first item the man put up for bid was an antique Baldwin grand piano. The man announced that the bidding would be “without reserve” and said, “What’s the opening bid on this beautiful Baldwin grand piano?” A woman opened the bidding with $100. The man then commented, “This piano is worth at least $5,000. What other bids am I offered?” When the man did not receive any other bids, he informed the woman that he would not accept $100 and was removing the piano from the sale.
    If the woman asserts an action against the man for breach of contract, the woman will most likely
    (A) prevail, because goods put up at an auction “without reserve” may not be withdrawn.
    (B) prevail, because whether or not the auction is “without reserve,” goods cannot be withdrawn after the auctioneer calls for bids.
    (C) not prevail, because at an auction “without reserve,” the auctioneer may withdraw goods until he announces completion of the sale.
    (D) not prevail, because at an auction “without reserve,” the auctioneer invites offers, which he may accept or reject.
A
  1. (A) According to UCC 2-328, an auction is deemed to be with reserve unless specifically stated to be without reserve. Here, the man stated in his advertisement and again when he put the piano up for auction that the auction would be without reserve. When goods are put up witho ut reserve, the auctioneer makes an offerto sell at any price bid by the highest bidder, and after the auctioneer calls for bids, the goods cannot be withheld unless no bids are made within a reasonable time. Since the woman made a bid and the auction was without reserve, the woman would be entitled to buy the piano for $100. Choice (B) is wrong, because if the auction was with reserve, the goods can be withdrawn from sale at any time prior to the announced completion of the sale. Choices (C) and (D) are wrong, because they misstate the rule described above, covering an auction without reserve.
How well did you know this?
1
Not at all
2
3
4
5
Perfectly
52
Q
  1. A mushroom farmer agreed in a signed writing on January 2 to deliver to a chef at a local restaurant on March 1 a specified quantity of mushrooms at a specified price. The mushrooms on the farmer’s farm were usually picked and packaged by the farmer’s three sons. On February 27, two of the Sons were injured in a fanning accident and hospitalized. As a result, the farmer encountered a manpower shortage and could not process all of the mushrooms for the chef’s order.
    On February 28, the farmer telephoned the chef and said, “Because my two sons were injured yesterday, I won’t be able to deliver your mushrooms on March 1. However, I am trying to hire some other farm hands to help process your order. Although I can’t promise it, I should be able to deliver the mushrooms by the end of the week.” The chef, who knew that the fanner’s sons were responsible for the mushroom farming, said, “No problem. I think I’ll be able to get by without them for a few days. However, be advised that I will hold you liable for any loss I sustain as a result of your failure to deliver the mushrooms on March 1 .“ When the chef failed to receive the mushrooms on March 5, he sent the following fax to the fanner:
    “I must have the mushrooms no later than March 9.” This fax was received and read by the farmer on the same day.
    If the fanner delivers the mushrooms to the chef on March 9 and the chef accepts them, can the chef successfully maintain a contract action against the farmer to recover damages resulting from the delay in delivery?
    (A) No, because temporary impracticability excused his duty to deliver the mushrooms on March 1.
    (B) No, because the buyer’s statements and acceptance of the mushrooms constituted a waiver of the condition of timely delivery.
    (C) Yes, because his statements to the farmer did not constitute a promise to forgo any cause of action he then had or might later acquire.
    (D) Yes, because there was no consideration to support his waiver, if any, of timely delivery.
A
  1. (C) This question deals with a single delivery contract for the sale of perishable goods, i.e., mushrooms. UCC rules thus apply, since this is a sale of goods. The farmer’s performance was due on March 1. Since he failed to deliver the goods at that time, he committed a breach of contract. Under UCC 2-715, an aggrieved buyer such as the chef can recover incidental damages resulting from the seller’s breach: these damages include “expenses reasonably incurred in inspection, receipt, transportation and care and custody of goods rightfully rejected, and commercially reasonable charges, expenses or commissions in connection with effecting cover and anyother reasonable expense incident to the delay or other breach.” Choice (C) is therefore correct. Where the chef replied to the farmer’s phone call by saying, “…be advised that I will hold you liable for any loss I sustain as a result of your failure to deliver the mushrooms on March 1,” the chef only waived the delivery date, not the farmer’s performance. Choice (B) is therefore incorrect. Choice (A) is wrong, because performance was not objectively impaired; by hiring other workers, the contract could have been performed. In addition, there is no showing that the injuries to the two sons caused severe hardship of performance to the farmer. Choice (D) is incorrect, because under the UCC a waiver does not require new consideration.
How well did you know this?
1
Not at all
2
3
4
5
Perfectly
53
Q
  1. Needing a new supply of basketballs for the upcoming season, a basketball coach placed an order with a sporting goods store to purchase 25 regulation basketballs. Since the season was scheduled to begin on November 1, the coach stated that he needed the basketballs delivered no later than October 15. The sporting goods store said it would deliver the basketballs by that date. On October 10, one of the sporting goods store’s employees was moving items in the warehouse and accidentally destroyed a container of basketballs. As a result, the owner of the sporting goods store telephoned the coach on October 11 and said, “We had an accident in our warehouse yesterday, and as a result we do not have enough basketballs to fill all of our orders. Therefore, I won’t be able to deliver your basketballs on October 15. However, I have contacted our supplier to see if they can rush a delivery of basketballs to us. Although I can’t promise it, I should be able to deliver the basketballs you ordered by October 20.” The coach replied, “No problem. I think I’ll be able to get by until then.” When the coach failed to receive the basketballs on October 20, he sent the following e-mail to the sporting goods store:
    “I must have the basketballs no later than October 23.” The e-mail was received and read the same day by the owner of the sporting goods store.
    If the sporting goods store fails to deliver the basketballs to the coach on October 23, will the coach be entitled to cancel the contract?
    (A) Yes, provided both parties are viewed as being merchants.
    (B) Yes, provided that the three-day notice afforded the sporting goods store a reasonable time in which to perform.
    (C) No, because the coach’s October 11 statement effectuated a waiver of any condition of timely delivery.
    (D) No, because the sporting goods store, by its October 11 statement, did not promise to deliver the basketballs by October 20.
A
  1. (B) This question deals with the buyer’s right to cancel forfailure of timely performance. Where the time set for performance has passed, the party awaiting performance may agree to a new commercialLy reasonable time for performance prior to cancellation. The coach could have cancelled the contract for non-delivery on October 15 in accordance with UCC 2-711, which states that “(1) Where the seller fails to make delivery … the buyer may cancel and … (a) cover … or (b) recover damages for non-delivery …“ The coach nevertheless chose to await performance, although no specific date was set during the October11 telephone conversation. Under UCC 2-309, “(1) The time for shipment or delivery or any other action under a contract if not provided in this Article or agreed upon shall be a reasonable time.” Comment 5 provides: “The obligation of good faith … requires reasonable notification before a contract may be treated as breached because a reasonable time for delivery or demand has expired. This operates both in the case of a contract originally indefinite as to time and of one subsequently made indefinite by waiver.” Therefore, Choice (B) is correct, since the coach will be entitled to cancel provided the e-mail received by the sporting goods store (i.e., 3-day notice) was a reasonable time. Choice (A) is wrong, because the rule described above is not limited to contracts between merchants. Choice (C) is incorrect, because where time of delivery is waived or not provided, a reasonable time is implied in view of good faith commercial dealing. A waiver does not negate any condition of timely delivery. Choice (D) is wrong, because the coach is entitled to cancel the contract on reasonable notice, since the sporting goods store failed to meet the original October15 delivery date.
How well did you know this?
1
Not at all
2
3
4
5
Perfectly
54
Q
  1. An owner and operator of a restaurant contracted in writing with a produce distributor to buy 50 pounds of heirloom tomatoes. At the time the contract was signed, the owner orally said to the distributor, “We do have an understanding that our chef must approve the quality of the heirloom tomatoes before I will pay you.” The distributor acknowledged the owner’s request and responded, “If you say so.”
    Thereafter, the distributor delivered the heirloom tomatoes to the owner. After inspecting the tomatoes, the chef refused to give his approval, because he felt that the tomatoes were not of sufficient quality. As a result, the owner refused to accept and pay for the tomatoes.
    The distributor brought a breach of contract action against the owner because he refused to pay for the tomatoes.
    How should the court rule on the owner’s offer to prove, over the distributor’s objection, that the chef refused to approve the tomatoes that were delivered?
    (A) The evidence is admissible to show frustration of purpose.
    (B) The evidence is admissible to show that the written agreement was subject to an oral condition precedent.
    (C) The evidence is barred, because the written contract appears to be a complete and total integration of the parties’ agreement.
    (D) The evidence is barred, because the oral agreement is within the statute of frauds.
A
  1. (B) This Multistate question presents an interesting interplay between Contracts and Evidence. Frequently, paroL evidence questions will involve the admissibility of an oral condition. Where the parties agree that a condition precedent must occur before the contract is effective, it is generally agreed that the failure of the condition to occur may be shown despite what otherwise would be deemed a total integration. Thus, even if there is a merger clause, it may be shown that the instrument was handed over to another with an oral condition attached to delivery. The theory is that the agreement is not to take effect until the condition occurs, and thus there is no contract to be added to or contradicted until that time. In this fact pattern, the owner will be permitted to introduce evidence of the chefs refusal to approve the tomatoes, since the entire agreement was subject to this condition precedent. Choice (C) is therefore wrong. Choice (A) is wrong, because generally a totaL frustration is required to discharge a contract. Here, it was still possible for the tomatoes to be used, so there was not a total frustration. Choice (D) is incorrect, because the facts state that the parties contracted in writing, so there was a sufficient writing to satisfy the Statute of Frauds.
How well did you know this?
1
Not at all
2
3
4
5
Perfectly
55
Q
  1. A law student was accepted by a law school for the fall term. Several generations of the law student’s family had attended this prestigious law school. One week before the start of the fall term, at a party to celebrate the law student’s acceptance to the school, his father announced to the law student, in the presence of the partygoers, “Son, it’s your obligation to uphold the family tradition for excellence at the school. In this regard, if you promise to study a minimum of five hours a day, I shall pay you $1,000 for each ‘A’ you achieve during your first year; $2,500 for each ‘A’ you achieve during your second year; and $5,000 for each ‘A’ you achieve during your third year.”
    The most accurate statement concerning the father’s promise to reward the law student for achieving “A’s” would be that
    (A) the promise constituted an unenforceable, conditional gift.
    (B) the promise would not be legally binding, since it was non-detrimental.
    (C) the promise would be enforceable if a bargained-for exchange was so intended.
    (D) the promise constituted a voidable proposal.
A
  1. (C) The promise would be enforceable if a bargained-for exchange was so intended. In
    the present hypo, the law student’s return promise to study for five hours per day
    would constitute adequate consideration, since he was under no legal duty to study
    at all. Under Restatement of Contracts 2d, Section 75, to constitute consideration,
    a performance or a return promise must be bargained-for. A performance or return
    promise is bargained-for if it is sought by the promisor in exchange for his prom
    ise and is given by the promisee in exchange for that promise. With respect to the
    requirement of consideration, the performance may consist of: (a) an act other than a promise, or (b) a forbearance, or Cc) the creation, modification, or destruction of a legal relation. Choice (A) is incorrect, because the presence of a bargained-for exchange would make this a contract rather than an unenforceable conditional gift. Choice (B) is incorrect, because the bargained-for exchange would provide the legal detriment. The law student was promising to study five hours per day and the father was promising to pay for each “A.” That would be a sufficient legal detriment to each party. Choice (D) is incorrect, because there is no reason why this deal should be considered voidable.
How well did you know this?
1
Not at all
2
3
4
5
Perfectly
56
Q
  1. On his 21st birthday, a son enlisted in the army. As he was about to leave for eight weeks of basic training, his father said, “I will buy you a new car at the end of basic training if you promise me not to take illegal drugs or drink alcohol while at basic training.” The son replied, “You can order the new car right away. I promise not to take illegal drugs or drink alcohol, as you requested.”
    At the end of basic training, the son was given a leave and returned home. His father asked, “Did you abide by your promise not to take illegal drugs or drink alcohol?” The son replied, “Yes, Father.” The father then told the son that he had already ordered the new car and that it would be available for delivery within one month.
    One week later, the son received the tragic news that his father had died suddenly. At the funeral, the executor of the father’s estate told the son that he did not feel compelled to give the son the newly arrived car.
    In a suit against the executor of the father’s estate to recover the new car, the son will
    (A) succeed, because the son’s promise to refrain from drinking alcohol provided sufficient consideration.
    (B) succeed, because the son’s promise to refrain from taking illegal drugs and drinking alcohol provided sufficient consideration.
    (C) not succeed, because the father’s promise was only a conditional gift.
    (D) not succeed, because the father’s promise was void.
A
  1. (A) In a bilateral contract, both promises must be legally binding orthe contractisvoid for lack of consideration. This is the doctrine of mutuality of obligation. Since the exchange for a promise must be something of value, and since value is tested in terms of detriment or benefit, it follows that a non-detrimental promise is insufficient consideration. In this regard, if the promise is wholly void for illegality (or is illusory), it is non-detrimental. Thus, suppose A makes a definite legal promise in exchange for a promise by B of an illegal performance or of one so indefinite in terms as to be incapable of enforcement. Here, B’s promise is void for illegality or for uncertainty, so A’s promise lacks consideration. A, the promisor, has received no legal benefit, for she has gained no enforceable right against B; and B has suffered no detriment, for he has undertaken no legal duty to A. This principle is often stated axiomatically, that “both parties must be bound or neither is bound.” However, in this question, note that only part of this promise is illegal; only the promise not to take illegal drugs would be void. The son is of legal drinking age, since he is 21. As a result, the part of the promise to refrain from drinking alcohol will still be enforceable, since the son gave consideration for the father’s promise by not drinking. Because of this, Choice (A) is a better choice than Choice (B). Choice (C) is incorrect, because the son’s promise to refrain from drinking alcohol would provide the necessary consideration to prevent this from being just a conditional gift. Choice (D) is wrong, because the valid consideration to refrain from drinking alcohol prevents the father’s promise from being voidable.
How well did you know this?
1
Not at all
2
3
4
5
Perfectly
57
Q
  1. A graduate from law school had decided against taking the bar exam. Her mother, who had always dreamed of having a child who was a lawyer, told the graduate, “It would mean a lot to me if you at least gave the bar exam a try. Tell you what; if you will devote the next two months to studying for the bar exam and then take the exam, I’ll give you $2,000. And if you pass I’ll give you an additional
    $2,000.” The graduate replied, “You’ve got a deal! I’m going to get started studying today!” The graduate’s uncle was present when this conversation took place. After the mother had left the room, the uncle said to the graduate, “I want you to know that if anything ever happens to your mother, I’ll pay you as per your mother’s promise.”
    The graduate spent the next two months studying diligently for the bar exam. When the graduate returned home after taking the exam, she received the tragic news that her mother had died suddenly. At the mother’s funeral, the uncle approached the graduate and told her that he did not believe that he was obligated to pay anything to the graduate.
    The graduate passed the bar exam and wishes to collect from the uncle. The uncle’s promise to the graduate would constitute
    (A) an enforceable promise, binding the uncle as a surety.
    (B) an unenforceable promise, because the graduate’s mother had a pre-existing duty to pay the graduate.
    (C) a voidable promise as violative of the statute of frauds.
    (D) a void promise at the time of inception.
A
  1. (C) The uncle’s promise to answer for the debt of the mother, in the event that the mother did not pay the graduate, would be a voidable promise as violative of the Statute of Frauds. It is important to remember that a contract may be voidable by one party by reason of (a) incapacity, (b) mistake, or, as in the present case, (c) under the Statute of Frauds. A promise to answer for the debt or default of another is required to be in writing under the Statute of Frauds. Since the uncle’s promise (to answer for the mother’s debt) was never reduced to writing, it constituted a voidable promise. Choice (A) is therefore wrong. Choice (B) is wrong, because the pre-existing duty was on the mother, not the uncle. The uncle was not under a preexisting duty to pay, so his promise constituted good consideration. Choice (D) is incorrect, because there is no reason why the promise should be void. It is voidable because of the lack of a writing, but not void from its inception.
How well did you know this?
1
Not at all
2
3
4
5
Perfectly
58
Q
  1. A woman owned a 10-acre tract of rural farmland in fee simple absolute. The woman agreed to sell the farmland to a man, and each signed a writing stating that the farmland was beitig sold: “…for $10,000, receipt of which is acknowledged.” In actuality, the man had not yet paid the woman the $10,000. At the date set for closing, the woman transferred a deed to the farmland to the man, who gave the woman a check for $10,000. Howevei, a few days after the woman deposited the check, she received notice from her bank that the check had not cleared, due to insufficient funds in the account.
    The woman then brought suit against the man. At trial, the woman seeks to testify that the man did not in fact pay her the $10,000 as recited in their written instrument. The man objects to the woman’s proposed testimony.
    Will the trial court judge be correct in sustaining the man’s objection?
    (A) No, because the parol evidence rule does not apply to events occurring after the forming of the writing.
    (B) No, because the parol evidence rule does not operate to exclude evidence to show lack or want of consideration.
    (C) Yes, because the written instrument appears to be a complete integration of the parties’ agreement.
    (D) Yes, because the doctrine of promissory estoppel will prevent the woman from denying her own signed acknowledgment that she received the $10,000.
A
  1. (B) When two parties have made a contract and have expressed it in a writing to which they have both assented as the complete and accurate integration of that contract, evidence, whether parol or otherwise, of antecedent understandings and negotiations will not be admitted for the purpose of varying or contradicting the writing. An exception to this general rule is where the evidence is being introduced to show lack of consideration. Therefore, (B) is the correct answer. Choice (A) is wrong, because this does not involve a post-writing event. This question is whether the woman received the consideration stated in the writing as already having been received. Choice (C) is wrong, because the exception to the parol evidence rule mentioned above would make the evidence admissible even if this were an integrated writing. Choice (D) is incorrect, because promissory estoppel is an argument to enforce a promise that has produced foreseeable detrimental reliance by the promisee. This is not the situation described here.
How well did you know this?
1
Not at all
2
3
4
5
Perfectly
59
Q
  1. After retiring from a long career in education, a teacher decided to leave the city and move to a community in the mountains. She began looking for a home to purchase in the mountain community. She found one suitable location offered for sale by the owner. The asking price for the home was $100,000. Although the teacher was very interested in purchasing the home, she told the owner that she was not sure it would fit within her budget. The owner the grabbed a piece of paper and wrote the following:
    “December 1
    In consideration of one dollar, receipt of which is acknowledged, I hereby offer to sell to you my home for $100,000. This offer shall remain open until 4:00 p.m. December 30.” The owner then signed the paper and handed it to the teacher, who left without paying the $1.
    On December 15, the owner sent the teacher a letter in which the owner stated, “Please be advised that I am hereby withdrawing my offer to sell you my home.” This letter was received and read by the teacher on December 16. The next day, the teacher tendered a cashier’s check to the owner in the amount of $100,001 (which included the $100,000 purchase price plus the $1 consideration) and demanded a deed of conveyance to the property. The owner refused to accept the cashier’s check.
    The teacher has now brought suit against the owner for specific performance. In her pleadings, the teacher admits that the recited $1 was not in fact paid to the owner when their written agreement was executed.
    Assume the teacher lives in a jurisdiction that follows the Restatement of Contracts. As a result, which of the following is the teacher’s best argument that such failure of payment does not bar her claim to specific performance?
    (A) The recited consideration was only a sham pretense of bargained-for consideration, and was therefore de minimis and of no legal significance.
    (B) The teacher’s inclusion of the $1 in her tendered check on December 17 was a timely cure of an immaterial breach of a bilateral real estate option contract.
    (C) By promising to hold the offer open until December 30 in a signed writing, the owner could not revoke the offer prior to December 30.
    (D) The written instrument embodied a proposal for a fair exchange within a reasonable time period and was therefore an enforceable option contract, regardless of whether the nominal consideration recited was bargained for or paid.
A
  1. (D) At common law, an offer is always revocable up to the time of acceptance, even though it contains a promise not to revoke. Where, however, the promise to keep the offer open is supported by consideration (i.e., something of value bargained for and given in exchange for the promise), it becomes legally binding. Choice (A) is wrong, because the overwhelming majority of cases hold that one dollar may be sufficient consideration for an option contract as long as a bargained-for exchange was so intended. The mere fact that the price exacted for the promise was one dollar (or less) will not defeat the contract on the ground of lack of consideration. Choice (B) is wrong, because the option to cure is something that exists under the UCC, which would not apply here. Choice (C) is incorrect, because it refers to the merchant’s firm offer rule, which only applies to a sale of goods. So, that leaves Choice (D). The Restatement of Contracts 2d, Section 87, states: “An offer is binding as an option contract if it is in writing and signed by the offeror, recites a purported consideration for making the offer, and proposes an exchange on fair terms within a reasonable time…” Footnote (c), in discussing this provision, states: “…the option agreement is not invalidated by proof that the recited consideration was not in fact given.” Therefore, the writing prepared by the owner is an enforceable option that cannot be revoked prior to December 30.
How well did you know this?
1
Not at all
2
3
4
5
Perfectly
60
Q
  1. A debtor owed a creditor $12,000 under a promissory note. Under the applicable statute of limitations, a suit to collect on the promissory note had to have been filed by September 30 of last year. On June 1 of this year, the creditor received a letter from the debtor stating, “I shall pay you $5,000 on July 1 in full satisfaction of what I owe you.” However, the debtor failed to make the payment on July 1.
    If, on August 1, the creditor brings suit against the debtor and the debtor asserts the statute of limitations as an affirmative defense and refuses to pay the creditor anything, which of the following accurately states the creditor’s legal rights against the debtor?
    (A) On June 1 the creditor became entitled to a judgment against the debtor for $5,000 only.
    (B) On July 1, not June 1, the creditor became entitled to a judgment against the debtor for $5,000 only.
    (C) On July 1, not June 1, the creditor will be entitled to a judgment against the debtor for
    $12,000.
    (D) The creditor is not entitled to anything, on either June 1 or on July 1.
A
  1. (B) An express promise by a debtor to pay all or part of an antecedent debt barred by the statute of limitations or by a decree in bankruptcy is legally enforceable without new consideration. The majority of jurisdictions require that a promise or acknowledgement of a debt barred by the statute of limitations must be in writing and signed by the debtor, but only a few states require the same formality as to the express promise to pay a debt discharged in bankruptcy. Choice (C) is wrong, because the original promise was barred by the statute of limitations and no longer enforceable. Choice (D) is wrong, because, based on the debtor’s new promise, the creditor is entitled to $5,000. The tough choice is deciding between Choices (A) and (B). However, it is not the date the promise was made that is crucial. The debtor promised to make the payment on July 1. The creditor had no right to collect on that promise until that date. When the debtor failed to make the JuLy 1 payment, a cause of action was created against the debtor to collect on the debtor’s broken promise. Choice (B) is therefore the best answer.
How well did you know this?
1
Not at all
2
3
4
5
Perfectly
61
Q
  1. After his 16th birthday, a youth decided to purchase a new sports car from a dealership for the sales price of $36,000. The youth thereafter entered into a written contract with the dealership, which provided that the youth would make monthly payments of $1,000 for three years. During the first year of the contract, the youth made 12 monthly payments totaling $12,000. However, the youth failed to make any payments during the second year of the contract.
    The dealership repeatedly threatened to sue the youth for the remaining balance that it claimed was due under the contract. Finally, in order to avoid litigation, the youth sent the dealership the following letter:
    “The car I purchased from you is a real lemon. I have spent a considerable sum of money during the last two years in repair work. I don’t believe that the car is worth $36,000, but I am willing to pay you $10,000 if I find ajob in the next month.”
    The youth had just turned 18 when he sent this letter to the dealership. The day after mailing this letter, the youth was involved in an automobile accident, and the sports car was totally demolished. Following the accident, the youth remained unemployed, and he refused to make any payments to the dealership. At the time of the accident, the sports car was worth $18,000. In this jurisdiction, the age of majority is
    18 years of age.
    If the dealership brings suit against the youth for breach of contract, what, if any, is the plaintiff’s proper recovery?
    (A) Nothing.
    (B) $10,000.
    (C) $18,000.
    (D) $24,000.
A
  1. (A) Another popular Multistate Contracts area deals with contracts of infants. It is important to note that contracts of an infant are voidable at the option of the infant. On the contrary, an adult party to the transaction cannot avoid the contract on the ground of the other’s infancy. In other words, the one-sided power of avoidance is held by the infant only. An infant may disaffirm his or her contract at any time prior to ratification. After reaching the age of majority, however, the infant may ratify or affirm the contract. Here, the youth ratified the contract by his letter, mailed shortly after his 18th birthday. It is the promise made in the ratification that is enforceable, not the original promise. In the present case, the youth’s affirmation (namely, his promise to pay the dealership $10,000) was conditioned upon his finding a job. Since that condition apparently did not occur (because the accident took place the day after the mailing of the letter), the youth will not be liable under the contract. Choices (B), (C), and (D) are all therefore wrong, because the youth will not owe any money, since he will not be able to meet the condition.
62
Q
  1. After riding the bus to and from her high school for two years, a girl decided that she was tired of relying on the bus and wanted her own mode of transportation. Since she had just turned 16 and received her driver’s license, she decided to purchase a motor scooter from a local retailer for $12,000. The written contract with the retailer provided that the girl would make monthly payments of $500 for two years. During the first year of the contract, the girl made 12 monthly payments totaling $6,000. However, the girl failed to make any payments during the second year of the contract.
    The retailer repeatedly threatened to sue the girl for the remaining balance that it claimed was due under the contract. However, the retailer never followed through on this threat. Shortly after the girl turned 18, she sent the following letter to the retailer:
    “I had to stop making payments because I ran out of money. I am hoping that will change shortly. However, I’ve been told I am not obligated to pay anything more to you, because I was underage when we entered into the contract. But I want to be fair. I am willing to pay you $3,000 by the end of the year if my financial position improves.”
    The girl never made any further payment to the retailer.
    If the retailer brings suit to recover damages from the girl, which of the following, if it were found to be true, would provide the retailer with its best argument for recovery?
    (A) The motor scooter was demolished in an accident shortly after the girl sent the letter.
    (B) The retailer was unaware that the girl was a minor at the time the contract was formed.
    (C) The girl’s financial position has improved to the extent that she is able to pay the $3,000 she had promised to pay.
    (D) The motor scooter that the girl purchased is a necessary of life.
A
  1. (C) When the girl reached the age of majority, she promised to pay the retailer $3,000. It is important to note that the promise was conditioned upon the girl’s financial condition improving. Therefore, Choice (C) is the best answer, since the girl’s liability is based upon her subsequent conditional promise. The initial contract between the girl and the retailer is voidable because the girl was a minor at the time of execution. While the girl ratified the contract in her letter, sent shortly after she turned 18, only the ratified promised can now be enforced. Choice (A) is wrong, because the fact that the motor scooter had been destroyed would not prevent enforcement of the promise the girl made when she turned 18. Moreover, choice (A) will not help the retailer, because it does not explain why the girl should now be required to pay the $3,000. But if the financial position improved, that would help the retailer recover, because of the conditional promise made, so(C) is a better choice. Choice (B) is incorrect, because the contract is voidable by the minor regardless of whether the other party is aware of the minority so this argument does not help the retailer. Choice (D) is wrong, because while it is true that an infant may be held liable in quasi-contract for the reasonable value of necessaries, this principle generally applies to such items as food, shelter, clothing and medical treatment.
63
Q
  1. On February 1, a homeowner entered into a contract with a famous landscape architect, to landscape her five-acre estate. The homeowner entered into the contract with the architect because she was pleased with the work he had done on her neighbor’s gardens. The architect promised to complete the job before May 30, the date of the homeowner’s annual garden party. On March 1, the homeowner left for a month’s vacation. Unknown to her, the architect, because of previous commitments, assigned the contract to a gardener. After the gardener had completed 60 percent of the work, the homeowner returned and was displeased with the landscaping already completed. The homeowner contacted the architect, who informed her of the assignment to the gardener and told her that he was no longer obligated under their original contract. Disgruntled, the homeowner fired the gardener.
    If the homeowner asserts a claim against the architect based on breach of contract, the fact that the architect assigned the contract to the gardener will
    (A) relieve the architect of liability, because the contract was assignable.
    (B) relieve the architect of liability, because the gardener’s work was the cause of the homeowner ‘s dissatisfaction.
    (C) not relieve the architect of liability, because a personal service contract of this nature is non- delegable.
    (D) not relieve the architect of liability, unless there was an express contractual provision against assignments.
A
  1. (C) First, it should be noted that an assignment of the contract is both an assignment of rights and a delegation of duties, unless stated otherwise. So, by assigning the contract, the architect attempted to delegate his duty to the gardener. All duties can be delegated unless the personal skill, expertise or identity of the actor is a material segment of the contract. Clearly, in the instant case, the homeowner contracted for the personal landscaping services of the architect. This would make it a non-delegable duty. Choices (A) and (B) are wrong, because, even if the duty was delegable, a delegation of duties still does not relieve the delegator of liability absent a novation, which did not occur here. Choice (D) is incorrect because an express contractual prohibition would have resulted in the delegation’s being void, as this was a personal services contract.
64
Q
  1. A homeowner hired a painter to paint his house for
    $10,000. The day before the painter was to begin work, he decided to go on vacation for a month. The painter telephoned a handyman who had done painting work for the painter in the past, and asked if he would be willing to paint the homeowner’s house for him. The painter said that if the handyman would paint the house, the handyman could keep the $10,000 that the homeowner promised to pay. The handyman agreed.
    The next day, the handyman went to the homeowner’s house to begin the painting job. The homeowner had already left for work, so the handyman began to paint the house. When the homeowner arrived home that evening, he saw the handyman painting his house. Confused, the homeowner asked the handyman why he was doing the painting instead of the painter. The handyman told the homeowner of the arrangement he had with the painter. Although the handyman was only 50% finished with the painting, the homeowner told the handyman to immediately get off his land, so the handyman left.
    If the handyman brings an action against the homeowner, he will most likely
    (A) recover for unjust enrichment.
    (B) recover under the doctrine of substantial performance.
    (C) not recover, because his arrangement with the painter was not in writing.
    (D) not recover, because there was no contract between the handyman and the homeowner.
A
  1. (A) This is a classic example of a “best of the lot” question. Remember, you are to pick the best answer of the fouryou are given to choose from. You may not always be presented with a perfect answer; you are to choose the best option provided. Choice (B) is not the best choice, because the handyman has not yet substantially performed. He has finished only 50 percent of the job. With substantial performance, the other party is receiving substantially what he bargained for. That clearly did not occur here. Choice (C) is wrong, because there is no reason why the contract between the painter and the handyman needed to be in writing. It does not fall under any provision of the Statute of Frauds. Choice (D) is incorrect, because the handyman was a delegate of the contract between the painter and the homeowner, and since the duty was delegable, the handyman can sue the homeowner. That leaves Choice (A). The handyman will be able to recover for the reasonable value of his work on the theory of quasi-contract to prevent injustice by allowing unjust enrichment (to the defendant). Students should note that the handyman most likely would have been able to sue the homeowner for expectation damages for the homeowner’s breach, but that was not among the choices provided.
65
Q
  1. A husband and wife were about to leave on vacation for one month. They thought it would be a great idea to have new carpeting installed in their home while they were away. They contacted a reputable carpeting company, which offered to do the job for $7,000. The husband and wife agreed, gave a set of house keys to the carpeting company, and left on vacation.
    A week later, the carpeting company discovered that they had overbooked their installation staff and could not finish all of the jobs for which they had contracted. The carpeting company contacted a workman whom they had used before, and told him about the job they were to perform for the husband and wife. The workman agreed to do the installation in exchange for the payment from the husband and wife.
    When the husband and wife returned from vacation, they were happy with the work and sent a check for $7,000 to the carpeting company, which deposited the check.
    If the workman now seeks to recover for services he performed, he will
    (A) recover against the husband and wife only.
    (B) recover against the carpeting company only.
    (C) recover against either the husband and wife or the carpeting company.
    (D) not recover, because he was not in privity of contract.
A
  1. (B) The workman was an assignee of the contract between the carpeting store and the husband and wife because the carpeting store assigned the right to receive payment form the husband and wife. However, an obligor is only obligated to perform to the assignee when notified of the assignment. Here, neither the carpeting store nor the workman informed the husband and wife of the assignment, so the sending of the check to the carpeting store was proper performance, which fulfilled their contractual obligation. So Choices (A) and (C) are incorrect, because the workman cannot recover from the husband and wife. Choice (D) is wrong, because, at the very least, the workman was in privity of contract with the carpeting store in that they entered into a contract for the workman to perform the installation on behalf of the carpeting store. Choice (B) is therefore the best answer. The workman is entitled to recover from the carpeting store for their breach of contract in that they promised the payment to the workman, but did not give it to him.
66
Q
  1. A patient went to a doctor for treatment of an abnormal growth on the back of her right shoulder. Under a written contract, the doctor agreed to surgically remove the growth for $750.
    As agreed, the surgery was performed on an outpatient basis at the doctor’s office. He applied a localized anesthesia that enabled the patient to remain conscious during surgery. While the operation was being performed, the doctor noticed a brown mole on the patient’s back. As the doctor subsequently recalled, he informed the patient about the mole and asked whether she wanted it removed as well. According to the doctor’s account, the patient orally agreed to have the mole removed for an additional $150. The patient has no recollection of any such oral agreement.
    The doctor erformed both removal procedures satisfactorily. Following the surgery, the patient paid the doctor $750, but refused to pay any additional money for the mole removal. After unsuccessfully attempting to collect the $150, the doctor brought suit against the patient to recover the additional fee. At trial, the patient objected to the introduction of testimony concerning any oral agreement made during surgery.
    Which of the following considerations is most important to a judicial ruling on the patient’s objection?
    (A) The doctor’s removal of the mole conferred a clear benefit on the patient.
    (B) The proffered oral agreement regarding payment of the additional $150 occurred after the written agreement had been signed.
    (C) The written agreement was for services rather than the sale of goods.
    (D) The proffered oral agreement regarding payment of the additional $150 concerned a new medical procedure that was unrelated to the subject matter of the prior written contract.
A
  1. (B) The parol evidence rule states that once the parties have reduced their agreement to a writing, evidence of any prior oral or written, or contemporaneous oral agreements is inadmissible to alter, vary, or contradict the terms of the writing. The parol evidence rule has never prevented proof of an oral or written agreement that varies or contradicts the terms of a prior written contract. Since the oral agreement, which the doctor claims took place during the operation, occurred at a point in time after the written contract to perform the surgery had been signed, the doctor will contend that the parol evidence rule is not applicable as applied to asubsequentoral agreement. Choice (B) correctly states this consideration. Choice (A) is incorrect, because the conferral of a clear benefit would be relevant to recovery in quasi-contract for the value of services rendered; however, quasi-contract recovery is unavailable where the parties already have a valid existing contract. Choice (C) is incorrect, because the Statute of Frauds is not a relevant consideration. Even though the Statute of Frauds may not apply to personal service contracts, the parol evidence rule must be considered whenever an existing written contract is affected by a prior or contemporaneous oral agreement. Choice (D) is incorrect, because it states an irrelevant consideration. Regardless of the nature of the second procedure, the payment for it will be determined by the court’s interpretation of the subject matter of the original written contract.
67
Q
  1. A woman took her car to a mechanic to be serviced. After doing a thorough inspection of the car, the mechanic told the woman that her car needed extensive work and that he could do all that was needed for $1,000. The parties then entered into a written contract to have the car serviced for $1,000. As the woman remembers it, at the time the written contract was formed, the parties orally agreed that if the car required any additional servicing above that detailed in the contract, it would be included in the $1,000 fee. The mechanic, however, has no recollection of any such oral agreement.
    While servicing the car, the mechanic discovered that a belt was severel’ worn and need to be replaced. The belt replacement was not listed as work to be performed in the written contract. The mechanic telephoned the woman and told her he had found the worn belt and felt he should replace it. The woman told him to do so.
    When the mechanic was finished, he presented the woman with a bill for $1,100, representing the $1,000 called for by the contract and $100 for the replacement of the worn belt, which is a reasonable amount. The woman refused to pay any more than
    $1,000.
    The mechanic sued the woman to collect the $100 for the replacement of the worn belt. If the court rules in favor of the mechanic, which of the following is the most likely cause for such a ruling?
    (A) The writing was construed as a complete integration of the parties’ agreement.
    (B) The writing was construed as a partial integration of the parties’ agreement.
    (C) An implied-in-law contract was created at the moment the mechanic replaced the worn belt.
    (D) The parties entered into a collateral oral agreement for the replacement of the worn belt that was excludable from the integrated writing.
A
  1. (A) If the court agrees with the mechanic that the woman is obligated to pay $100, it will be because the written contract was construed as a complete integration of the parties’ agreement. Choice (A) is correct. As a general rule, where there is complete integration of a writing, no extrinsk parol evidence is admitted. The rationale is that since the writing embodies the full and final expression of the parties’ agreement, it is complete on its face and may not be altered or varied in any way by extrinsic evidence in the form of parol. Choice (B) is incorrect, because if the court found the written contract merely to be partial integration of the parties’ agreement, then the parol evidence would be admissible, and the mechanic would be obligated to replace the worn belt as part of the original contract. Choice (C) is incorrect, because the doctrine of quasi-contract, or contract implied in law, is used to impose a legal obligation and afford a remedy, without which injustice would result, under circumstances where it is clear no promise was ever made or intended. The written contract between the parties clearly evidences that a promise was made. Choice (D) is incorrect, because the subject matter required to make an oral agreement “collateral”to a written agreement must be so far separate that it is not covered, dealt with, or even mentioned in the writing. Therefore, any oral agreement involving service to the car could not have been collateral to a written agreement for similar work during the course of service.
68
Q
  1. An actress went to a plastic surgeon for removal of an abnormal growth on her right cheek. Under a written contract, the plastic surgeon agreed to surgically remove the growth for $1,000. As the actress subsequently recalled, just before signing the agreement the parties orally agreed that if any additional medical procedures were required during surgery, they would be included in the $1,000 fee. The plastic surgeon, however, has no recollection of any such oral agreement.
    As agreed, the surgery was performed on an outpatient basis at the plastic surgeon’s office. She applied a localized anesthesia that enabled the actress to remain conscious during surgery. While the operation was being performed, the plastic surgeon noticed another growth on the actress’s neck. As the plastic surgeon subsequently recalled, she informed the actress about the growth and recommended that it be removed as well. According to the plastic surgeon’s account, the actress orally agreed to have the growth removed for an additional $500. The actress has no recollection of any such oral agreement.
    The plastic surgeon performed both removal procedures satisfactorily. Following the surgery, the actress paid the plastic surgeon $1,000, but refused to pay any additional money for the removal of the second growth. After unsuccessfully attempting to collect the $500, the plastic surgeon brought suit against the actress to recover the additional fee. At trial, the plastic surgeon objected to the introduction of testimony regarding any oral agreement made before the written contract was signed. By the same token, the actress objected to the introduction of testimony concerning any oral agreement made during surgery.
    Will the plastic surgeon be entitled to recover the additional $500 if both offerings of parol evidence are admitted and believed?
    (A) No, because the oral agreements contradict each other; neither would be enforceable under the so-called knock-out rule.
    (B) No, because in removing the growth from the actress’s neck, the plastic surgeon performed a legal duty already owed her that was neither doubtful nor the subject of an honest dispute.
    (C) Yes, because the oral modification of the prior writing was fair and equitable in view of circumstances not anticipated by the parties when the writing was executed.
    (D) Yes, because the actress would be unjustly enriched by not paying the additional $500, and injustice can be avoided only by enforcement of her oral promise.
A
  1. (B) The key to this question is to realize the effect of the court’s admission of the
    actress’s oral testimony. If it is believed that the original written agreement incor
    porated in its terms the understanding that “any additional medical procedures…
    would be included in the $1,000 fee,” then the plastic surgeon would be under a
    pre-existing duty to perform the second procedure, removing the growth from the
    actress’s neck for no additional charge. Therefore, the plastic surgeon will not be
    entitled to recover the $500. Only Choices (A) and (B) reach the correct conclusion.
    Since the so-called knock-out rule applies only to UCC situations, Choice (B) is cor
    rect. Choice (C) is wrong, because by admitting the actress’s testimony that addi
    tional medical procedures are to be included, the circumstances encountered were
    not unanticipated by the parties but specifically addressed. Choice (D) is incorrect,
    because there would be no unjust enrichment. The plastic surgeon agreed to do all
    work for the $1,000 fee, if the actress’s testimony is admitted and believed.
69
Q
  1. The owner of a large family estate decided to sell the property. The owner entered into a valid written brokerage agreement with a real estate broker. According to terms of the agreement, the broker promised to undertake best efforts to sell the estate for a purchase price of not less than $1,250,000. The contract also provided that the broker was to be paid a commission of 5 percent on the gross sale price following the consummation of the sale and transfer of title. The brokerage agreement was effective for a period of two months.
    One month later, the broker notified the owner that she had found a buyer who wanted to purchase the property for $1,500,000. That same day, the broker handed the owner a real estate sales contract, signed by the buyer, in which the buyer agreed to pay $1,500,000 for the purchase of the estate.
    The owner investigated the buyer and discovered he had sufficient funds to purchase the estate. However, the owner changed his mind and decided not to sell the property. He told the broker that he would not sign the agreement. In addition, the owner told the broker that he was canceling their brokerage agreement because he was withdrawing the estate from the market. The owner also refused to pay the broker any commission.
    If the broker now sues the owner for breach of contract, which of the following, if anything, is the broker’s proper measure of recovery?
    (A) Nothing, because a condition precedent, namely the consummation of the sale and transfer of title, has failed to occur.
    (B) Quantum meruit for the reasonable value for services rendered in obtaining a buyer for the property.
    (C) $75,000, or the commission equivalent of 5 percent on the sale of the property for
    $1,500,000, since all conditions precedent to the owner’s duty to pay the commission were fulfilled when the broker produced a buyer who was ready, willing and able to perform.
    (D) $75,000, or the commission equivalent of 5 percent on the sale of the property for
    $1,500,000, since the consummation condition to the owner’s duty to pay the commission was excused by the owner’s refusal without cause to accept the buyer’s offer and perform the land sale contract.
A
  1. (D) In this Multistate example, students are being tested on the rule dealing with hindrance or failure to cooperate as an excuse of condition. The brokerage agreement between the owner and the broker provided that the broker was to be paid a commission “following the consummation of the sale and transfer of title.” Thus, the condition (i.e., the consummation of sale) would be viewed as a condition precedent, since it must occur before the owner’s duty of performance can arise (i.e., his obligation to pay the commission). However, after the broker found a buyerwho was willing to perform, the owner decided not to go through with the sale. In this situation, would the broker be entitled to her commission? The answer is yes, because in this case an implied duty of performance is imposed upon the seller. Choice (A) is therefore wrong. Choice (B) is incorrect, because quantum meruit would be a restitutionary remedy that is imposed when there is not an enforceable contract between the parties. Since the broker is entitled to recover on the contract, restitution is not needed. Choice (C) is not the best answer, because it states that all conditions were fulfilled. That is incorrect. The condition requiring consummation of sale was not fulfilled, because of the owner’s failure to cooperate.
70
Q
  1. A nephew inherited a large parcel of unimproved land from his uncle. In need of cash, the nephew decided to sell the parcel. He contacted a real estate agent in the area about listing the parcel for sale. The nephew and the agent entered into a valid written contract whereby the agent promised to undertake best efforts to find a buyer for the parcel. The contract also provided that the agent was to be paid a commission of 5 percent on the gross sale price following the consummation of the sale and transfer of title.
    The agent succeeded in finding a buyer for the parcel. The agent notified the nephew that he had found a developer who wanted to purchase the parcel for $500,000. The agent handed the nephew a real estate sales contract, signed by the developer, in which the developer agreed to pay $500,000 for the purchase of the parcel.
    The nephew then signed the agreement himself. However, before consummation of the sale and transfer of title, the developer, without cause, repudiated the contract. Despite the agent’s insistence, the nephew refused to either sue the developer to enforce the land sale contract or pay the agent his commission.
    If the agent sues the nephew for breach of the brokerage agreement, which of the following, if anything, is the agent’s proper measure of recovery?
    (A) Nothing, because as a third-party beneficiary of the contract between the nephew and the developer, the agent can enforce the contract only against the developer, but not against the nephew.
    (B) Nothing, because the consummation condition has not been fulfilled, and the nephew’s refusal to sue the developer will not excuse that condition.
    (C) $25,000, or the commission equivalent of 5 percent on the sale of the property for
    $500,000, because fulfillment of the consummation condition was prevented by an event beyond the agent’s control.
    (D) $25,000, or the commission equivalent of 5 percent on the sale of the property for $500,000, because all conditions precedent to the nephew’s duty to pay the commission were substantially fulfilled when the nephew and the developer entered into the land sale contract.
A
  1. (B) In the case of Amies v. Wesnofske [174 N.E. 436 (1931)], the plaintiff’s right to a brokerage commission from the vendor was, by agreement, conditioned upon closing of title. The vendee defaulted under the contract, but the vendor took no legal action against the defaulting vendee. The broker insisted that the condition was excused because of the vendor’s failure to bring an action for specific performance against the vendee. The court held, however, that the duty to cooperate did not extend to bringing an action forspecific performance. Choice (A) is wrong, because the agent is not asserting his rights as a third-party beneficiary. Rather, he is suing the nephew under the terms of their real estate brokerage agreement. Choice (C) is incorrect, because, as explained above, the nephew’s failure to bring an action for specific performance is not viewed as an act of wrongful prevention. Choice (D) is incorrect, because it states that all conditions precedent were substantially fulfilled. The condition precedent that there be a sale and transfer of title has not occurred, nor was it substantially fulfilled.
71
Q
  1. On September 1, a manufacturer of portable drinking fountains mailed to a drinking fountain retailer a signed offer that stated: “Have 200 drinking fountains available at $100 each for October delivery. Be advised that this offer will remain open until October 1.”
    On September 30, the retailer mailed the following letter, which was received by the manufacturer on October 1: “Your offer is hereby accepted, but request delivery of 100 drinking fountains in October and 100 in November.”
    However, on September 29, the manufacturer sent a fax to the retailer revoking its offer, which was received the same day.
    This revocation is
    (A) valid, because the retailer had not changed its position in reliance on the September 1 offer.
    (B) valid, because there was no consideration to support an option contract.
    (C) not valid, because the retailer had 90 days in which to accept.
    (D) not valid, because the manufacturer gave assurance that the offer would remain open until October 1.
A
  1. (D) According to UCC Section 2-205, “an offer by a merchant to buy or sell goods in a signed writing which by its terms gives assurance that it will be held open is not revocable, for lack of consideration, during the time stated or if no time is stated for a reasonable time, but in no event may such period of irrevocability exceed three months; but any such term of assurance on a form supplied by the offeree must be separately signed by the offeror.” Under the “firm offer” rule, the manufacturer’s offer would remain open until October 1. Therefore, Choice (D) is correct. Choices (A) and (B) are wrong, because the offer becomes irrevocable as soon as it is made by the merchant. Neither consideration nor detrimental reliance on the offer is required. Choice (C) is wrong, because the manufacturer stated that the offer would be open until October 1, not for 90 days.
72
Q
  1. On January 1, a manufacturer of widgets received an e-mail from a widget distributor about purchasing widgets. The manufacturer sent the following returh e-mail: “Have 1,000 widgets available at $10 each for February delivery. Be advised that this offer will remain open until February 1.”
    On January 31, the distributor sent the following fax to the manufacturer: “Your offer is hereby accepted, but request delivery of 500 widgets in February and 500 widgets in March.” The manufacturer received the fax the same day, but did not respond.
    Which of the following is the most accurate statement regarding the legal effect of the distributor’s January 31 fax?
    (A) It constitutes a counteroffer, because it contains different terms from those contained in the original offer.
    (B) It constitutes a rejection, because the offer implied limited acceptance to the terms contained therein.
    (C) It creates an enforceable contract with delivery of 500 widgets in February and delivery of 500 widgets in March.
    (D) It creates a reformation integrating the terms of both writings.
A
  1. (C) Under UCC Section 2-207, “A definite and seasonable expression of acceptance
    operates as an acceptance even though it states terms additional to or different from those offered…unless acceptance is made expressly conditional on assent to the additional or different terms.” Here, the fax was a definite and seasonable acceptance, so it operates as an acceptance. Choices (A) and (B) are therefore wrong. As to the fate of the additional terms in the acceptance, the UCC says “…additionaL terms are to be construed as proposals for addition to the contract. Between merchants such terms become part of the contract unless: (a) the offer expressly limits acceptance to the terms of the offer; (b) they materially alter it; or (c) notification of objection to them has already been given or is given within a reasonable time after notice of them is received.” Here, the additional term regarding February and March delivery did not materially alter the contract, nor was the offer expressly limited. Thus, it will become part of the agreement unless the manufacturer objects within a reasonable time. Choice (C) is therefore the best answer. Choice (D) is wrong, because reformation does not apply here. Reformation is where a writing is altered to reflect the true agreement of the parties. No alteration is required here.
73
Q
  1. On March 1, a homeowner and a painter entered into a written contract wherein the painter promised to paint the exterior of the homeowner’s house for $3,000. Prior to the signing of the contract, the homeowner and the painter orally agreed that their contract would be null and void unless the homeowner was able to obtain a $3,000 loan from National Bank before April 1. On March 31, the homeowner was informed by National Bank that his loan application had been rejected. The next day, the homeowner telephoned the painter and informed him that the deal was off.
    If the painter brings an action for breach of contract against the homeowner, would the latter’s inability to secure the loan provide him with a valid defense?
    (A) No, because the homeowner is estopped to deny the validity of the written contract.
    (B) No, because the agreement regarding the loan varied the express terms of the writing.
    (C) Yes, because the agreement regarding the loan constituted a valid modification of the writing.
    (D) Yes, because the loan agreement was a condition precedent to the existence of the contract.
A
  1. (D) Where the parties agree that a condition precedent must occur before the contract is effective, it is generally agreed that the failure of the condition to occur may be shown despite what otherwise would be deemed a total integration. The theory is that the agreement is not to take effect until the condition occurs, and thus there is no contract to be added to or contradicted until that time. Therefore, the admission of the oral condition is not barred by the parol evidence rule. Choice (B) is therefore wrong. Choice (A) is incorrect, because estoppel would not apply here. The homeowner is entitled to assert that a valid contract was never formed under the rule stated above. Choice (C) is incorrect, because the agreement concerning the loan occurred before the contract was signed. A modification is an agreement between the parties entered into after the contract is formed.
74
Q
  1. An owner entered into a written contract with a landscaper whereby the landscaper promised to landscape the exterior of the owner’s house for the sum of $5,000. According to their agreement, the owner was to pay the money to the landscaper’s son. The landscaper intended to have the $5,000 given to his son as a graduation present.
    After the landscaper completed the job, the landscaper requested that the owner pay the $5,000 to him instead of to his son. The owner paid the landscaper. The next day, the son learned of the contract between the landscaper and the owner, as well as the payment to his father.
    In an action by the son against the owner for $5,000, the plaintiff will most likely
    (A) prevail, because the written contract between the owner and the landscaper operated as a valid assignment to the son.
    (B) prevail, because the son was the intended beneficiary under the terms of the written contract between the owner and the landscaper.
    (C) not prevail, because the owner and the landscaper effectively modified their agreement, thereby depriving the son of any rights he may have had.
    (D) not prevail, because the son did not give any consideration.
A
  1. (C) Students should be aware that after the right created by a third-party contract has vested in the beneficiary, subsequent rescission and release of the promisor by the promisee is inoperative to affect the right. By the majority view, however, modification or rescission is possible up to the time the third-party’s rights vest. A third-party beneficiary’s rights vest when he learns of the contract and 1) manifests assents to it at the request of one of the parties, 2) materially changes position in reliance on the contract, or 3) files suit to enforce rights under the contract. In the present example, the son’s rights did not vest, because he did not learn of the contract until after the landscaper and the owner had effectively modified their agreement. Choice (B) is therefore wrong, because the modification eliminated the son’s rights under the contract. Choice (A) is incorrect, because the son is a third-party beneficiary of the contract between the landscaper and the owner, not an assignee. Choice (0) is incorrect, because a third-party beneficiary does not have to give consideration to acquire rights under the contract.
75
Q
  1. The owner and operator of a pizza restaurant entered into a written contract with an employee that provided that the employee would be employed as manager of the restaurant for a period of three years. The contract provided that the owner was to pay the employee a salary of $2,000 per month, payable one-half to the employee and one-half to the employee’s elderly mother. The contract further stipulated that “the monies due hereunder shall not be assignable.”
    When the mother learned of this contract, she wrote a letter to the owner stating, “Kindly pay the amounts due me under your contract with my son directly to the nursing home, where I am presently a patient.” During the first year of the contract, the owner paid $1,000 per month to the employee and $1,000 per month to the nursing home.
    After one year, another pizza restaurant opened across the street, creating competition. During the next few months, business steadily declined. As a result of the loss in business, the owner informed the employee that unless he agreed to take a cut in his salary, the employee would be fired. Reluctantly, the employee orally consented to a salary reduction of $500 per month. By the terms of their oral agreement, the owner promised to continue to pay $1,000 to the employee, but to pay only $500 per month to the nursing home.
    If the nursing home brings suit against the owner for breach of contract, who is most likely to prevail?
    (A) The owner, because the mother’s assignment to the nursing home was void as violative of the anti-assignment clause.
    (B) The owner, because the agreement between the owner and the employee reduced the owner’s obligations to the nursing home.
    (C) The nursing home, because the mother’s assignment would be enforceable despite the agreement between the owner and the employee.
    (D) The nursing home, because the mother’s gratuitous assignment was irrevocable.
A
  1. (C) First, the mother was an intended third-party beneficiary under the terms of the owner-employee contract. In accordance with the Restatement of Contracts 2d, Section 302, “unless otherwise agreed between promisor and promisee, a beneficiary of a promise is an intended beneficiary if recognition of a right to performance in the beneficiary is appropriate to effectuate the intention of the parties and…the circumstances indicate that the promisee intends to give the beneficiary the benefit of the promised performance.” It is important to note, however, that an intended beneficiary can only enforce a contract after his/her rights have vested. One of the ways that a beneficiary’s rights can vest is when the beneficiary learns of the contract and assents to it. In the present case, the mother’s rights as an intended third-party beneficiary clearly vested when she learned of the contract and wrote to the owner, impliedly assenting to the terms of the deal. Moreover, the parties to a contract (namely the owner and the employee) cannot by agreement discharge, modify, or preclude themselves from varying their duties to each other by subsequent agreement unless the beneficiary consents. Therefore, Choice (C) is correct, because the owner-employee agreement modifying the contract would be ineffective. Choice (B) is therefore wrong. Choice (A) is not the best choice. While it is arguable that the prohibition on assignments may be enforceable, if the obligor consents to the assignment, the prohibition will not enforced. Here, the owner consented to the assignment by making payments to the nursing home. Choice (D) is wrong, because a gratuitous assignment is revocable. However, since the mother never revoked the assignment it remains in effect.
76
Q
  1. Expecting an increase in business for the holiday season, an owner of a retail store decided to hire another employee. On October 31, the owner entered into a written contract with a worker that provided that the worker would be employed for the months of November and December to work at the retail store for $2,000 per month.
    At the end of November, the owner noticed that business had not increased as much as he had expected. The owner informed the worker that unless he agreed to take a cut in his salary, he would be fired. Reluctantly, on December 1, the worker orally consented to a salary reduction of $500 for the month of December.
    Which of the following statements regarding the December 1 agreement between the owner and the worked is most accurate?
    (A) It effectuated a valid reformation of their original written contract.
    (B) It effectuated a valid modification of their original written contract.
    (C) It effectuated a novation of their original written contract.
    (D) It did not alter the rights and obligations of the parties under the terms of their original contract.
A
  1. (D) Under the pre-existing duty rule, neither doing nor promising to do or refrain from doing that which one is already legally bound to do or refrain from doing cannot furnish consideration for a promise. In such case, there is no bargained-for exchange. As a consequence, under the pre-existing duty rule, since the owner wa legally bound to pay the worker a salary of $2,000 per month, the December 1 agreement would be unenforceable as it was not supported by any new consideration. Choice (B) is therefore incorrect. Choice (A) is wrong, because a reformation consists of changing a writing to conform to the true agreement of the parties. That is not the situation presented by this fact pattern. Choice (C) is incorrect, because a novation is a three-party agreement that grows out of a delegation. That also is not the situation presented by this fact pattern.
77
Q
  1. An employee was hired under a written contract to work for an employer for five years at a monthly salary of $1,500. In addition, the employment contract provided that the employer would pay the employee an annual bonus “to be determined by the employer within two weeks after the year’s profits have been ascertained.”
    At the end of the first year, the business showed a net profit of $50,000. Accordingly, the employer and the employee agreed that the employee should receive a bonus of $5,000. This amount was subsequently paid to the employee.
    At the end of the second year, the business realized a net profit of $10,000. Once the profit had been calculated, the employer telephoned the employee and offered him a second-year bonus of $1,000. The employee refused and demanded a bonus of $2,000. The employer refused the employee’s demand and immediately terminated his employment.
    Following his dismissal, the employee brought suit for breach of contract against the employer. Which of the following is the most accurate statement?
    (A) The employee has a cause of action against the employer to recover a bonus for the second year in an amount to be determined by the court.
    (B) The employer is excused for his obligation to pay a bonus, because of the unforeseen downtum in business during the second year.
    (C) The employee is not entitled to any recovery, because employment contracts are generally terminable at will.
    (D) The employee is not entitled to a bonus, but the employee can sue for loss of wages.
A
  1. (D) The bonus provision is unenforceable, because it is illusory. If, at the formation stage, one of the parties does not incur legal detriment because he retains an unfettered election to perform or not, his promise is illusory. The employer’s promise to pay a bonus “to be determined by the employer…after the year’s profits have been ascertained” would fail for lack of consideration because the employer has the choice to not perform if the employer simply believes that the profits were not high enough to warrant paying a bonus. Because the empLoyer’s promise to pay the bonus will fail, Choice (A) is incorrect. Choice (B) is not the best choice, because the employer’s promise is unenforceable as illusory regardless of whether the business experienced a downturn in business. Choice (C) is incorrect. While it is true that employment contracts are generally terminable at will, this employment contract had a stated duration (five years). Therefore, it is not an at-will employment contract, and the employee would be entitled to recover for his lost wages over the remaining three years of the contract, subject to his duty to mitigate damages.
78
Q
  1. On January 1, an employee is hired to work for an employer for one year at a weekly salary of $500. After six months the employee is fired.
    In determining what rights the employee has against the employer, the agreement between the employer and the employee may best be interpreted as
    (A) an entire contract.
    (B) a divisible contract.
    (C) an installment contract.
    (D) neither divisible nor entire.
A
  1. (B) A divisible contract is one in which the parties have divided their respective performances into separate units, so that performance of an installment on one side entitles such party to the other’s performance of that installment. In this regard, employment contracts are regarded as divisible for the purpose of permitting the employee to recover the agreed price for the number of months of service he has put in. Choices (A) and (D) are therefore incorrect. Choice (C) is incorrect, because installment contracts generally apply to a sale of goods where the goods are to be delivered in more than one lot.
79
Q
  1. A homeowner was interested in changing the look of his home. He contacted an architect, who designed plans and specifications for a major “facelift” of the homeowner’s home. The homeowner was very pleased with the plans and specifications. He showed the plans and specifications to his neighbor, who agreed that the facelift would greatly enhance the value of the homeowner’s home.
    The homeowner contacted a contractor, who was considered to be the best in the area, to rebuild his house according to the plans and specifications. The contractor reviewed the plans and specifications and told the homeowner that he could do all the work called for by the plans and specifications for $100,000. The homeowner replied, “That’s a little more than I can afford. I really can only free up $80,000 for this project. Would you do it for that amount?” The contractor responded, “That’s awfully low. I’ll have to think about it,” and then left.
    The next day, the neighbor telephoned the contractor. The neighbor explained to the contractor that he realized the value of his home would be enhanced by the improvement done to the homeowner’s property. For that reason, the neighbor told the contractor he would pay him $10,000 if he would accept the homeowner’s proposal and do the work called for by the plans and specifications. The contractor agreed. The next day the contractor informed the homeowner that he would accept his $80,000 offer.
    The contractor did all the work called for by the plans and specifications, and collected the $80,000 due him from the homeowner. However, the neighbor refused to pay the contractor the $10,000.
    If the contractor brings a breach of contract action against the neighbor, judgment for whom?
    (A) The contractor, because his signing a contract with the homeowner and completing the work according to the plans and specifications was legally sufficient consideration to support the neighbor’s promise to pay him the $10,000.
    (B) The contractor, provided the neighbor realizes an increase in his property value as a result of the work done by the contractor.
    (C) The neighbor, because his promise to the contractor was illusory.
    (D) The neighbor, because at the moment the contractor accepted the homeowner’s offer, he came under a pre-existing obligation, which was insufficient consideration to support the neighbor’s promise.
A
  1. (A) This is a rather straightforward Contracts example dealing with consideration. The test of a legally enforceable promise at common law is whether it is supported by consideration. To constitute consideration, a performance or a return promise must be bargained for. In the present example, the agreement between the contractor and the neighbor was clearly supported by consideration. There was a bargainedfor exchange in that the contractor agreed to accept the homeowner’s offer and do the work according to the plans and specifications, and in return, the neighbor promised to pay him $10,000. Choice (B) is wrong, because there is sufficient consideration regardless of whether the neighbor realizes any increase in his property value. Choice (C) is incorrect, because the neighbor’s promise was not illusory; he was committing himself to pay $10,000 upon the completion of the work by the contractor. Choice (D) is incorrect, because the contractor accepted the neighbor’s offer before he contracted with the homeowner.
80
Q
  1. A university fired its head basketball coach after a series of losing seasons. The university contacted a coach who had a strong reputation as a person who cottld “turn around” a program and produce winning teams. The university offered the coach an annual salary of $250,000 to coach its basketball team, but the coach declined the offer, hoping to find another institution that would pay him a higher salary.
    Thereafter, a wealthy booster, who was interested in enhancing the university’s basketball program, contacted the coach and told him that if he would sign a coaching contract with the university, the booster would pay him $50,000 a year as an unpublished supplement to his salary. Furthermore, the booster promised to contribute $5,000 to the university athletic fund every time the basketball team won a game under the coach’s tutelage. The coach thereupon signed a three-year contract as the head basketball coach at the university at an annual salary of $250,000.
    During the coach’s first year as coach, the university basketball team compiled a record of 18 wins and 12 losses. The booster, however, refused to pay anything to either the coach or the university athletic fund. Assume that the agreement between the coach and the booster does not violate any rule of the university or of any intercollegiate athletic association.
    The university brought suit against the booster to recover $90,000 for breach of his promise to pay $5,000 into the athletic fund for each game won by the basketball team.
    Is it likely that the university will prevail in this contract action?
    (A) Yes, because the university is an intended third-party beneficiary of the contract between the coach and the booster.
    (B) Yes, because the university detrimentally relied on the booster’s promise by hiring the coach.
    (C) No, because the booster’s promise was, in essence, an illegal form of procurement and void as against public policy.
    (D) No, because the booster’s promise was made to the coach rather than to the university and, therefore, was not a charitable subscription.
A
  1. (A) In order to determine whether the university is an intended or an incidental beneficiary, the key is to ascertain the primary intent or purpose of the booster, the promisee, in exacting his promise from the coach. If the promisee’s main purpose is to confer a benefit upon the third party, then the latter is viewed as an intended beneficiary. On the other hand, where the promised performance is to be rendered to the promisee, then no inference of a benefit promise for the third-party’s benefit is possible no matter how much the third party may collaterally benefit. Here, the booster’s main purpose is to benefit the university by hiring the coach as its basketball coach and contributing to the university’s athletic fund. Thus, the university is viewed as an intended beneficiary. Choice (B) is wrong, because the university did nothing that would constitute detrimental reliance on the booster’s promise. They merely agreed to pay the coach $250,000 to coach their basketball team. Choice (C) is incorrect, because there is no reason why the booster’s promise should be considered illegal, especially since the facts state that the agreement between the coach and the booster does not violate any rule of the university or of any intercollegiate athletic association. Choice (D) is wrong, because the booster’s promise does not need to be a charitable subscription to be enforceable. The university can enforce it as an intended beneficiary, as discussed above.
81
Q
  1. A homeowner said to a roofer, “My roof leaks. I think the old tiles are cracked. If you will replace them with all new tiles, I will pay you $5,000.” The roofer replied, “Sure, if I can clear my busy schedule.” The homeowner then remarked, “That’s all right, but let me know soon.” Three days later, the roofer drove his pickup truck to the homeowner’s home and unloaded the materials and equipment needed to perform the roofing job. When the homeowner looked out his window and saw what was transpiring, he immediately ran outside and exclaimed, “Stop! The deal’s off. I decided to repair the roof myself.”
    In an action by the roofer against the homeowner for breach of contract, which of the following would provide the roofer with his best theory of recovery?
    (A) A bilateral contract was formed when the roofer purchased the materials and equipment needed to do the job.
    (B) A bilateral contract was formed when the roofer said, “Sure, if I can clear my busy schedule.”
    (C) The homeowner made an offer that proposed a unilateral contract, and the offer became irrevocable when the roofer purchased the materials and equipment needed for the job.
    (D) The homeowner made an offer that proposed a unilateral contract, and the roofer manifested an intent to accept the offer when he began performance by unloading the materials and equipment at the homeowner’s house.
A
  1. (D) A unilateral offer which invites performance of an act as acceptance, rather than a return promise, becomes irrevocable as soon as the offeree has started to perform the act. Choice (C) is wrong, because purchasing the materials is mere preparation and not the commencement of performance. Neither Choice (A) nor Choice (B) would be a good argument for the roofer. If this was interpreted as a bilateral contract, it would take an exchange of promises for the contract to be formed. Neither the purchasing of the material nor the statement by the roofer would be a sufficient promise to bind the parties to a bilateral contract.
82
Q
  1. An owner of a large, three-story office building planned to renovate the building and requested several contractors to submit bids to perform the work. One of the contractors in turn sought bids from several subcontractors to determine the overall cost of the job. An electrician submitted a bid to do the electrical work to the contractor, which the contractor used in computing the bid he sent to the owner. The contractor notified the electrician to that effect.
    Two weeks later, the contractor submitted his bid to the owner, in which he offered to perform the renovation work for $75,000. The next day, the electrician notified the contractor that he was unable to perform the electrical work on the owner’s project. Two days later, the owner sent the contractor a signed confirmation letter wherein she stated, “I hereby accept your offer to perform the renovation work for the quoted cost of $75,000.”
    Thereafter, the contractor hired another electrician to complete the electrical work at a cost of $5,000 above the first electrician’s bid. The contractor explained the situation to the owner and informed her that the overall cost of the job would have to be increased to $80,000. The owner responded that she would hold the contractor to his original bid of $75,000 and would not be responsible for any additional costs. The contractor then performed the renovation work, but the owner has not yet paid him anything.
    In an action by the contractor against the owner for the services rendered, the contractor will probably be able to recover
    (A) only in quantum meruit, because of the doctrifle of commercial frustration.
    (B) only in quantum meruit, because by demanding $80,000 the contractor, in effect, repudiated his contract with the owner.
    (C) $75,000 only, because that was the contract price.
    (D) $80,000, because the contractor reasonably relied to his detriment on the electrician’s bid in formulating his job estimate.
A
  1. (C) Another important Multistate testing area deals with damages (both in Contracts and Torts law). Choices (A) and (B) are incorrect, because quantum meruit (i.e, reasonable value for services rendered) is a restitutionary form of relief. In this particular situation, the plaintiff’s proper form of relief is damages for breach of contract, not restitution. Where the plaintiff has fully performed his side of the contract, and the defendant’s only duty is the payment of the agreed price in money, the remedy of restitution of the value of his performance is not available to the plaintiff. His proper remedy is damages for breach. Choice (D) is incorrect, because the owner is not obligated to pay any more than the contract price of $75,000. The contractor’s reliance on the electrician’s bid may allow him to recover from the electrician, but not from the owner. Therefore, Choice (C) is the best answer.
83
Q
  1. A developer recently acquired a large vacant lot and wished to build a series of 10 tract homes on the lot to offer for sale. On May 1, he telephoned a number of contractors whom he had dealt with in the past and asked them to submit bids to build the homes. One of the contractors in turn sought bids from several subcontractors to determine the overall cost of the job. He notified each subcontractor that he intended to use their bids in submitting a general bid to the developer. A plumber submitted a bid to the contractor in the amount of $50,000, which the contractor used in computing the bid he sent to the developer.
    On June 1, the contractor submitted a bid to the developer in which he offered to build the 10 tract homes for $500,000. On June 4, the plumber notified the contractor that he had overbooked his crews and could not possibly do the work on the developer’s job.
    On June 5, the developer sent the contractor a signed confirmation letter wherein he stated, “I hereby accept your offer to build the 10 tract homes for $500,000.’, Thereafter, the contractor hired another plumber to complete the plumbing work at a cost of $60,000. The contractor then explained the situation to the developer and requested that the overall cost of the job be increased to $510,000. The developer refused on the grounds that he had a contract for $500,000 and that this was the contractor’s problem. The contractor then built the 10 tract homes, and the developer sent him a check for $500,000.
    Which of the following best supports a claim for $10,000 by the contractor against the plumber who refused to do the work?
    (A) The plumber made an offer that the contractor accepted by using the plumber’s bid in computing the bid he submitted to the developer.
    (B) The plumber’s bid was a “firm offer” that was not revocable, since both the plumber and the contractor were merchants.
    (C) The contractor made an offer to the plumber that the plumber accepted when he submitted his bid.
    (D) An option contract was created, because the contractor, with the plumber’s knowledge, used the plumber’s bid in computing the bid he submitted to the developer.
A
  1. (D) When a general contractor, about to submit a bid on a construction project, secures a bid from a subcontractor for a definite part of the proposed work and uses the bid to determine that part of his cost, he often finds after the principal contract is let to him that the subcontractor refuses to go through with the job. He must then find another to do the job, usually at a price much higher than the promised figure. Can he recoup his loss from the defaulting subcontractor? Yes, the subcontractor is bound under the doctrine of promissory estoppeL Thus, a promisor who induces substantial change of position by the promisee in reliance on the promise is estopped to deny its enforceability as lacking consideration. The reason forthe doctrine is to avoid an unjust result. Choice (D) is therefore correct, because Restatement of Contracts, 2d, Section 87(2), provides: “An offer which the offeror should reasonably expect to induce action or forbearance of a substantial character on the part of the offeree before acceptance and which does induce such action or forbearance is binding as an option contract to the extent necessary to avoid injustice.” Choice (A) is wrong, because the contractor’s use of the bid was not an acceptance of the offer; it was justifiable reliance. Such sub- contra ctoffers are not acce pted until the general contractor wins the bid. Choice (B) is incorrect because the firm offer rule applies to a sale of goods contract only. Choice (C) is wrong, because the contractor did not make an offer to the plumber; he simply asked for a submission of bids.
84
Q
  1. Over the years, a coin collector had amassed a large and valuable collection. On May 1, the collector sent the following letter to a retailer who had long coveted the collector’s coin collection:
    “I’ve decided to part with my coin collection. Since you’ve always been interested in purchasing my collection for your store, I’ll sell you my entire collection for $50,000. You have until May 15 to make up your mind.”
    The retailer received this letter on May 3. On May 10, without notif’ing the retailer, the collector sold her coin collection to a novice collector, for $60,000. The next day, the collector sent a letter to the retailer, regretfully informing him that she had sold the coin collection. This letter was received by the retailer on May 13. However, on May 12, the retailer read an article on the internet, published by a reputable site that reports coin collecting news, concerning the collector’s sale of her coin collection to the novice collector. After reading the article, the retailer immediately sent an e-mail to the collector accepting her offer to buy the coin collection. This e-mail was read by the collector that same afternoon, May 12.
    In all likelihood was the retailer’s e-mail to the collector on May 12 effective as an acceptance?
    (A) Yes, because the collector’s May 11 letter did not effectuate a revocation, since it was not received by the retailer until May 13.
    (B) Yes, because consideration is not necessary under the UCC for the creation of a sale of goods contract.
    (C) No, because the collector’s sale of the coin collection on May 10 terminated the retailer’s power of acceptance.
    (D) No, because the retailer’s reading the article on the internet on May 12 terminated the retailer’s power of acceptance.
A
  1. (D) This question deals with revocation of an offer. As a general rule, the offeror may at any time before acceptance terminate her offer by revoking it. This is true even though the offeror has promised not to revoke for a stated time, unless the promise is supported by consideration. Revocation takes effect only when communicated to the offeree. This is true even when the offeree learns through a third party that the offeror is no longer willing to enter into the proposed contract. By the same token, Restatement of Contracts 2d, Section 43, states that “an offeree’s power of acceptance is terminated when the offeror takes definite action inconsistent with an intention to enter into the proposed contract and the offeree acquires reliable information to that effect.” Choice (D) is correct, because the revocation took effect when the retailer learned of the collector’s sale of the coin collection in the article on the internet. Choice (A) is wrong, because the offer was therefore terminated the day before the receipt of the collector’s May 11 letter. Choice (B) is an incorrect statement of the law. Consideration is certainly required to form a sale of goods contract; consideration is not necessary to modify such a contract. Choice (C) is incorrect, because it is not the sale of the coin collection that terminates the offer; it is the retailer’s learning of the sale from a reliable source that is the indirect revocation.
85
Q
  1. An avid stamp collector had amassed a large and valuable collection. Many of his stamps were extremely rare and highly coveted. On numerous occasions the collector had rejected offers to sell his prized collection. Finally, on December 1, the collector sent his cousin, who was also a stamp collector, the following letter:
    “I’ve decided to part with my stamp collection. Since I want to keep it in the family, I’ll sell you my entire collection for $75,000.”
    The collector had dictated this letter to his secretary, who mistakenly typed “$75,000,” instead of $78,000 that the collector had specified. After typing the letter, the secretary gave it to the collector, who hastily signed it without noticing the mistaken price. The cousin received this letter on December 3. On December 5, the cousin sent a letter accepting the collector’s offer.
    Which of the following correctly states the agreement’s price term and its legal effect?
    (A) The price term is $75,000, and it is enforceable.
    (B) The price term is $78,000, and it is enforceable.
    (C) The price term is $75,000, but the court will reform the price to $78,000 in order to correct a mistake in integration.
    (D) The price term is $75,000, but either party can rescind the contract, because there was a mutual mistake as to a basic assumption of fact.
A
  1. (A) As a general rule, where only one of the parties is mistaken about facts relating to the agreement, the mistake will not prevent formation of a contract. If the other party forms reasonable expectations based upon the apparent terms of the bargain, the contract is generally enforceable against the mistaken party on those terms. Relief is only granted is a situation where the non-mistaken party is or should have been aware of the mistake. That is not the situation here, because the cousin would have no reason to know of the mistake in the offer. Therefore, the price term stated in the offer would control. Choice (B) is therefore incorrect. Choice (C) is wrong. Reformation is used to amend a writing so that it reflects the true agreement of the parties. The true agreement of the parties here was to sell the stamp collection for $75,000, since the collector cannot get relief for his unilateral mistake. Therefore, reformation would not be allowed. Had the parties agreed to a sale price of $78,000 and then, when that agreement was reduced to writing, the secretary mistakenly typed “$75,000” into the writing, reformation of the writing would be allowed. Choice (D) is wrong, because this was not a mutual mistake. The only mistaken party was the collector.
86
Q
  1. An avid baseball fan learned that a local professional baseball club was conducting a baseball camp for fans who wanted to meet and receive instruction from the club’s players. The cost of the two-week camp was advertised for $2,500, which included meals and housing. The fan, a 54-year-old salesman, forwarded the club a $50 registration deposit, which reserved him a spot in the baseball camp.
    Thereafter, the fan received a contract from the club, which all baseball camp attendees were required to sign. The agreement provided that the $2,500 entrance fee was nonrefundable. According to the agreement, all attendees would receive group instruction during the baseball camp. Consequently, the club’s operating costs would not be reduced if one or more of the attendees failed to participate or complete the two-week program. The fan signed the contract and forwarded it with his $2,500 entrance fee to the club. Two days before the start of the baseball camp, however, the fan died from a heart attack.
    In a restitutionary action, can the executor of the fan’s estate, a surviving brother, recover on behalf of the estate either all or part of the $2,500 paid to the club?
    (A) No, but only if the club can show that before the start of the baseball camp it rejected another applicant because of its commitment to the fan.
    (B) No, because under the terms of the agreement the $2,500 entrance fee was nonrefundable.
    (C) Yes, because the club would otherwise be unjustly enriched at the fan’s expense.
    (D) Yes, under the doctrine of frustration of purpose.
A
  1. (B) In the interpretation of a contract, where the intention (of the parties) clearly appears from the words used, there is no need to go further. In such cases, the words (or language in the contract) must govern, or, as it is sometimes said, where there is no doubt, there is no need for interpretation. Here, the contract clearly stated that the entrance fee was non-refundable. Since the risk of loss falls on the fan, Choice (B) is correct. Choice (A) is wrong, because there would be no need for the club to show reliance to recover. A contract was formed between the parties, which clearly stated that it was non-refundable. Choice (C) is wrong, because there would be no unjust enrichment. As stated in the facts, the club’s operating costs were fixed and would not be reduced by a failure of a participant to attend. Choice (D) is incorrect, because, as stated above, the fan is charged with assuming the risk, since the contract was non-refundable. When a party assumes the risk of something happening, that party cannot claim discharge on the happening of that event.
87
Q
  1. On January 31, a supermarket entered into a written contract with a produce company to purchase oranges. The contract contained a provision wherein the supermarket promised to purchase “as many oranges as required, but at a minimum 100 bushels per month at a price of $20 per bushel for the next year.” The agreement also provided that any modifications must be in writing.
    On February 1, the produce company shipped the supermarket 70 bushels of oranges, which were accepted and paid for. On March 1, the produce company tendered 80 bushels of oranges, which the supermarket accepted and paid for. On April 1, the produce company delivered 100 bushels of oranges to the supermarket. This shipment was accepted and also was promptly paid for.
    On April 2, the manager of the supermarket became concerned because a Florida drought had resulted in a sharp increase in the price of oranges. Consequently, the manager consulted the supermarket’s attorney, who advised him to demand adequate assurances that the produce company would perform its obligations under the terms of the contract. Heeding the attorney’s advice, the manager sent a letter on April 3 to the produce company, expressing his concern and requesting an adequate assurance of due performance for the balance of the contract. This letter was received by the produce company on April 4.
    Which of the following best states the supermarket’s legal rights against the produce company?
    (A) Cancel the contract on May 1, cover immediately and then sue for damages.
    (B) Wait until the May 1 shipment; if it doesn’t comply, demand adequate assurances, and not obtaining them, then sue for damages.
    (C) Wait until the end of the contract and then sue for damages.
    (D) Wait a reasonable time not to exceed 30 days; if adequate assurances are not received, cancel the contract, cover immediately and then sue for damages.
A
  1. (D) UCC Section 2-609 authorizes one party upon “reasonable grounds for insecurity” to “demand adequate assurance of due performance and until he receives such assurance… if commercially reasonable suspend any performance for which he has not already received the agreed return.” Subsection (4) provides that a prospective repudiator’s “failure to provide within a reasonable time not exceeding thirty days such assurance of due performance” constitutes a “repudiation of the contract.” In which case, the buyer may cancel the contract, “cover,” and recover damages as provided in Sections 2-711 and 2-713. Choice (A) is incorrect, because it is not clear whether the produce company will make the May I shipment. If it does, there would be no grounds to cancel the contract. The supermarket cannot use the prior shortages as grounds to terminate the contract, because they accepted those shipments without objection. Choice (B) is wrong, because the supermarket has already made a demand for adequate assurances, and the previous shortages coupled with the sharp increase in the price for oranges gave them reasonable grounds to make such a demand on April 3. Choice (C) is wrong, because the supermarket would not have to wait until the end of the contract to sue. If they do not receive the adequate assurances within a reasonable time, not to exceed 30 days, they can immediately file suit.
88
Q
  1. On December 30, a restaurant entered into a written contract with a bakery to supply the restaurant with all of its bread needs for the next calendar year. The contract contained a provision wherein the restaurant promised to purchase “a minimum of 100 loaves per month at $1 per loaf.” On a separate sheet, there was a note stating that any modifications must be in writing. The parties signed each sheet.
    Both sides performed fully under the contract for the first four months. On May 1, the president of the bakery telephoned the manager of the restaurant and told him that, because of an increase in the cost of wheat, the bakery would be forced to raise its prices to $1.20 per loaf. The manager said he understood and agreed to the price increase. The bakery then shipped 100 loaves (the amount ordered by the restaurant) to the restaurant, along with a bill for $120. The restaurant sent the bakery a check for
    $100 and refused to pay any more.
    Is the restaurant obligated to pay the additional $20?
    (A) Yes, because the May 1 modification was enforceable even though it was not supported by new consideration.
    (B) Yes, because the bakery detrimentally relied on the modification by making the May shipment to the restaurant.
    (C) No, because there was no consideration to support the modification.
    (D) No, because the modifying contract was not in writing; it was, therefore, unenforceable under the UCC.
A
  1. (D) With respect to modifications under the UCC, be advised that the parties can provide in their original agreement that its terms may be modified only in writing. UCC Section 2-209 (2) specifically validates such a clause, “A signed agreement which excludes modification or rescission except by a signed writing cannot be otherwise modified or rescinded, but except as between merchants such a requirement on a form supplied by the merchant must be separately signed by the other party.” Therefore, since the contract expressly provided that “any modifications must be in writing,” choice (D) is correct in accordance with Section 2-209(2). Choice (A) is not the best answer. Even though consideration is not required under the UCC, the modification would not be enforceable, because it was not in writing as required by the contractual provision. Choice (B) is incorrect, because the bakery did not detrimentally rely on the oral agreement. The bakery merely shipped what the contract required them to ship; this is not detrimental reliance. Choice (C) is wrong, because, as stated above, consideration is not required for the modification of a sale of goods contract.
89
Q
  1. On October 15, an aspiring young actress entered into a written contract with a nationally famous photographer. Their contractual agreement provided that the photographer was to supply the actress with twelve glossy prints “designed to capture and convey the actress as a gifted dramatic actress with varied talents.” Their contract further stipulated “said twelve prints to be delivered on or before November 15; with payment of $1,500 thirty days thereafter.” Another contractual provision recited that the photographer “guarantees that the prints will be fully satisfactory and delivered on time.”
    The following day, the photographer accidentally fell down a flight of stairs, and sustained a broken leg. As a result of his injury, the photographer was unable to photograph the actress until October 31. At that time he informed the actress that because of the delay, “I am going to need a few more days beyond November 15 to make delivery.” The actress responded, “Please hurry with the pictures, because I need to submit my promotional portfolio for the leading role in an upcoming movie.”
    The photographer diligently worked to speed production and delivered the prints to the actress’s manager on November 18. After the manager looked over the photographs, she told the photographer, “I’m sure these will be acceptable to my client.” However, the following day the photographer received a telephone call from the actress, who said, “These prints are just awful. They make me look horrible. I’m sending them back to you and not paying anything. Besides, these prints were two days late.”
    If the actress initiates an action against the photographer for breach of contract, which of the following would be the photographer’s best defense?
    (A) The photographs, if subjectively viewed, were not satisfactory.
    (B) The photographs, if objectively viewed, were not satisfactory.
    (C) The photographs were not delivered on time, thereby resulting in a failure of condition precedent to the actress’s liability.
    (D) The photographer’s injury constituted a temporary impracticability of performance, which excused his duty to perform for a reasonable
    period of time.
A
  1. (D) The key to this question was a careful reading of the interrogatory. You were asked to identify the photographer’s best defense. Therefore, you should have quickly eliminated Choices (A) and (B), since they speakto reasons why the plaintiff should prevail. If the photographs were not satisfactory, that would be a good argument as to why the actress should prevail, not a defense for the photographer. Choice (C) is not a good answer; unless excused, the failure to deliver on time would constitute a breach by the photographer, not a defense to liability. The photographer’s best defense would be temporary impracticability of performance. It is important to note that temporary impracticability only suspends contractual duties; it does not discharge these duties. When performance once again becomes possible, the duty of performance “springs” back into existence. In personal service contracts, the courts will frequently excuse performance temporarily because of health reasons of the promisor. Therefore, since the photographer broke his leg and was unable to photograph the actress until October 31, his performance would be temporarily suspended by this unexpected accident.
90
Q
  1. A homeowner entered into a written contract with a house painter to have his house painted for $10,000. The contract stated in part: “It is agreed that the house painter will paint the house using off-white latex paint, and will finish the job within 30 days from the execution of this agreement. If the homeowner finds the completed job to be satisfactory, he will pay the house painter $10,000 within one week after the job is completed.”
    Which of the following is the most accurate statement regarding the homeowner’s contractual obligation to pay the house painter $10,000?
    (A) Payment of the $10,000 by the homeowner would be an express condition precedent to the house painter’s duty of performance.
    (B) Payment of the $10,000 by the homeowner would be an express condition subsequent to the house painter’s duty of performance.
    (C) The house painter’s performance under the contract would be an express condition precedent to the homeowner’s duty of payment of the $10,000.
    (D) The performances of the homeowner and the house painter were, in essence, concurrent conditions.
A
  1. (C) The house painter’s performance under the contract would be an express condi
    tion precedent to the homeowner’s duty to pay the house painter the $10,000. An
    express condition precedent is: 1) a fact expressed in a contract; 2) which must
    exist or occur before a duty of immediate performance by the promisor can arise. In
    effect, it qualifies a promisor’s present duty of performance in a contract. Since the
    house painter must fully perform, and that performance must be acceptable to the
    homeowner before any payment is due, an express condition precedent was cre
    ated. Choice (A) is therefore incorrect. Choice (B) is also incorrect. A condition sub
    sequent is an operative fact that will extinguish a duty to make compensation for
    breach of contract after the event has occurred. That was not the case here. Please
    note that examples of true conditions subsequent are rare in contract law. Lastly,
    Choice (D) is incorrect, since concurrent conditions are those that are capable of occurring together and that the parties are required to perform simultaneously.
91
Q
  1. After a severe thunderstorm had struck the area, a homeowner discovered that her roof had suffered extensive damage. She telephoned a roofer to inquire about having the necessary repairs done to her roof. The roofer agreed to finish the work within 30 days, and the homeowner promised to pay
    $10,000.
    Which of the following statements is most accurate?
    (A) The performance of the roofer would be a constructive condition precedent to the performance by the homeowner.
    (B) The performance by the homeowner would be an implied condition subsequent to the performance by the roofer.
    (C) The performances of the homeowner and the roofer would be constructive concurrent conditions.
    (D) The contract would be unenforceable, because it was oral.
A
  1. (A) The most accurate statement is that the performance of the roofer would be a constructive condition precedent to the performance by the homeowner. Constructive conditions are “implied in law” conditions that are read into a contract by the courts to insure that the parties to the contract receive the performance that they have bargained for. Where the contract contains no provision relating to the order of the respective performances, and performance on one side will necessarily take time, while the other can be performed in an instant, the former is a constructive condition precedent to the latter. In other words, the performance that takes time must first be substantially completed before the other party need render his performance. Since it will take the roofer a period of time to complete the repairs, while the homeowner’s act of paying takes no time to perform, (A) is the most accurate statement. Choices (B) and (C) are therefore wrong. Choice (D) is incorrect, because there is no reason why the contract would need to be in writing. It does not fall under any provision of the Statute of Frauds.
92
Q
  1. A man entered into a written contract with a nationally famous artist. Their contract provided that the artist would paint a full-color portrait of the man, to be delivered by November 1, in exchange for $15,000. A contractual provision recited that the artist “guarantees that the quality of the portrait will be fully satisfactory.”
    The artist finished the portrait and presented it to the man on November 2. The man looked at the portrait and said, “This is awful! I really expected better. I’m not paying for this!” The artist was enraged by the man’s reaction, as he felt the finished portrait was one of his best works.
    Which of the following arguments would be LEAST helpful to the artist in his efforts to recover on a breach of contract theory?
    (A) The man’s dissatisfaction with the quality of the portrait was not genuine.
    (B) The man failed to make an adequate examination of the quality of the portrait.
    (C) The man’s dissatisfaction with the portrait was the result of delivery after November 1, rather than of any lack of quality in the portrait.
    (D) Because of a recent hand injury, the artist did not have sufficient time to produce a portrait of a quality satisfactory to the man.
A
  1. (D) Choice (D) would be the LEAST helpful argument in the artist’s breach of contract action. The artist’s injury would not excuse his obligation to produce a quality portrait that was satisfactory to the man. The condition of the man’s satisfaction with the artist’s performance is subjective in our question. Please note that where the promisee’s performance must satisfy the promisor personally and in a subjective sense, and the promisor in “good faith” is dissatisfied with the performance, his (the promisor’s) liability is discharged. Generally, in contracts involving aesthetic taste, art, or personal judgment, the only performance that will satisfy the condition of satisfaction is that the promisor be satisfied with the result. Choice (A) would therefore be a good argument, because if the man was not genuinely dissatisfied, the condition would be excused. Choice (B) would be a good argument, because if the man failed to make an adequate examination, it can be argued that his dissatisfaction is not genuine. Choice (C) would be a good argument, because the condition of satisfaction should be about the quality of the portrait, not the timing of its delivery. Since Choices (A), (B), and (C) are all good arguments, they would all be helpful to the artist.
93
Q
  1. On August 1, a mother hired a photographer to take a series of photographs of her two-year-old daughter. The written contract called for the photographer “to produce a minimum of six acceptable photographs by August 15.” In exchange, the photographer was to be paid $200.
    Which of the following is the LEAST accurate statement with regard to the photographer’s taking of the photographs?
    (A) By entering into the contract as written, the photographer assumed the risk that the mother would fail to cooperate in arranging for the taking of the photographs.
    (B) The mother was under an implied duty to act in good faith to reasonably cooperate with the photographer in arranging for the taking of the photographs.
    (C) An implied condition of the photographer’s duty of performance was that the mother reasonably cooperate in arranging for the taking of the photographs.
    (D) The mother’s refusal to cooperate reasonably in arranging for the taking of the photographs would excuse the photographer from further obligation under the contract and also give the photographer a right of action against the mother for breach of contract.
A
  1. (A) Choice (A) is the LEAST accurate statement. Under the terms of this contract, the photographer would not have to assume the risk that the mother would fail to cooperate in arranging for the taking of the photographs. The mother’s failure to cooperate with the photogra pher would be indicative of “bad faith” on her part. Furthermore, an implied condition exists in a personal services contract that requires both parties to act reasonably in order to carry out their mutual obligations under the terms of the contract. Choices (B) and (C) would therefore be accurate statements. Choice (D) is also accurate. The mother’s failure to cooperate would be a breach of her implied duty to cooperate, entitling the photographer to treat the contract at an end and sue for breach.
94
Q
  1. A developer acquired a large parcel that he intended to convert into a residential subdivision. The developer met with the man who owned the adjoining property and informed him of the proposed development plans. During their discussion, the man expressed concern about the increased traffic and noise from the subdivision. Thus, the parties orally agreed that the developer would contract with someone to erect a brick wall between their respective properties, and that the man would reimburse the developer for half the expenses the developer incurred in having the wall erected. Thereafter, the developer hired a mason to erect the wall for $10,000. The developer explained to the mason that the wall was being built between his property and the man’s. He further advised the mason that although he (the developer) would be paying the mason the $10,000, the man had agreed to reimburse the developer for half the price.
    The mason built the wall as agreed. However, the developer refused to pay the mason anything, and the mason, in turn, did not sue the developer for the agreed price.
    The mason has now brought an action against the man to recover $5,000. Which of the following is the man’s best defense?
    (A) By suing the man for half the contract price, the mason is wrongfully splitting his cause of action.
    (B) The mason is not an intended beneficiary of the agreement between the developer and the man.
    (C) The agreement between the developer and the mason was not in writing.
    (D) The agreement between the developer and the man was not in writing.
A
  1. (B) Clearly, the man’s strongest defense is to argue that the mason was not an intended beneficiary of the agreement between the developer and the man. As such, the mason would be viewed as an incidental beneficiary, and he would acquire no right to enforce the contract between the developer and the man. The only third persons given rights under a contract to which they are not parties are the donees and the creditors of the promisee. Other third persons might be benefited if the promised performance occurs, and correspondingly disappointed if it does not, but they are given no right to enforce the promise. Such persons are called incidental beneficiaries. Choice (A) is wrong, because there is no “joint” obligation between the developer and the man. Here, the mason’s only cause of action is against the developer, based on their agreement. Since the parties contracted separately, no joint liability attached to the man. Choice (C) is incorrect, because it is irreLevant whether the agreement between the deveLoper and the mason is in writing inasmuch as the mason is suing the man. As such, the mason is basing his cause of action as a beneficiary under the agreement between the developer and the man. Finally, Choice (D) is not the best answer, because the agreement between the developer and the man does not come within the Statute of Frauds; the man is not promising to answer for the debt or default of another.
95
Q
  1. A woman and her neighbor had often talked about the need for a fence between their properties. Finally, the woman hired a handyman to build a wooden stake fence between her property and her neighbor’s property for $10,000. The handyman built the fence as agreed, and the woman paid him the sum of $10,000. Two weeks later, the neighbor was operating his lawn mower near the property line between his and the woman’s property when a section of the fence suddenly fell on top of him, seriously injuring him. A subsequent investigation revealed that the handyman did not properly reinforce the fence.
    Which of the following is the handyman’s best defense in a contract action brought against him by the neighbor, in which the only damages alleged are those for the neighbor’s personal injuries?
    (A) Damages for personal injuries cannot be recovered in a contract action.
    (B) Damages for personal injuries to the neighbor were not within the contemplation of the woman and the handyman at the time they entered into their agreement.
    (C) The neighbor is only an incidental beneficiary of the agreement between the woman and the handyman.
    (D) The neighbor has no standing to assert such an action, since he was not a party to the agreement between the woman and the handyman.
A
  1. (B) According to Restatement of Contracts 2d, Section 351, “Damages are not recoverable for loss that the party in breach did not have reason to foresee as a probable result of the breach when the contract was made.” A contracting party is generaLly expected to take account of those risks that are foreseeable at the time he makes the contract. He is not, however, liable in the event of breach for loss that he did not at the time of contracting have reason to foresee as a probable result of such a breach. Based upon this rule, damages for the neighbor’s personal injuries were not foreseeable or within the contemplation of the woman and the handyman at the time they entered into their contract. Choice (B) is therefore correct. Choice (A) is wrong, because damages for personal injuries can be recovered in a contract action if such a loss was foreseeable. In fact, in certain situations where breach of a contract is likely to result in serious emotional disturbance, recovery for such disturbance is even allowed. Common examples are contracts of carriers and innkeepers with passengers and guests. Choices (C) and CD) are incorrect. The neighbor is suing to recover damages for personal injuries, and not to enforce any rights under the contract As a result, the neighbor would have standing to sue regardless of whether or not neighbor is deemed to be an incidental beneficiary.
96
Q
  1. A bank loaned a company $1,500,000 for the manufacture of a widget control system. As a condition of the loan, a majority shareholder in the company agreed in writing to personally guarantee the loan. Thereafter, the company defaulted on the loan and entered into a repayment agreement with the bank. This agreement provided that the bank would “use maximum efforts in selling the company’s assets at the highest possible price.” The bank proceeded to sell the company’s assets, discharging the indebtedness of the widget company.
    Later, it was ascertained that the bank did not realize the “highest possible price” in administering the sale of the company’s assets. Consequently, the shareholder was forced into bankruptcy and lost her entire investment in the company.
    The shareholder brings an appropriate action against the bank to recover her investment in the company. She will most likely
    (A) not prevail, because the shareholder was an incidental beneficiary of the bank’s sale of the company’s assets.
    (B) not prevail, because the bank’s sale of the company’s assets discharged whatever contractual relationship existed between the shareholder and the bank.
    (C) prevail, because the bank did not realize the highest possible price from the sale of the cornpany’s assets.
    (D) prevail, because the bank breached its fiduciary duty to the shareholder under the terms of the contract between the bank and the company.
A
  1. (A) Here, the shareholder is the guarantor of the Loan between the bank (the promisor) and the company (the promisee). As a guarantor, she is not viewed as an intended beneficiary of the promisee, and the only third parties given rights under a contract to which they are not parties are the intended beneficiaries. Other third persons might be benefited if the promised performance occurs. and correspondingly disappointed if it does not, but they are given no right to enforce the promise. Such persons are called incidental beneficiaries. An incidental beneficiary acquires no right to enforce the promise. Choice (B) is wrong, because the contractual relationship of the shareholder as incidental beneficiary would still exist following the sale; the shareholder wouldiust not have rights upon whkh it could sue. Choice (C) Is incorrect, because the shareholder, as an incidental beneficiary, would have no standing to challenge the bank’s actions in selling the company’s assets. Choice (D) is incorrect, because no such fiduciary duty existed between the bank and the shareholder, as explained above.
97
Q
  1. A candidate for governor hired a production company to shoot a series of commercials that the candidate planned to use in his campaign. The written contract entered included a “production fee clause” that provided that the production company would be “paid $100,000 for the filming and editing of ten 30-second commercials” that would be suitable for the candidate’s television campaign broadcasts. The production fee clause also stipulated that the $100,000 would be paid to the production company “on condition that the filming and editing be directed under the personal supervision of the president of the production company.” The contract made no other reference to compensation.
    Thereafter, the production company filmed and edited the ten campaign commercials, which the candidate approved. When the production was completed, the production company submitted to the candidate an invoice statement in the amount of $150,000. Besides the $100,000 contract figure, the bill included a $50,000 charge for the president’s full-time services in directing the filming and editing of the videos. Denying any additional liability, the candidate sent the production company a check for $100,000. The production company then brought suit against the candidate to recover the $50,000 to cover the president’s services.
    Which of the following arguments would be most persuasive in the candidate’s efforts to prevent the introduction of parol evidence to show that prior to the parties’ execution of the written contract they had orally agreed on payment by the candidate to cover the president’s salary in addition to the $100,000 production fee?
    (A) There was no latent ambiguity contained within the actual written contract.
    (B) The written “production fee clause” is clear on its face, and no patent ambiguity is present in the writing.
    (C) Parol evidence of a prior oral agreement is barred if it contradicts a term of a written contract.
    (D) Since the agreement contained a compensation clause that specified a stipulated amount, the contract was fully integrated on that subject.
A
  1. (D) The written agreement between the candidate and the production company to shoot the campaign commercials was a personal services contract. Common law principles as to parol evidence will therefore apply. The parol evidence rule is a rule of substantive contract Law that bars evidence of any prior oral or written agreements or contemporaneous oral agreements to alter, vary, or contradict the terms of a writing, once the parties have expressed in writing their full and final agreement. Restatement of Contracts, 2d, Section 213, states that “(1) A binding integrated agreement discharges prior agreements to the extent that it is inconsistent with them. (2) A binding completely integrated agreement discharges prior agreements to the extent that they are within its scope.” The key issue in a parol evidence problem is to determine whether the agreement is completely or partially integrated. Where there is complete integration (i.e., a full and final expression of the parties such that the writing appears complete on its face) no extrinsic evidence is permitted. The candidate would argue that since the contract made no other reference to compensation other than the “production fee clause,” it was completely integrated and no parol evidence will be admitted to vary or contradict the express terms. Choice (D) is therefore the candidate’s strongest argument. Note: Determination of whether an agreement is fully or partially integrated is a question for the trier of fact. Choice (C) is incomplete. Parol evidence that contradicts the terms of a writing is barred only when the contract is fully integrated. Choices (A) and (B) deal with exceptions to the parol evidence rule. Extrinsic evidence is admissible to interpret and give meaning to the terms of a contract where there is ambiguity, but not where the parol evidence varies or contradicts the terms. Since evidence of the $50,000 fee to the president would vary the compensation stated in the “production fee clause,” any argument as to ambiguity would be irrelevant.
98
Q
  1. A man hired a videographer to film his daughter’s wedding. The written contract entered included a “payment clause,” which provided that the videographer would be “paid $10,000 for the filming and editing of a 60-minute video of the wedding and the reception.” The man included in the contract a stipulation that the video would be filmed using high definition equipment. The contract made no other reference to compensation.
    Thereafter, the videographer filmed and edited the 60-minute video, using high definition equipment, and presented it to the man. The videographer then submitted to the man an invoice statement in the amount of $15,000. Besides the $10,000 contract figure, the bill included a $5,000 charge for the use of the high definition equipment. Denying any additional liability, the man sent the videographer a check for $10,000. The videographer then brought suit against the man to recover the additional $5,000.
    Which of the following arguments would be most persuasive to support the videographer’s contention that when the written contract was executed, the man agreed to pay the videographer $5,000 for use of the high definition equipment in addition to the $10,000 fee?
    (A) According to the customary trade practice of the video industry, a $10,000 fee for filming and editing means $10,000 in addition to a supplemental charge if high definition equipment is used.
    (B) An oral agreement to that effect, if provable, would only supplement, not contradict, the “payment clause” as written.
    (C) Under the UCC, extrinsic evidence, if available, of additional terms agreed to by the parties is admissible unless such terms “would certainly vary or contradict those contained in the document.”
    (D) Assuming arguendo that the written “payment clause” was fully integrated and neither patently nor latently ambiguous, equitable considerations require admission of extrinsic evidence, if available, of the parties’ intent, since the videographer would stand to lose $5,000 on the contract.
A
  1. (A) This question deals with trade usage as an element in contract interpretation. Usage may give particular meaning to an agreement, or may supplement or qualify it.
    Restatement of Contracts 2d, Section 220, provides, “An agreement is interpreted, in accordance with a relevant usage if each party knew or had reason to know of the usage and neither party knew or had reason to know that the meaning attached by the other was inconsistent with the usage.” Moreover, there is no requirement that an agreement be ambiguous before evidence of a usage of trade can be shown, nor is it required that the usage of trade be consistent with the meaning the agreement would have apart from the usage. Choice (B) is incorrect, because such an agreement would be a clear contradiction of the writing. It should be noted, however, that trade usage can contradict the written contract. Choice (C) is not the best answer, because the UCC is not applicable (this question does not deal with sale of goods). Moreover, Choice (D) is incorrect, because contract interpretation is a question of lawnot equity. According to Restatement Section 212(2), “A question of interpretation of an integrated agreement is to be determined by the trier of fact if it depends on the credibility of extrinsic evidence or on a choice among reasonable inferences to be drawn from extrinsic evidence. Otherwise a question of interpretation of an integrated agreement is to be determined as a question of law.”
99
Q
  1. Ann, Bea, and Carla were three friends who lived in the same neighborhood. While Ann was away on a business trip, someone broke into her garage and stole her golf clubs. The next week, Ann was planning to go on vacation and asked Bea if she could borrow her golf clubs. Bea agreed and loaned her golf clubs to Ann, who promised to return them after her vacation. When Ann returned home, she kept the golf clubs and continued to use them.
    A few weeks later, Bea was having dinner with Carla and learned that Carla owed Ann $4,000. Carla had just been laid off from her job and did not have the money to repay Ann. Bea told Carla that she would contact Ann and make arrangements to repay the loan on her behalf.
    Thereupon, Ann and Bea entered into a written agreement wherein Bea promised to pay Ann, at a rate of $400 a month, the matured $4,000 debt that Carla owed Ann. In the same written instrument, Ann promised to return Bea’s golf clubs, which she still had in her possession. Ann, however, made no written or oral. commitment to forbear to sue Carla to collect the $4,000 debt; and Bea made no oral or written request for any such forbearance.
    After this agreement between Ann and Bea was signed and executed, Ann promptly returned the golf clubs to Bea. For the next six months, Bea made and Ann accepted the $400 monthly payments as agreed. During that period, Ann, in fact, did forbear to take any legal action against Carla. However, Bea then repudiated her agreement with Ann, and 30 days later Ann filed a contract action against Bea.
    Assume that the applicable statute of limitations on Ann’s antecedent claim against Carla expired the day before Ann filed her contract action against Bea.
    Which of the following is the most persuasive argument that Bea is not liable to Ann under the terms of their written agreement?
    (A) Since Ann did not expressly promise to forbear to sue Carla to collect the antecedent $4,000 debt, Ann’s forbearance for six months could not constitute consideration for Bea’s promise.
    (B) Since the written agreement between Bea and Ann shows a gross imbalance between the values of the promises exchanged, the consideration for Bea’s promise was legally insufficient to support it.
    (C) Since Carla, when the agreement between Ann and Bea was made, had a pre-existing duty to repay the $4,000 debt to Ann, there was no consideration for Bea’s promise to Ann.
    (D) Since Ann had a pre-existing duty to return Bea’s golf clubs to her when the agreement between Ann and Bea was made, there was no consideration for Bea’s promise to Ann.
A
  1. (D) This difficult Contracts fact pattern deals with the area of consideration. Restatement of Contracts 2d, Section 71, states that “(1) To constitute consideration, a performance or a return promise must be bargained for. (2) A performance or return promise is bargained for if it is sought by the promisor in exchange for his promise and is given by the promisee in exchange for that promise. (3) The performance may consist of (a) an act other than a promise, or (b) a forbearance, or (c) the creation, modification, or destruction of a legal relation. (4)The performance or return promise may be given to the promisor or to some other person. It may be given by the promisee or by some other person.” In addition to their bargain eLement of consideration, legal detriment is also required such that the promisee must promise to do something he does not legally have to do (i.e., forbearance to sue), or refrain from doing something that he has a legal right to do. The key fact students must realize is that although Bea promised to pay Ann the $4,000 debt owed by Carla, Ann did not suffer any legal detriment by making a return promise in exchange. Ann’s forbearance to sue Carla would have provided adequate consideration, but instead Ann only promised to return Bea’s golf clubs. Ann was already under a pre-existing duty to do so. Therefore, Bea’s best defense in a contract action brought by Ann would be that since Ann had a pre-existing duty to return the golf clubs, there was no consideration for Bea’s promise to Ann. Choice (D) is therefore correct. Choice (A) is not the best answer. While Ann did not expressly promise to forebear from suing Carla, her actual forbearance may constitute detrimental reliance, which would substitute for the missing consideration and make the promise enforceable. Choice (B) is wrong, because as a general rule, courts will not examine the sufficiency of the exchange so long as there is a bargained-for exchange. Choice (C) is wrong, because a pre-existing duty owed by Carla would have no bearing on the promise made by Bea.
100
Q
  1. A son owed a creditor $5,000. The son’s father contacted the creditor and told him that he wanted to pay the son’s debt. The father signed a document that stated the father would pay the son’s debt at a rate of $500 a month for 10 months. The creditor made no written or oral commitment to forbear to sue the son to collect the $5,000 debt, and the father made no oral or written request for any such forbearance.
    For the next five months, the father made and the creditor accepted the $500 monthly payments as agreed. During that period, the creditor, in fact, did forbear to take any legal action against the son. However, the father then informed the creditor that he would make no further payments on the debt.
    Which of the following is the most persuasive argument that the father is liable to the creditor under the terms of their agreement?
    (A) The father’s promise and the creditor’s reliance thereon, if proved, gave rise to a valid claim by the creditor against the father based on the doctrine of promissory estoppel.
    (B) Because it was foreseeable that the father’s promise would induce the creditor to forbear taking any action against the son, such forbearance was, as a matter of law, a bargained-for consideration for the father’s promise.
    (C) The father’s five payments to the creditor totaling $2,500 manifested a serious intent on the father’s part to be contractually bound, and such manifestation is generally recognized as an effective substitute for consideration.
    (D) By assuming the antecedent debt obligation that the son owed to the creditor, the father became a surety whose promise to the creditor was enforceable, since it was in writing and supported by adequate consideration.
A
  1. (A) The creditor suffered no legal detriment, since he made no sufficient return promise in exchange for the father’s promise to pay the son’s $5,000 debt. Therefore, the agreement between the creditor and the father would fail for lack of consideration. Since this question asks for the creditor’s most persuasive argument, he must attempt to argue that a valid substitute for consideration existed. Such an alternative would be promissory estoppel. Restatement of Contracts 2 d, Section 90, states that “(i) A promise which the promisor should reasonably expect to induce action or forbearance on the part of the promisee or a third person and which does induce such action or forbearance is binding if injustice can be avoided only by enforcement of the promise.” The father’s promise and the creditor’s reliance, if proved, could provide a valid claim based on promissory estoppel. Therefore, choice (A) is correct. Choice (B) is incorrect, because it goes against the facts. The creditor never in fact promised to forbear from suing the son; therefore, his failure to sue for five months cannot be considered bargained-for consideration. Choice (C) makes no legal sense, because consideration must be supplied by a showing of detriment on the creditor’s part, not by a mere showing of the father’s intent to be bound—such an intent cannot be transferred. Choice (D) is also incorrect. Even if a surety arrangement had existed between the father, the creditor, and the son, it would be voidable due to lack of consideration to support the father’s promise.
101
Q
  1. A woman owned an extensive art collection that she displayed in a special room of her home. While the woman was away on a vacation, there was a burglary at her home, and her favorite painting was stolen. Although the painting was insured for $1,000,000 by an insurance company, it had a market value of over $1,500,000.
    When the woman returned from vacation, she met with a detective employed by the insurance company to investigate the theft. During their meeting, the woman told the detective that she would pay him an extra $50,000 if he recovered the paihting. For the next three weeks, the detective investigated the theft as part of his job responsibilities with the insurance company. Within the course of this investigation, the detective learned who was responsible for the burglary. As a consequence, the culprit was apprehended, and the painting was recovered and returned to the woman.
    The detective then requested the $50,000 that the woman had promised to pay him. After the woman refused to make the payment, the detective sued the woman for breach of contract.
    Who is most likely to prevail?
    (A) The woman, because her promise was gratuitous.
    (B) The woman, because the insurance company owed her a pre-existing duty to find the painting.
    (C) The detective, because he did the act necessary to constitute an acceptance of the woman’s offer.
    (D) The detective, because the market value of the painting exceeded its insured value, so there was sufficient consideration to support the woman’s promise.
A
  1. (B) The issue here is the pre-existing duty rule. It does not constitute LegaL detriment when a party performs an act that he is legally obligated to do. The detective was hired by the insurance company to investigate the theft. Since he had a pre-existing duty to perform this service, his promise to the woman was not supported by consideration and therefore unenforceable. Choice (A) is not the best answer. While it is true that the woman’s promise was gratuitous, Choice (B) explains why that was the case. Since Choice (B) is a more complete answer, it wins out over Choice (A). Choice (C) is incorrect, because it does not matter whether the detective had accepted the woman’s offer; there is no consideration to support her promise to pay. Choice (D) is wrong for the same reason. The increased market value cannot supply the missing consideration.
102
Q
  1. A man told his neighbor, “If you will mow my lawn, I will pay you $50.” The neighbor then purchased gasoline for the mower so he could mow the lawn. The neighbor wheeled his lawn mower to the man’s property and started the engine. At that moment, the man suddenly yelled, “I hereby revoke my offer.”
    If the neighbor thereafter mows the man’s lawn, will he recover the $50?
    (A) Yes, because there was an offer for a unilateral contract that became irrevocable prior to the man’s attempted revocation.
    (B) Yes, under the doctrine of quasi-contract.
    (C) No, because the man’s revocation was effective, since the neighbor had not completed performance.
    (D) No, because the neighbor had done nothing more than prepare to accept the offer prior to the man’s revocation.
A
  1. (A) An offer that invites performance of an act as acceptance rather than a return promise becomes irrevocable as soon as the offeree has started to perform the act. This rule is deemed essential to prevent hardship to the offeree, where his part performance does not benefit the offeror and so would give him no recovery in quasi- contract. Although some of the older decisions have applied the logical view that a unilateral offer may be revoked at any time prior to full completion of the act bargained for, by the majority rule today such an offer becomes irrevocable as soon as the offeree has started to perform the act requested. Choice (B) is incorrect, because quasi-contract (which is a contract implied at law) is not needed, since there was an enforceable contract between the parties. Choice (C) is wrong, because the fact that the neighbor had started performance made the offer irrevocable, as explained above. Choice (D) is wrong, because the neighbor had gone beyond mere preparation. Buying the gasoline was preparation, starting the lawn mower on the man’s property would constitute starting into the requested act.
103
Q
  1. On April 10, the owner of a golf course entered into an oral agreement with a gardener whereby the gardener promised to install all new sprinkler heads on the sprinkler system at the 18-hole golf course. In return, the owner promised to pay the gardener $2,400 upon completion of the job. Since a golf tournament was scheduled for the weekend of April 20—21, the gardener agreed to replace all the sprinkler heads no later than April 19. Before accepting the job, the gardener had inspected the golf course and determined that 240 sprinkler heads had to be replaced.
    By April 14, the gardener had installed 80 new sprinkler heads on the first six holes of the 18-hole golf course. That afternoon, however, the owner learned that the gardener had been adjudicated bankrupt on April 12, so he notified the gardener to discontinue the job. The next day, the owner hired a landscaper to complete the installation work at $8 per head. The landscaper installed the remaining 160 sprinkler heads and completed the work on April 19. Despite making reasonable efforts, the gardener was unable to find any gainful employment during the period. Also, the gardener’s application for unemployment compensation was rejected at the same time.
    Which of the following statements, if found to be true, would provide the owner with legally justifiable grounds for discharging the gardener?
    (A) The gardener had been adjudicated bankrupt onApril 12.
    (B) The gardener had only completed 33 percent of the installation work when he was discharged.
    (C) The contract between the owner and the gardener was not in writing.
    (D) Half of the sprinkler heads installed by the gardener were determined to be defective, and the gardener refused to replace the sprinkler heads.
A
  1. (D) The gardener’s refusal to replace the sprinkler heads would amount to a material breach. If a defect in the promisor’s performance substantially affects the reasonable expectations of the aggrieved promisee, the breach is “material.” Here, the owner’s reasonable expectations were to have a set of sprinklers that actually functioned properly. Because the gardener’s refusal to replace the defective sprinkler heads substantially affected what the owner expected to get from the contract, the owner can treat this as a material breach and a reason to discharge the gardener. Thus, Choice (D) is correct. Choice (A) is incorrect, because insolvency or bankruptcy of the promisee does not of itself constitute such prospective failure of consideration as will discharge the promisor. See Restatement of Contracts 2d, Section 252. Choice (B) is not a strong enough grounds for discharge. The gardener promised to complete the job by April 19. Even though he had only finished 33 percent of the job byApril 14, he would still have until April l9to complete the work. Choice (C) is not a valid reason for discharging the gardener; since the agreement was for the installation of sprinkler heads, it did not cover the sale of goods. Therefore, it did not need to be in writing.
104
Q
  1. The owner of a soon-to-open resort complex entered into an agreement with an electrician to install lighting fixtures on the walking paths located on the resort property. The contract stipulated that the work had to be completed by December 1 (the anticipated opening date for the resort), and that the electrician would be paid $5,000 upon completion of the job. Before accepting the job, the electrician had inspected the property and determined that 500 lighting fixtures would be needed to adequately light the pathways.
    By November 15, the electrician had installed 150 lighting fixtures. That evening, the owner inspected the property and discovered that the lighting fixtures installed by the electrician did not provide adequate lighting. The next day, the owner immediately discharged the electrician and hired a lighting specialist to complete the installation work at $20 per fixture. The lighting specialist installed the remaining 350 fixtures, completing the work on December 1. The electrician filed suit against the owner.
    If it is determined that the owner was legally justified in discharging the electrician, which of the following is the electrician’s proper measure of recovery?
    (A) $7,000.
    (B) $3,500.
    (C) Quantum meruit for the reasonable value of his services rendered in installing the 150 fixtures.
    (D) Nothing, because the electrician did not complete performance of the entire job.
A
  1. (C) This is a rather tricky question dealing with Remedies. The electrician was hired to perform the work at a contract price of $5,000. According to the terms of the agreement, the electrician would be paid upon completion of the job. As such, the contract does not appear to be divisible. Therefore, Choice (A) is wrong. Although the electrician did not complete performance, he would, nevertheless, be entitled to receive restitution for the part performance that he rendered. A recovery on quantum meruit usually applies in situations where the plaintiff has performed services for the defendant, but he cannot recover on the contract (either because there was not full performance or because the contract was unenforceable). Choice (B) is wrong, because the measurement of recovery would be based on the reasonable value of the work performed, not on the contract price. The contract price ($5,000 for 500 fixtures) would be a factor in determining the reasonable value, but it would not be conclusive. Choice (D) is incorrect, because the electrician is entitled to some compensation for the work performed to prevent an unjust enrichment to the owner.
105
Q
  1. On August 10, a retail stationery store sent the following purchase order to a wholesaler of office supply equipment: “Please ship immediately 24 pairs (two dozen) 3 1/2-inch, right-handed scissors at your current list price of $4 per pair.”
    The wholesaler received this purchase order on August 12. The next day, the wholesaler ascertained that there were only 18 pairs of 3 1/2-inch, right- handed scissors in stock. The wholesaler, however, found that he had six pairs of 3 1/2-inch, lefthanded scissors in stock. Without noti1’ing the stationery store, the wholesaler shipped the 18 pairs of right-handed scissors, along with the six pairs of left-handed scissors, to the stationery store. The stationery store was aware that the wholesale price for the left-handed scissors was $3 per pair, or $1 less than the list price for the right-handed scissors.
    Was an enforceable contract formed when the wholesaler shipped the 24 pairs of scissors to the stationery store?
    (A) Yes, because the wholesaler’s shipment constituted acceptance of the offer, absent a seasonable notification by the wholesaler to the stationery store that the shipment was made for accommodation only.
    (B) Yes, because the wholesaler acted in good faith in making the shipment in reliance on the stationery store’s offer.
    (C) No, because the wholesaler could accept the stationery store’s offer only by a prompt promise to ship the goods ordered.
    (D) No, because acceptance by performance of an offer for immediate or prompt shipment is legally binding, unless the nonconforming goods are not reasonably resalable.
A
  1. (A) This is a rather tricky unilateral Contracts example. UCC Section 2-206(1)(a), provides that offers generally invite acceptance “in any manner and by any medium reasonable in the circumstances.” Generally, an offer may call for acceptance by either (1) a return promise or by (2) aspecified act. Here, the stationery store’s offer expressly caLled for the wholesaler to ship immediately the scissors. Thus, this is an example of acceptance by performance. Under 2-206(1)(b), it is not necessary that the seLler’s shipment constituting the acceptance be conforming goods. In this situation, if the seller sends nonconforming goods, then the buyer has a cause of action for breach of contract, unless the seller includes a notice of accommodation. Choice (B) is wrong, because, under Section 2-206, the “good faith” of the wholesaler is not a determining factor as to whether a contract was formed. Choice (C) is incorrect, because the offer called for a shipment, not a promise to ship. In addition, 2.206(B) states that an offer to buy goods can be accepted by either a prompt shipment or a prompt promise to ship. Choice (D) is a misstatement of the law. It does not matter whether the goods are reasonabLy resalable. If they are nonconforming, they will still constitute an acceptance of the offer (absent a notice of accommodation), but the offerorwould have a cause of action for breach of contract for the nonconforming shipment.
106
Q
  1. The operator of a hot dog stand sent the following purchase order to a bakery: “We are expecting an increase in business over the upcoming holidays. Please ship immediately 36 dozen 10-inch hot dog buns at your current list price of $6 per dozen.”
    The day after receiving the purchase order, the bakery shipped 24 dozen 10-inch hot dog buns and 12 dozen 8-inch hot dog buns. The owner of the bakery had instructed his staff to include a message that the bakery did not have enough 10-inch hot dog buns to fill the order, so they were including the 8-inch hot dog buns as an accommodation. However, the shipping department forgot to ‘include the message with the shipment.
    Which of the following is the most accurate statement regarding the operator’s legal rights following receipt of the hot dog buns?
    (A) The operator may either accept or reject all of the hot dog buns upon seasonable notice to the bakery, but it cannot accept only the 10-inch hot dog buns without the bakery’s approval.
    (B) The operator may either accept or reject all of the hot dog buns, or accept any commercial unit and reject the rest, but must give the bakery seasonable notice of either total or partial rejection.
    (C) The operator may either accept or reject all of the hot dog buns, or accept the 10-inch buns and reject the 8-inch buns, but it cannot accept any combination of the hot dog buns.
    (D) The operator may either accept or reject all of the hot dog buns, or, provided the bakery gives seasonable notice that the shipment was made for accommodation only, the operator may accept any combination of hot dog buns and reject the rest.
A
  1. (B) Both UCC Sections 2-601 and 2-608 make it clear that the buyer need not revoke or reject the entire amount (of nonconforming goods) but may accept and keep “any commercial unit or units and reject the rest.” Therefore, Choices (A) and (C) are incorrect. Likewise, the buyer may elect his remedy to either reject or revoke acceptance and recover damages for non-delivery under 2-713, or he may cover and collect damages under 2-712. In any event, a buyer who wishes to reject must “seasonably notify” the seller under Section 2-602; and Section 2-608(2) contains a similar requirement for one revoking acceptance. Choice (D) is wrong. If seasonable notice is given that the nonconforming shipment is meant as an accommodation, the shipment becomes a counteroffer that can either be accepted or rejected.
107
Q
  1. An owner of a tract of land in a foothill community hired a general contractor to construct a small summer cottage on the tract of land. The parties entered into a valid, written construction contract that contained a provision that called for an architect’s certificate of completion.
    After completion of the cottage, the owner hired an architect to inspect the contractor’s work and issue the certificate. The architect refused in bad faith to execute the certificate. The contractor then sued the owner for the entire contract amount. The owner argued that the lack of issuance of the certificate excused her from further performance under the contract.
    Upon hearing this argument, most courts will
    (A) order the architect to execute the certificate.
    (B) require the contractor to proceed in equity for reformation to eliminate the clause.
    (C) award the contractor damages against the architect.
    (D) excuse the condition and require the owner to pay damages to the contractor.
A
  1. (D) In building contracts, the architect’s certificate of completion, if made an express condition to the owner’s duty of payment, must be produced or excused before the builder will be entitled to the price. The certificate will be excused where its non-production is due to fraud or “bad faith” on the part of the architect. If the architect’s refusal to issue the certificate amounts to bad faith, in fact or in law, the courts will excuse the condition. The builder can then recover the contract price on proof of completion of the work without producing the certificate. The recovery will prevent unjust enrichment to the owner. For this reason, choice (D) is the best choice; even though there was an express condition precedent to the owner’s duty to pay, that condition will be excused and the owner will be required to pay the contractor. Choice (A) is incorrect, because the majority of courts dispense with the necessity of architects’ certificates when withheld in bad faith (excuse their performance as a condition) rather than order their execution. Choice (B) is incorrect, because in most cases the builder does not attack the validity of the contract’s provision for an architect’s certificate, which would require reformation. Choice (D) is incorrect, because the express condition is excused by the architect’s bad faith.
108
Q
  1. The owner of a large unimproved tract of land leased it to a tenant for five years at a monthly rental of $1,500. The rental agreement provided that the land was to be used as farmland. Shortly after the tenant took possession of the tract of land, he built thereon, at his own expense, a barn made of lumber, which was 15 feet wide, 20 feet long, and set on loose bricks. The barn increased the appraised market value of the tract of land from $250,000 to $275,000. The tenant then began farming operations on the land.
    Toward the end of the lease period, the owner informed the tenant that he was going to put the tract of land up for sale. The next month, when the lease expired, the parties settled a dispute over the tenant’s right, if any, to compensation for the improvements by the following written agreement:
    “On the sale of the tract of land, the owner hereby agrees to pay the tenant two-thirds of any sale proceeds in excess of $250,000, provided that tenant may remain on the farm for the next year, rent-free, while the owner tries to sell it. After that time, the tenant may remain on the land until closing of the sale, at a monthly rental of $1,000.”
    The owner initially set the asking price at $300,000. After receiving scant interest, the owner decided to reduce the price to $260,000. This price reduction so infuriated the tenant that he thereafter made negative comments about the farm to all of the prospective buyers. Finally, 14 months after the agreement between the owner and the tenant was executed, and after rejecting offers for $240,000 and $250,000, the owner sold the tract of land for $256,000 to a buyer who visited the land while the tenant was away on vacation. Thereupon, the tenant, who had failed to pay the last two months rent as per the agreement, moved out. After closing, the owner refused to pay the tenant any of the sale proceeds, and the tenant brought suit to recover damages for breach of contract.
    Which of the following is the owner’s most persuasive argument in defense of the tenant’s suit?
    (A) The tenant committed an uncured material breach of an implied promise to cooperate in the owner’s efforts to sell the property, or at least not to hinder the proposed sale.
    (B) The tenant’s negative comments about the farm to prospective buyers amounted to an anticipatory repudiation of the agreement between the owner and the tenant.
    (C) The tenant’s failure to pay any rent for the last two months was a material breach of contract that discharged the owner’s remaining duties of performance.
    (D) The agreement between the owner and the tenant was voidable because it was a restraint on alienation, since it conditioned a promise to pay for a conveyance of land upon an otherwise invalid leasehold contract.
A
  1. (A) First, it is important to realize that the owner and the tenant entered into an enforceable bilateral contract that was supported by adequate consideration. The owner is promising to pay the tenant a share of the proceeds from the sale of the tract of land, and in return, the tenant is promising not to sue the owner for the improvements he made on the property. Thereafter, the tenant committed a material breach of an implied promise to cooperate on the owner’s efforts to sell the property by making negative comments to prospective buyers. Where a party to a contract for an agreed exchange of performance knowingly prevents, hinders, or makes more costly the other’s performance, such conduct is a breach of contract for which an action will lie. In this situation, the breach is of an implied promise against prevention. For example, an owner who excludes a contractor from his premises prevents performance of the work to be done. Thus, he is liable in damages for such breach, measured by the contractor’s lost profit on the job. In brief, the implied promise against prevention is that neither party will knowingly obstruct or make more expensive the other’s performance. Choice (B) is wrong, because anticipatory repudiation is where one party to an executory bilateral contract repudiates the contract in advance of the time set for performance by announcing that he will not perform. Here, the tenant did not anticipatorily repudiate. Choice (C) is incorrect, because failure to pay rent for two months is a partial, non-material breach. Choice (D) is clearly wrong, because the contract between the owner and the tenant does not restrain the alienation of the property.
109
Q
  1. A homeowner owned a single-story ranch- style home that was her primary residence. The homeowner received notice that her uncle had passed away and left her a two-story mansion in a neighboring city. The homeowner decided to move her primary residence to the mansion and rent the ranch-style home. She entered into a one- year written lease agreement with a tenant. The agreement set the monthly rent at $1,000. Shortly after the tenant took possession of the home, he built, at his own expense, a room addition onto the home. The room addition increased the appraised market value of the home from $200,000 to $250,000.
    At the expiration of the lease, the homeowner informed the tenant that she had decided to sell the home. She offered the tenant the first opportunity to buy the home, but the tenant replied that he could not afford to do so. The tenant did claim that he should be entitled to compensation for the room addition, since it had increased the value of the home, and the homeowner agreed. The tenant and the homeowner then executed the following agreement:
    “On the sale of the ranch-style home, the homeowner hereby promises to pay the tenant one- half of any sale proceeds in excess of $200,000, in compensation of the tenant’s efforts in constructing the room addition onto the home. In addition, it is hereby agreed that the tenant may remain on the land until the sale is finalized, at a monthly rent of
    $500.,’
    The homeowner initially set the asking price at $250,000, but received no offers to purchase the home. The homeowner decided to reduce the price to $210,000. This price reduction so infuriated the tenant that he thereafter made negative comments about the home to all of the prospective buyers who visited the home.
    Two months later, the homeowner sold the home to a buyer for $206,000. The buyer had visited the home while the tenant was away on a business trip and therefore did not hear the tenant’s negative comments. Thereupon, the tenant, who had paid no rent for the final two months, moved out. After the sale was finalized, the homeowner refused to pay the tenant any of the sale proceeds.

Which of the following statements, if true, most persuasively supports the tenant’s contention that he is entitled to recover at least $4,000 from the owner (or the equivalent of one-half of the sale proceeds in excess of $200,000, minus two months’ unpaid rent at $500 per month)?
(A) The owner breached an implied promise by failing to attempt to sell the property at $250,000, which was the appraised market value of the home.
(B) Since the tenant made no negative comments about the home to the buyer, there is no showing that the tenant’s remarks to the other prospective buyers necessarily caused any loss to the owner (i.e., prevented her from selling the home for more than $210,000).
(C) The agreement between the homeowner that the tenant contained only one express condition (i.e., the tenant was permitted to remain in the home during the owner’s efforts to sell it), and since that condition has occurred, the tenant is entitled to his share of the proceeds from the sale.
(D) Even if the tenant’s failure to pay any rent for the last two months was a material breach of contract, the owner’s promise to pay the tenant a share of the proceeds of the sale was an independent covenant.

A
  1. (B) The most persuasive argument for the tenant should be one in which the tenant will be deemed not to have breached his contract with the owner. Choice (B) is the correct answer. By showing that the tenant made no negative comments to the buyer, there is no evidence that the tenant breached his implied promise to cooperate by preventing the owner from realizing the maximum possible sales price. Absent such a breach, the tenant would be entitled to recover the proceeds. Choice (C) is incorrect, because the owner’s promise to pay the sale proceeds was not conditioned on the tenant’s remaining on the premises, but rather in settlement of the tenant’s claim for compensation for the room addition. Choice (D) is incorrect, because, while it is true that the owner’s promise to pay the tenant a share of the proceeds was an independent covenant from the payment of rent, the owner’s duty to pay would still be discharged by the tenant’s material breach of an implied promise to cooperate. Choice (A) is incorrect, because the seller is obligated merely to use best efforts to sell at the maximum possible price, not necessarily the appraised market value. The owner did not receive any offers at the appraised value before finally selling.
110
Q
  1. An employee had been employed by a hotel as a window cleaner for eight years when the hotel issued to him and other employees the following certificate:
    “In appreciation of the faithful service hitherto rendered by you as an employee of this hotel, there will be paid in the event of your death, if still an employee of this hotel, to the party designated by you below as your beneficiary, the sum of $5,000. The issuance of this certificate is understood to be purely gratuitous.”
    Upon receipt of the certificate, the employee designated his wife as beneficiary, signed it, and returned the form to the hotel.
    One year later, while still employed by the hotel, the employee died. In an action by the employee’s widow against the hotel to recover the sum of $5,000, which of the following, if established, is the strongest argument against enforcement of the certificate agreement?
    (A) The widow was unaware of the certificate agreement until after the employee died.
    (B) There was no privity of contract between the widow and the hotel.
    (C) The widowin no way relied on the hotel’s promise to pay the $5,000.
    (D) There was insufficient consideration to support the certificate agreement.
A
  1. (D) The trick to this question is to realize this is really a formation problem. Many students will get distracted by the fact that a third-party beneficiary (the widow) is attempting to sue. However, a third-party beneficiary does not receive any rights unless it is first determined that a valid contract was formed. Here, the hotel made a gratuitous promise to all of its employees. As such, its promise was not enforceable. Since no valid contract involving the payment of the $5,000 was ever formed, the widow never received any right that could be enforced. Choices (A), (B) and (C) are all therefore wrong. Each might have been a concern if the widow had received rights under a validly formed contract, but since that is not the case, they must aLL be discounted.
111
Q
  1. One morning, an employee arrived at work and found his boss passed out on the floor of his office. The employee determined that his boss was not breathing, immediately rendered CPR to his boss, and succeeded in reviving him. The employee then called the paramedics, who took the boss to the hospital.
    Two week later, the boss returned to work. Grateful for the employee’s quick action in saving his life, the boss said to the employee, “You’ll have ajob with me for life.”
    Which of the following best characterizes the legal relationship between the boss and the employee on that date?
    (A) &s per the boss’s oral promise, the employee had an enforceable lifetime employment contract.
    (B) The boss had a duty to pay the employee reasonable compensation for saving his life, based upon a contract implied-in-fact.
    (C) The employee’s act of saving the boss’s life was sufficient past consideration to render enforceable the boss’s subsequent promise.
    (D) Since the employee gratuitously rendered assistance to the boss, there was insufficient consideration to support the boss’s subsequent promise to provide the employee with lifetime employment.
A
  1. (D) The general rule is that past consideration is not good consideration. Since the boss made his promise in exchange for something the employee had already done, this is past consideration. Some jurisdictions recognize an exception to this rule and will enforce such a promise on the grounds of moral obligation. However, those jurisdictions will do so only when the promise should be enforced to avoid injustice. Here, the employee gratuitously conferred a benefit on the boss; it would not produce an injustice if the boss failed to perform his gratuitous promise. Therefore, even in those jurisdictions that recognize moral obligation, the promise would not be enforced. Choices (A) and (C) are therefore incorrect. Choice (B) is wrong, because there was no implied-in-fact contract formed between the parties. When the employee saved the boss’s life, he had no reasonabLe expectation of compensation for his services.
112
Q
  1. A buyer contracted in writing to purchase 1,000 gidgets (which are similar to widgets but are more popular because of their greater durability and- resiliency) from a manufacturer for a total contract price of $4,000. After this agreement was executed and before delivery, the buyer and the manufacturer agreed in a telephone conversation to change the quantity term from 1,000 to 800 gidgets.
    This modification is probably unenforceable, because
    (A) it violates the statute of frauds.
    (B) there was no consideration.
    (C) the original contract was in writing.
    (D) the parol evidence rule bars contradictory terms.
A
  1. (A) According to UCC Section 2-201 (1), “Except as otherwise noted in this section a contract for the sale of goods for the price of $500 or more is not enforceable by way of action or defense unless there is some writing …“ Since this section also applies to (oral) modifications, choice (A) is clearly the best answer. Choice (B) is wrong, because in contrast to the common law rule, modifications under the UCC do not require new consideration to be enforceable. Choice (C) is wrong, because oral modifications of written contracts are permissible under the UCC unless the contract states that it can onLy be modified in writing. Choice (D) is incorrect, because the parol evidence rule does not apply to subsequent oraL agreements.
113
Q
  1. A small electronics corporation decided to radically overhaul its manufacturing processes and borrowed $200,000 from the bank for this purpose. The loan was secured by a mortgage on the corporation’s plant and building site.
    When the debt came due, the corporation was short of ready cash and the bank threatened to foreclose. One of the shareholders of the corporation then intervened on behalf of the corporation and told the bank officials that if they would refrain from any legal action against the company for a year, she would personally see that the debt was paid. The bank orally agreed to the shareholder’s arrangement. However, it was never reduced to writing.
    The bank’s promise to the shareholder to refrain from foreclosing on the mortgage would constitute
    (A) a void promise at the time of inception.
    (B) a voidable promise as violative of the statute of frauds.
    (C) an unenforceable promise, because the corporation had a pre-existing duty to pay the debt at maturity.
    (D) an enforceable promise, binding the shareholder as a surety.
A
  1. (B) The following contracts come within the Statute of Frauds and must be in writing to be enforceable: 1) a contract in consideration of marriage; 2) a contract which, by its terms, cannot be performed within the span of one year; 3) a contract of an executor or administrator; 4) a contract of guarantee or surety; and 5) a contract for the sale of goods of $500 or more. Based on 4), an oral promise to answer for the debt or default of another is unenforceable under the Statute of Frauds. Choice (D) is therefore incorrect. Exam Tip: An oral contract that does not comply with the Statute is “unenforceable” rather than “void.” As a consequence, Choice (A) is less preferred. Choice (C) is wrong, because there is no reason why the firm’s pre-existing duty should also bind its shareholder. The corporate form protects shareholders from liability on corporate debts.
114
Q
  1. Upon graduation from high school, a student enrolled in a college close to where he lived. However, since the student did not have a car, he rode the bus to and from the college each day. The student soon discovered that he was spending up to two hours each day to make the round trip to and from college on the bus, whereas the trip could be made in half that time if he had a car. He decided to buy a car and asked his father to accompany him as he looked for a car to purchase.
    After visiting a number of car lots, the son finally found a used car to his liking. However, the son only had $2,500 in his bank account, and the price of the car was $10,000. Since the son had not built up a credit history, the car lot was reluctant to sell the car to the son. The father then told the finance manager of the car lot, “If you will sell the car to my son and for any reason he defaults on the payments, I will pay the balance due.” The finance manager then approved the deal, which required the son to pay $2,500 down and monthly payments of $500 for 15 months. Six months later, the son defaulted on the payment plan, and the car lot sought recovery from the father without first suing the son.
    With regard to the father’s obligation under his promise to the finance manager, the court would most likely
    (A) relieve the father of liability, because the car lot failed to first sue the son.
    (B) relieve the father of liability, because the father was never under a duty as a surety.
    (C) not relieve the father of liability, because of the main purpose exception.
    (D) not relieve the father of liability, because parents are responsible for the debts of their children.
A
  1. (B) The father’s oral promise to personally guarantee the son’s debt is unenforceable under the Statute of Frauds. As a result, Choice (B) is correct, because the father was never under a surety duty to pay the debt. Choice (A) is incorrect, because the contract did not require the car lot to sue the son before liability falls to the father. Choice (C) is incorrect, because under the “main purpose rule,” an oral promise by a promisor to pay the debt of another is enforceable where the party making the promise does so to further his own economic advantage. In other words, the consideration for the promise must be beneficial to the promisor either personally or directly. Since the father did not stand to directly benefit himself in guaranteeing the son’s loan, the main purpose rule is inapplicable. Choice (D) is an incorrect statement of the law. As a general rule, parents have no such responsibility.
115
Q
  1. A president of a small computer company decided to expand his operation. On behalf of his company, he borrowed $500,000 from a credit union for this purpose. The company was to repay the loan at the rate of $2,500 per month. The loan was secured by a mortgage on the building that housed the company’s operation center.
    Eight months later, the company’s sales started to drop and the company began experiencing cash flow problems. As a result, the company failed to make its loan payments for three consecutive months, causing the credit union to threaten to foreclose on the mortgage. The president’s father, a retired wealthy investor, then intervened on behalf of the company, paid the three missed loan payments, and told the credit union that if they would refrain from any legal action against the company for a year, he would personally see that the debt was paid. The credit union orally agreed to the father’s surety arrangement. However, it was never reduced to writing.
    Six months later, the company once again missed consecutive payments, and the credit union filed a foreclosure suit against the company. The father did not learn of the suit until a week later, but he raised no objection, since he thought the credit union was violating its agreement with him by foreclosing within the one-year period, thus relieving him of his part of the bargain.
    Two weeks later, the credit union’s loan officer called the father and said that the credit union would hold off on the foreclosure suit as per their agreement, since the company had just made a new technological development that would place it in a very lucrative and competitive position. Soon after the new technological development took place, the company’s business fortunes declined, which resulted in the company’s insolvency.
    In an action by the credit union against the appointed receiver in bankruptcy and the father, the credit union will most likely recover for the outstanding loan from
    (A) the father only.
    (B) the receiver only.
    (C) either the father or the receiver.
    (D) both the father and the receiver.
A
  1. (B) The father’s oral promise to personally guarantee the company’s debt was voidable under the Statute of Frauds. Since the father was therefore relieved of liability when his surety promise became voidable, Choice (B) is the only correct answer, since Choices (A), (C), and (D) all provide for recovery from the father.
116
Q
  1. In a written contract, a seller agreed to deliver to a buyer 1,000 widgets at a stipulated price of $10 each, FOB at the seller’s place of business. The contract stipulated that “any party who wishes to assign this contract must have the written consent of the other party.” On March 1, the seller placed the widgets on board a cargo vessel that was destined to transport the widgets to the buyer. On March 2, the buyer received the following telegram from the seller:
    “Please be advised that the widgets are in transit. In addition, I hereby assign all my rights under our contract to our creditor.”
    The buyer did not consent to the assignment. The next day, the ship carrying the widgets sank in a violent storm, destroying its entire cargo.
    If the creditor brings an appropriate action against the buyer, the former will most likely recover
    (A) nothing, because the buyer never assented to the assignment.
    (B) nothing, because the buyer never received the widgets.
    (C) the contract price of$10,000.
    (D) the difference between the contract price and the market value of the widgets.
A
  1. (C) If a contract reads “FO.B. seller’s place of business,” UCC Section 2-319(1)(a) indicates that the seller must then: “ship the goods in the manner provided in Section 2-504 and bear the expense and risk of putting them into the possession of the carrier.” Thus, under 2-509(1)(a), the risk in such a case passes to the buyer when the goods are “duly delivered to the carrier.” Since the buyer bore the risk of loss, choice (B) is wrong. Choice (D) is also wrong, because the buyer would have no right to recover damages, since he bore the risk of loss on the widgets. As for the anti- assignment clause, UCC Section 2-210(2) states: “A right to damages for breach of the whole contract or a right arising out of the assignor’s due performance of his entire obligation can be assigned despite agreement otherwise.” Therefore, the seller was entitled to assign the right to payment without the buyer’s consent. Choice (A) is therefore wrong.
117
Q
  1. A buyer, located on the west coast contacted a seller, located on the east coast, about purchasing flanges that the buyer used in their manufacturing process. The parties entered into a contract whereby the seller agreed to deliver to the buyer 1,000 flanges for $10,000. The contract stipulated that it was FOB at the seller’s place of business. The seller then delivered the flanges to a delivery company, which was to transport them to the buyer. While en route, the vehicle that was transporting the flanges was involved in an accident and the flanges were destroyed. When the buyer received notice of the accident, the buyer immediately contacted the seller and demanded that it ship replacement flanges. The seller refused to do so.
    In an action by the buyer against the seller for breach of contract, the buyer will
    (A) succeed, because the risk of loss was on the seller.
    (B) succeed, because the carrier was the seller’s agent.
    (C) not succeed, because the risk of loss was on the buyer.
    (D) not succeed, because of frustration of purpose.
A
  1. (C) Under UCC Section 2-509(1)(a), where a contract reads “F.O.B. seller’s place of business,” the risk of loss shifts to the buyer once the seller delivers the goods to the carrier. Therefore, the buyer bore the risk of loss at the time the flanges were destroyed, so the buyer has no right of recovery from the seller. Choices (A) and (B) are therefore wrong. Choice (D) is incorrect, because when a party is charged with assuming the risk of something happening, the occurrence of that event cannot be the basis of a discharge argument, such as frustration of purpose.
118
Q
  1. A farmer owned a piece of land on which he grew strawberries. Over the years, the farmer had earned a reputation for growing strawberries that were extremely sweet and juicy. For that reason, consumers always looked forward to strawberry season, when the farmer’s strawberries would be available.
    Hoping to capitalize on the farmer’s reputation, a produce retailer who operated three stores in the area contacted the farmer about the possibility of carrying the farmer’s strawberries in his stores. After lengthy negotiations, the parties executed the following written agreement:
    “The farmer will supply the retailer with all of its needs of strawberries from April through July each year for the next three years. The retailer will pay $5.00 per bushel of strawberries delivered.”
    On April 1 of the first year of the contract, the retailer submitted an order for 75 bushels of strawberries. Upon receiving the order, the farmer sent the retailer the following fax:
    “Your order was way more than I anticipated. If! sent you that many strawberries I would not have enough for my other customers. I can send you 50 bushels, but no more.”
    The retailer then notified the farmer that if he did not deliver all 75 bushels, the retailer would sue him for breach of contract.
    The agreement entered into between the farmer and the retailer would best be described as a (an)
    (A) illusory contract.
    (B) best efforts contract.
    (C) requirements contract.
    (D) aleatory contract.
A
  1. (C) The agreement whereby the farmer agreed to supply all of the retailer’s requirements of strawberries for three years would be an example of a requirements contract. With respect to requirements and output contracts, UCC Section 2-306 provides “a term which measures the quantity by the output of the seller or the requirements of the buyer means such actual output or requirements as may occur in good faith, except that no quantity unreasonably disproportionate to any stated estimate or in the absence of a stated estimate to any normal or otherwise comparable prior output or requirements may be tendered or demanded.” Moreover, under this Section, a contract for output or requirements is not too indefinite, since it is held to mean the actual good-faith output or requirements of the particular party. Nor does such a contract lack mutuality of obligation, since the party who will determine quantity is required to operate his plant or conduct his business in good faith and according to commercial standards of fair dealing. Therefore, Choices (A) and CD) are incorrect. Choice (B) is not the best answer. While it is true that in a requirements contract the buyer must use best efforts to have requirements, Choice (C) is a more direct label as to the type of contract involved.
119
Q
  1. A producer of widgets contacted a manufacturer of boxes about the possibility of purcIiasing specially designed boxes for shipping the widgets to customers. The producer sent the manufacturer a sheet containing the dimensions of the boxes’ needs, as well as the logo to be placed on each box. After negotiations, the parties entered into a written contract whereby the manufacturer agreed to supply the producer with as many of the boxes as the producer may require up to a maximum of 10,000 per month. Later that day, the producer telephoned the manufacturer and said that they were running a one-time promotion and expected to sell an unusually large number of widgets. Therefore, for the first month only, they would need 15,000 boxes. The manufacturer agreed to this request.
    The manufacturer’s oral agreement to supply the producer with the 15,000 boxes for the first month would most likely be held
    (A) enforceable.
    (B) unenforceable, because their written contract on the same date was the final integration of the terms of their agreement.
    (C) unenforceable, because the agreement was violative of the parol evidence rule.
    (D) unenforceable, because there was inadequate consideration to support the manufacturer’s unilateral promise.
A
  1. (A) Under the parol evidence rule, evidence of any prior oral or written agreements or contemporaneous oral agreements which alters, varies, or contradicts the terms of a fully integrated writing is barred from admission to contradict that writing. However, the parol evidence rule does not apply to subsequent agreements. Since the agreement in question was formed after the written contract was executed, the parol evidence rule would be inapplicable. Therefore, Choices (B) and (C) are wrong. Choice (D) is wrong because, under the UCC, a modification does not require new consideration to be valid. Thus the agreement is valid, and Choice (A) is the correct answer.
120
Q
  1. A restaurant ran a promotion in a local newspaper, stating the following:
    “MOTHER’S DAY SPECIAL
    We will be open for brunch on Mother’s Day from 10 am to 2 pm offering an extensive selection of dishes to honor mothers of all ages. Call us to reserve your table.”
    The response to the promotion was more than the restaurant expected, and the restaurant was soon overbooked for the Mother’s Day brunch. On the day before Mother’s Day, the restaurant owner decided to double-check inventory to make sure the restaurant had enough food and supplies to handle the large Mother’s Day crowd. To her horror, she discovered that the restaurant was almost out of eggs. Knowing that a large supply of eggs would be needed for the omelet station, the owner immediately sent the following e-mail to her egg supplier:
    “Desperately need 20 flats of Grade AA eggs for tomorrow’s brunch. Money is no object. The eggs must be delivered today.”
    The supplier e-mailed back:
    “No problem! I’ll load them on the truck and deliver them within the hour.”
    With respect to the agreement between the owner and the supplier, which of the following statements is most accurate regarding the omittance of a fixed contract price?
    (A) The contract is unenforceable as violative of the statute of frauds.
    (B) The contract is unenforceable, because of indefiniteness.
    (C) The contract may be enforceable if it is later modified to include the price term.
    (D) The contract is enforceable with reasonable price being fixed at time of delivery.
A
  1. (D) In compliance with UCC Section 2-305, the parties, if they so intend, can conclude a contract for sale even though the price is not settled. In such a case, the price is a “reasonable price” at the time of delivery if: (a) nothing is said as to price; or (b) the price is left to be agreed by the parties and they fail to agree; or (c) the price is to be fixed in terms of some agreed market or other standard as set or recorded by a third person or agency and it is not so set or recorded. Thus, when there is a gap as to price, UCC Section,2-305 directs the court to determine a “reasonable price.” Choices (B) and (C) are therefore wrong. Choice (A) is incorrect, because the e-mails exchanged between the parties would be sufficient to satisfy the Statute of Frauds.
121
Q
  1. An aluminum can manufacturer entered into negotiations with an aluminum sheeting supplier to supply the manufacturer’s requirements of aluminum sheeting. After extensive discussions, the parties signed the following agreement on June 1:
    “The supplier agrees to supply all of the manufacturer’s requirements of aluminum sheeting for a period of two years beginning on August 1, at a price of $3.00 per unit.”
    On June 16, the manufacturer notified the supplier that it would need a delivery of 2,000 units by August 1 to accommodate its needs. The supplier informed the manufacturer that it did not contemplate such high requirements, since its plant’s capacity was only 2,800 per month. Moreover, the supplier pointed out that in order to meet the manufacturer’s order of 2,000 units, it would probably lose two long-time customers, whose outstanding orders would have to be canceled.
    After a week of negotiations, on June 23 the supplier orally agreed to deliver 2,000 units of sheeting to the manufacturer by August 1. The parties then signed the following contract:
    “The supplier agrees to supply all of the manufacturer’s requirements of aluminum sheeting for a period of two years beginning August 1, at a price of $3.50 per unit. The manufacturer agrees that it will require a minimum of 800 units in every month of the agreement.”
    On June 25 the supplier notified its two longtime customers that it was canceling their August orders (of 800 units) because of other contract commitments.
    Which of the following is the most accurate statement regarding the written agreement between the parties on June 23?
    (A) The agreement constituted a valid modification of their June 1 contract.
    (B) The agreement was unenforceable, since the supplier was under a pre-existing duty to supply the manufacturer with the sheeting under their June 1 contract.
    (C) The agreement constituted an enforceable reformation of their June 1 contract.
    (D) The agreement was unenforceable, since there was no new consideration.
A
  1. (A) The June 23 agreement between the manufacturer and the supplier would constitute a validly enforceable modification of their June 1 contractual agreement. In accordance with UCC Section 2-209 (1), an agreement modifying a contract subject to the UCC needs no (e.g., new) consideration to be binding. Choice (D) is therefore wrong. Choice (B) is incorrect, because the pre-existing duty rule is a consideration concern, and as stated above, consideration is not required for the modification to be valid. Choice (C) is incorrect, because this is not a reformation. A reformation is where a writing is changed to reflect the true agreement of the parties. This goes beyond a reformation, since the entire agreement is being changed, not just the writing.
122
Q
  1. After a week of negotiations, an importer of cigars from a foreign country agreed in writing to supply a smoke shop with all of their needs of cigars for two years at a price of $1 per cigar. The following day, the foreign govermnent was overthrown. One week later, the United States govermnent announced an embargo on the importation of all products from the foreign country. The importer contacted the smoke shop and informed them that as a result of the embargo, the importer would not be able to fill any of the smoke shop’s orders for cigars from the foreign country.
    What is the probable legal effect of the United States government’s embargo on the contract between the importer and the smoke shop?
    (A) The smoke shop would be entitled to recover the difference between $1 and the cost of purchasing cigars manufactured in another country.
    (B) Both parties’ duties of performance would be discharged by frustration of purpose.
    (C) Both parties’ duties of performance would be suspended through temporary impossibility.
    (D) The parties’ duties of performance would be excused because of a supervening illegality.
A
  1. (D) The probable legal effect of the United States government’s embargo on the foreign country’s imports would be to excuse the duties of performance of the parties because of a supervening illegality. Where performance of a contract has become illegal by change in law after the time of contracting, the promisor’s duty is discharged unless he has assumed the risk of it or unless his fault has contributed to the prohibition. Therefore, Choice (D) is the best answer. If the embargo had been temporary, then Choice (C) would be correct. Choice (A) is wrong, because the discharge prevents the smoke shop from collecting damages for non-performance. Choice (B) is wrong, because the purpose behind the contract still exists; perfor. mance of the contract is not possible, because of the supervening illegality.
123
Q
  1. A law school advertised in various law journals for a legal ethics and jurisprudence professor. Although a number of lawyers applied for the position, one of the applicants submitted a particularly impressive resume. The dean of faculty for the law school immediately wrote to the applicant to offer him the position. In the letter, the dean offered to employ the applicant for the upcoming academic year, which started on August 15, at a salary of $75,000. The letter also listed the employment terms and conditions, one of which stated that professors are entitled to five paid sick days during the year. The dean included a detailed employment contract containing all these terms, for the applicant’s signature.
    After the applicant received this letter, he replied by mail, requesting a salary of $85,000 and sick leave of 10 days. Upon receipt of the applicant’s letter, the dean telephoned the applicant and told him that the law school followed the American Bar Association guidelines with respect to salary and sick leave for its professors. The applicant acquiesced on the salary question, but insisted that his sick leave be extended to 10 days.
    The dean replied, “Let’s not permit a minor issue to stand in the way of your coming to teach at our law school. In the event you require more than 5 sick days, I promise that the matter will be taken care of to your benefit.” The applicant informed the dean that he would sign the contract and forward it to the dean, which he promptly did.
    The applicant began teaching at the law school on August 15. Three months later, the applicant was out sick for five days with laryngitis. The applicant did not miss another school day until Monday, March 8, when he fell ill with food poisoning. This illness kept him home for five additional sick days. The applicant returned to work on March 15. When the applicant received his salary check at the end of the week, his check did not include payment for the previous week (from March 8 to March 12). Accompanying the check was a statement that read:
    “Salary payment for period from Monday, March 1 through Friday, March 19 with pro rata deduction to reflect five teaching days missed during said period.”
    When the applicant received his check, he immediately confronted the dean and requested full payment for the week of March 8 through 12. The dean responded, “I’m sorry, but there is absolutely nothing I can do about it.”
    The applicant asserted a claim against the law school for breach of contract. The applicant offers to introduce evidence that during his telephone conversation with dean of faculty, the dean promised, if necessary, to provide him with additional sick days.
    The most accurate statement concerning the dean’s oral promise would be that
    (A) parol evidence is admissible to show that the parties assented to their written contract only as a partial integration of their complete contract.
    (B) parol evidence is admissible to prove a subsequent oral agreement that varies or contradicts the terms of a prior written contract.
    (C) parol evidence is admissible to show that the written contract is not enforceable, because of undue influence or fraud.
    (D) parol evidence is inadmissible to prove contemporaneous oral agreements that vary or contradict the terms of a written contract.
A
  1. (D) In accordance with the parol evidence rule, when a contract is expressed in a writing which is intended to be the complete and final expression of the rights and duties of the parties, parol evidence of prior oral or written negotiations or agreements of the parties or of their contemporaneous oral agreements, which varies or contradicts the written contract, is not admissible. The dean’s contemporaneous oral promise to extend the applicant’s sick leave to 10 days would clearly vary the terms of the written employment contract, and therefore, the applicant would not be permitted to introduce parol evidence of the dean’s oral promise. Choice (A) is wrong, because the detailed nature of the employment contract would be strong evidence that it was intended to be a full integration of their agreement. When the face of the writing indicates it is intended to be a complete integration, parol evidence cannot be used to contradict that interpretation. Choice (B) is wrong, because the oral agreement was not subsequent to the forming of the contract; it took place prior to the applicant signing the employment contract. Choice (C) is incorrect, because there is no indication of any fraud or undue influence.
124
Q
124. A law student contacted a tutor about the possibility of hiring the tutor to help the law student prepare for his upcoming finals. The tutor sent the law student a copy of the tutor’s standard tutorial agreement. The agreement stated that the tutor’s rate was $50 per one-hour session. The law student then telephoned the tutor and said, “This agreement looks fine to me. I’d like you to give me 10 sessions over the next 10 days.” The tutor replied, That will be fine. Fill in 10 sessions on the agreement, sign it, and bring it to me tomorrow when we have our first session.” The student then said, “I want you to really push me to do my best. In fact, if I get an A in any class I’ll pay you an additional $100 for each A.”
The next day, the law student brought the signed agreement to the tutorial session. The tutor conducted all 10 sessions. Two weeks later, when the results of the finals were released, the law student excitedly called the tutor and exclaimed, “Thank you so much I just found out that I received two A’s and two B’s in my classes!” The tutor then sent the law student a bill for $700. Two days later the tutor received a check from the law student in the amount of $500. Included with the check was a note which read, “You taught me contract law very well. I now know that I am not obligated to pay you the additional $200 for the A’s, because that was just a gratuitous promise.” On the back of the check the law student typed the following:
“Endorsement of this check by payee constitutes surrender of all claims against me arising out of our tutorial arrangement.”
In need of money, the tutor endorsed and cashed the check.
What is the probable legal effect of the tutor’s endorsement of the check?
(A) It constituted a discharge of a liquidated claim.
(B) It constituted an accord and satisfaction of an unliquidated claim.
(C) Part payment of a liquidated claim would not constitute a discharge of the entire amount due.
(D) Part payment of an unliquidated claim does not constitute sufficient consideration for the discharge of the entire claim.
A
  1. (B) As a rule, in the case of an unliquidated or disputed obligation, if a tender is made by the debtor of money or other thing as in full satisfaction of his debt, the acceptance by the creditor of that which is tendered constitutes a complete discharge of the debt. Thus, where the debt is unliquidated, meaning the amount is in dispute, a payment of a less amount than the sum claimed on condition that it be accepted in full discharge of the debt constitutes an accord and satisfaction. Here, a dispute existed because the tutor thought she was entitled to $700, but the law student felt she was entitled to only $500. Choices (A) and (C) are therefore wrong, because a liquidated claim is one that is not in dispute. Choice (0) is incorrect, because the consideration to support the accord and satisfaction is deemed to flow from the dispute.
125
Q
  1. A businessman living was an avid baseball fan who frequently traveled to a casino and placed wagers on baseball games. One October, his beloved baseball team was playing in the playoffs and he wanted to place a large bet on his team. He told one of his employees that he wanted to bet $10,000 on his team to win the championship, but that he did not have the time to travel to the casino to place the bet. He then told his employee that he wanted the employee to travel to the casino and place the bet for him. The businessman and the employee agreed that the employee would travel to the casino, place a $10,000 bet for the businessman and then immediately return. In exchange, the employee would be paid $500. The parties reduced this agreement to writing.
    After the agreement was signed, the businessman handed the employee $10,000 and a round-trip airline ticket. The businessman then told the employee that he would be paid $500 after the employee returned.
    The employee arrived the next day and immediately went to the casino. There he noticed that the marquis in the parking lot was unlit and covered in a banner that read, “Casino Temporarily Closed Due to Renovations.” Unable to reach the businessman by telephone, the employee placed the bet at another gaming establishment located next door to the casino.
    The following morning, the employee flew back and gave the betting receipt to the businessman. When the businessman saw that the bet had been made at another gaming establishment, he angrily told the employee, “I purposely directed you to wager that $10,000 at the casino. Since you failed to follow my instructions, the deal’s off.” As a result, the businessman refused to pay the employee the $500.
    If the employee initiates suit for breach of contract, the court will most likely determine that placing the $10,000 wager at the other gaming establishment, rather than at the casino, constituted a
    (A) breach of contract.
    (B) modification.
    (C) constructive condition precedent that was substantially performed.
    (D) discharge by impossibility.
A
  1. (C) This is a very tricky question, in which process of elimination is a major factor in determining the correct answer. Choice (A) is incorrect, because it was not possible to place the bet at the casino, since it was temporarily closed. Therefore, the employee was not in breach for failing to do so. Choice (B) is wrong, because a modification is an agreement between the parties to change a contract. The employee tried to contact the businessman when he discovered the casino was closed, but he was unable to do so. There was thus no agreement reached to change the contract. Choice (D) is wrong, because the essence of the agreement was to place a bet on the baseball team to win the championship. In situations where there is an impediment to a matter that, while important, is incidental to the main obligations of the parties, it is permissible to use a commercially reasonable substitute when such is available. Therefore, the contract was not discharged by the closing of the casino, since a commercially reasonable substitute method of performance (placing the bet at the other gaming establishment) was available. That leaves Choice (C). Here, the employee was obligated to travel to the casino and wager $10,000 for the businessman. After the bet was placed, the employee would then receive $500 for his efforts. This meant that the placing of the bet was a constructive condition precedent to the employee’s being paid the $500. This condition was substantially performed when the bet was placed at the Keno Palace. As a result, the employee should be entitled to receive that which was promised him, i.e., $500.
126
Q
  1. As part of an externship program conducted by his law school, a law student went to work for a lawyer as a clerk. After six months, the lawyer was very impressed with the law student’s work. He called the law student into his office and told him, “I know you were only to work for me for six months in exchange for credit in your law school extemship program. However, I’d like you to stay on as a clerk. I’ll pay you $25 an hour for any work you do for me between now and the bar exam. In addition, if you put in a minimum of 10 hours per week during that time, I promise to hire you full-time as an associate when you pass the bar exam.” The attorney then reduced this agreement to writing, which both he and the law student signed.
    The law student continued to work for the lawyer throughout the rest of his law school career and while he was studying for the bar exam. During that time, the law student always worked a minimum of 10 hours per week. While the law student was awaiting the results of the bar exam, his uncle passed away and left him a large apartment building. The law student decided he did not want to practice law but wanted to spend all of his time managing the apartment building. He told his roommate, a fellow law student who was awaiting his bar results, about his plans and told the roommate he would assign to the roommate the right to be hired by the lawyer as an associate, in exchange for $100. The roommate paid the $100 to the law student, who then executed a written assignment of the right to be hired by the lawyer as an associate.
    The roommate passed the bar exam, but the lawyer refused to hire him as an associate. Which of the following is the most accurate statement regarding the legal effect of this assignment?
    (A) The lawyer would be required to recognize the validity of the assignment so that the roommate would be entitled to be hired as an associate.
    (B) The lawyer would be required to recognize the validity of the assignment, but would be entitled to demand adequate assurances of performance from the roommate as to the latter’s willingness and professional capacity to perform satisfactory work.
    (C) The lawyer would not be required to recognize the validity of the assignment of rights, because a contract to make a future assignment of a right is not an assignment.
    (D) The lawyer would not be required to recognize the validity of the assignment, because neither the rights nor the duties under a personal service contract are properly assignable.
A
  1. (D) As a general rule, rights and duties that are in their nature limited to exercise by the promisee alone are not transferrable. No right or duty that is limited in its nature to the personality of the promisee alone is capable of transfer without the assent of the promisor. Moreover, an employer’s right to an employee’s services, where the employee is to work under the personal direction of the employer, is not assignable without the employer’s assent, because the duty is impliedly conditioned to render the service to the promisee alone. In this regard, it is often said that personal service contracts are not assignable; in other words, that neither employer nor employee may assign his rights without the other’s assent. Therefore, Choices (A) and (B) are wrong, because the lawyer does not have to recognize the validity of the assignment. Choice (C) is wrong, because this is an existing contractual right, not a transfer of a future right that may or may not arise. A transfer of an existing contractual right (provided the transfer is permissible) that is not to arise until a future date is valid.
127
Q
  1. A gambler learned that a friend of his was planning a vacation to visit some casinos. The gambler, who was an avid college football fan, approached his friend. The gambler explained that he wanted to place a bet with a sports-betting agency that his beloved team would win the college football championship the next year. The gambler further explained that he had read that the odds of his team winning the football championship next year were listed as 100-1, and he wanted to place a $1,000 bet on his team. The gambler told his friend that he would pay him for his efforts. After further discussion, the gambler wrote up the following agreement:
    “I hereby promise to pay my friend $100 if he will place a $1,000 bet for me that my favorite team will win the college football championship next year. The bet is to be placed at my lucky casino sports book. I further promise to pay my friend an additional $100 within 30 days after the bet is placed.”
    Since the gambler knew his wife would be angry if she found out about the bet, the gambler included a clause that the friend promised not to tell anyone about the bet. After both the gambler and his friend signed the agreement, the gambler gave him $1,100, representing $1,000 to place the bet and $100 as the initial payment for the friend’s efforts.
    The friend then went on vacation. Shortly after arrival, the friend attempted to place the bet for the gambler. However, the friend discovered that the gambler’s lucky casino had recently closed its sports book. The friend then entered another casino next door and discovered that they had a sports book, also offering 100-1 odds on the gambler’s favorite team winning the college football championship the next year. The friend placed the bet with the other casino’s sports book.
    After returning from his vacation, the friend gave the betting slip to the gambler. Nothing more was said until 30 days later, when the friend asked the gambler when he was going to receive the final $100. The gambler then refused to pay anything more to the friend.

In a breach of contract action by the friend to collect the $100, which of the following, if true, is the gambler’s best defense?
(A) The contract was void ab initio, because the contract could not be performed in its entirety within the span of one year.
(B) The friend’s placing the $1,000 bet at the other casino instead of at the gambler’s lucky casino constituted a material breach of contract, thereby relieving the gambler of any further contractual obligations under the terms of their agreement.
(C) While on vacation, the friend told his girlfriend about the $1,000 that he bet on the gambler’s behalf.
(D) Since the gambler’s lucky casino no longer had a sports book at which to place the bet, all contractual obligations would be suspended under the doctrine of frustration of purpose.

A
  1. (C) The gambler’s best defense would be that the friend’s telling his girlfriend about the bet constituted a material breach of their agreement. Under the express terms of their written agreement, the friend was forbidden to tell anyone about the bet. Generally, a breach is material wherever the plaintiffs partial failure of performance defeats the purpose of the contract either wholly or in some vital aspect. However blameless the plaintiff may be in failing to perform his promise, if the result of the breach is to deprive the defendant of an essential part of the performance for which he bargained, he is not required to perform his own undertaking. Here, it was important to the gambler that the bet be kept a secret. The friend’s telling his girlfriend would thus constitute a material breach, excusing the gambler of further performance under the contract. Choice (A) is wrong for two reasons: First, the agreement was capable of full performance within one year, since the contract would be fully performed when the second payment was made within 30 days. Second, the agreement was in writing, so it would not matter if it was for longer than one year. The writing would satisfy the Statute of Frauds. Choice (B) is incorrect, because the essence of the agreement was to place a bet that his team would win the college football championship the following year. In situations where there is an impediment to a matter that, while important, is incidental to the main obligations of the parties, it is permissible to use a commercially reasonable substitute when such is available. Therefore, the contract was not discharged by the closing of the casino’s sports book, since a commercially reasonable substitute method of performance (placing the bet at the other casino at the same odds) was available. Choice (0) is incorrect for the same reason. The purpose of the contract was to place the bet on his team. The closing of the sports book at the casino did not frustrate that purpose.
128
Q
  1. A developer wished to build an apartment house in a local city. He raised $1 million from various sources to fund the project. After searching for a vacant lot that would be sufficient for development, the developer concluded that there was not a suitable vacant lot available. Reluctantly, the developer concluded that he would have to purchase a developed lot, have the existing structures removed, and then build the apartment house. The developer was reluctant to do this, because he had not budgeted for the additional cost of buying a developed property and removing the development.
    After a search for an appropriate parcel, the developer purchased a large lot, developed with a single-story residential home, for $200,000. He then spent $50,000 having the residential home removed from the lot. Knowing that it would cost approximately $850,000 to build the apartment house, the developer realized he would need to raise another $100,000. He approached an investor about lending him $100,000. The investor was apprehensive about the project because the developer had not yet lined up a construction firm to build the apartment house. As a result, the parties signed the following written agreement:
    “The investor promises to lend to the developer $100,000 at 5 percent interest within 30 days, provided the developer contracts with a construction company to build the apartment house for no more than $850,000. Loan to be repaid at the rate of $1,000 per month, starting one month after the completion of the apartment house. The loan is to be secured by a mortgage on the property.”
    The developer then contracted with a construction company to build the apartment for $800,000. After the contract was signed and construction had begun, the developer asked the investor when he would be receiving the $100,000 the investor promised to lend him. The investor replied, “I’ve changed my mind. I think this is too risky an investment. I’m not going to lend you the money.”
    In an appropriate action to secure specific performance of the investor’s promise to lend him the $100,000, the developer will
    (A) win, because there is a memorandum that satisfies the Statute of Frauds.
    (B) win, because land is unique, making the legal remedy inadequate.
    (C) lose, because the developer’s only remedy is for damages.
    (D) lose, because the developer cannot show detrimental reliance.
A
  1. (C) It is important to remember that a suit for specific performance will not lie if there is an adequate remedyat law. It wilL only lie where the loss cannot be compensated in damages. In the present case, the developer does have an adequate remedy at law, since his measurable damages would be the additional cost to obtain a similar loan from another source. Without a showing that the legal remedy is inadequate, Choice (C) is the preferred alternative. Choice (A) is wrong, because it does not matter whether the Statute of Frauds is satisfied and there is therefore an enforceable contract; the issue here is the remedy the developer is entitled to receive. Choice (B) is not the best answer. While it is true that the uniqueness of land can make the legal remedy inadequate, land is not directly involved in this transaction. This contract is a loan arrangement and nothing more. Choice (D) is incorrect. Detrimental reliance by the developer speaks to whether the contract should be enforced. That is not in issue here, as there clearly is an enforceable contract. The issue is the appropriate remedy.
129
Q
  1. A woman had spent many years converting her back yard to an authentic Japanese-style garden. Her garden had become a showplace for the neighborhood, and various civic groups and organizations frequently visited the woman’s property to enjoy the beauty of the yard. Recently, the woman had read about a particular cymbidium that was native to Japan. Desiring to have one of these cymbidiums planted in her garden, she contacted a number of plant distributors, only to discover that this particular cymbidium had not yet been imported to the United States. She contacted a plant distributor, who she knew had ties to Japan, and asked if it would be possible to acquire one of these cymbidiums from Japan. After checking with local and Japanese authorities, the distributor determined that he could indeed acquire the plant for the woman. The parties then signed a contract that stated the distributor would acquire the cymbidium from a nursery in Japan, have it shipped to the United States and have it planted in the woman’s garden for the price of $5,000. The distributor promised that the plant would be in place within six months, and the woman promised to pay $1,000 at the time of the execution of the contract and
    $4,000 when the cymbidium was planted in her garden. The woman then handed a check for $1,000 to the distributor.
    Two weeks later, agricultural officials discovered that a special type of beetle that attacked citrus plants had adapted to using cymbidiums for nesting purposes. The female beetle would lay eggs among the roots of the cymbidiums, and it would sometimes take up to one year for the eggs to hatch. Fearing for the safety of the citrus crops in the United States, the United States government adopted a ban on the importation of all cymbidiums grown in foreign countries. As a result, the distributor was not able to acquire the cymbidium for the woman.
    Which of the following best states the legal relationship at this point between the distributor the woman?
    (A) The distributor’s performance was excused because of impossibility, but the woman can seek restitution.
    (B) The distributor’s performance was not excused because of the supervening illegality, and the woman can recover damages because of the distributor’s prospective inability to perform.
    (C) The distributor’s performance was not excused because of the supervening illegality, and the woman can recover damages, provided she waits until the date performance was due before filing suit.
    (D) Both parties are excused from performance because of the supervening illegality, and neither can recover from the other.
A
  1. (A) Under the modern view, where performance of a contract has become illegal by change of law after the time of contracting, impossibility excuses the promisor’s duty unless he has assumed the risk of it or unless his fault has contributed to the prohibition. Since the distributor neither assumed the risk nor contributed to the prohibition, his obligation to provide the cymbidium to the woman would be excused by supervening illegality. Therefore, Choices (B) and (C) are wrong. In such a situation, if the promisee has already rendered performance (as in the present case, when the woman paid the initial $1,000), she may rescind and recover it back, or its value. Consequently, Choice (D) is incorrect.
130
Q
  1. After learning that a new housing development was about to be built, a developer began looking for property near the housing development on which he could build a shopping center. After an extensive search, he purchased a three-acre undeveloped tract of land for the purpose of developing a small shopping center. At the time the sale was finalized, the developer told his plans to the seller of the three-acre tract, and also mentioned that he was worried whether he had sufficient funds to complete the project. The seller agreed to lend money to the developer to allow for completion of the project. The seller then wrote out the following contract:
    “In order to help the developer with his plans for development of a shopping center on the three- acre tract he has this day purchased from me, I will lend him $50,000 at 10 percent interest provided he signs a contract with a builder for construction of the shopping center on this tract. Repayment to me at the rate of $5,000 per year to be secured by a mortgage on the three-acre tract.”
    Both parties then signed the contract.
    The seller’s promise to lend $50,000 to the developer is
    (A) a condition precedent in form but subsequent in substance to the developer’s duty to enter into a building construction contract.
    (B) a condition subsequent in form but precedent in substance to the developer’s duty to enter into a building construction contract.
    (C) a condition subsequent to the developer’s duty to enter into a building construction contract.
    (D) not a condition, either precedent or subsequent, to the developer’s duty to enter into a building construction contract.
A
  1. (A) The seller’s promise to lend the developer $50,000 is a condition precedent in form because this obligation was expressed in the contract before any reference was made to the developer’s obligation to enter into a construction contract on the adjoining lot. However, it should be noted that the seller’s promise was subsequent in substance because he would not be obligated to lend the developer the money until after the latter entered into the construction contract. Choice (B) states this result backwards. Choice (C) is wrong, because true conditions subsequent discharge a duty to perform after it has arisen. That is not the case here, since the seller would not be obligated to perform until after the developer enters into a building contract. That is why it is only subsequent in substance. Choice (D) is incorrect, because the seller’s promise is a constructive condition of some sort to the other side’s performance.
131
Q
  1. A noted author was writing a screenplay that he was adapting from his novel Quiet Winter. He assigned in writing 25 percent of any future royalties, when and if the screenplay was made into either a movie or a stage play, to his friend, who had subsidized him during his early years as a struggling writer. Shortly after the screenplay was completed, the author was killed in an auto accident. A movie studio then purchased the screenplay from the executors of the author’s estate and filmed the movie Quiet Winter, which was a great success.
    In an action against the executors of the author’s estate to recover her percentage of the movie royalties, the friend will most likely
    (A) lose, because under the circumstances an assignment of future rights is unenforceable.
    (B) lose, because the attempted gift of royalties failed for non-delivery.
    (C) win, because she was an intended beneficiary.
    (D) win, because the assignment of future rights is enforceable.
A
  1. (A) In accordance with Section 321 (2) of the Restatement of Contracts 2d, a purported assignment of a right expected to arise under a contract not in existence operates only as a promise to assign the right when it arises and as a power to enforce it. At the time the author made the purported assignment to the friend, there was no contract in place to turn the screenplay into a movie or a stage play, meaning that the author had no contractual right that he could transfer to the friend. Therefore, Choice (A) is best. Choice (B) is wrong, because immediate delivery is not necessary to make a gift promise enforceable. Choice (C) is wrong, because an intended beneficiary is someone who is designated to receive a benefit under a contract. Since the author had no contract to turn the screenplay into a movie or a stage play, the friend cannot be a beneficiary. Choice (D) is incorrect, because a future right is assignable only when it is an existing contractuaL right. For example, if the author had signed a contract to have his screenplay produced as a movie, with a promise that he would be paid royalties from that movie, the author would have had an existing future right that could have been assigned.
132
Q
  1. An athlete hoped to sign a contract with a professional baseball team to play baseball. He had succeeded in arranging for a tryout at the baseball team’s training facilities in a neighboring state. Short on funds and needing a way to get to the training facilities, the athlete approached his friend and said, “If you will drive me to and from the baseball team’s training facilities for my tryout and pay for our housing and meal expenses during our stay, I will assign to you 10 percent of my first-year earnings.” The friend agreed and drove the athlete to the training facilities. After the tryout, the baseball team offered the athlete a contract to play baseball for the team at $100,000 per year.
    When the friend asked when he would be receiving his 10 percent, the athlete replied that $10,000 was much more than the friend deserved and he would pay the friend $1,000 for his efforts. The friend then filed suit to enforce the assignment.
    If the court holds the assignment unenforceable, the most applicable legal principle would be
    (A) a purported assignment of a right expected to arise under a contract not in existence operates only as a promise to assign the right when it arises and as a power to enforce it.
    (B) a contract to make a future assignment of a right is not a validly enforceable assignment.
    (C) the friend did not detrimentally rely on the assignment prior to the attempted revocation of the assignment by the athlete.
    (D) a gratuitous assignment is revocable, and the right of the assignee is terminated by the assignor’s subsequent revocation.
A
  1. (A) An assignment of future rights under a contract not in existence is unenforceable. Since the athlete had not entered into a valid contract with the baseball team at the time, or prior to the time, of his assignment to the friend, there was nothing to assign. Choice (B) is wrong, because some assignments of future rights are enforceable. For example, an assignment of a right to payment expected to arise out of an existing employment or other continuing business relationship is effective in the same way as an assignment of an existing present right. Choice (C) is wrong, because detrimental reliance cannot change the fact that there is no existing contractual right that can be assigned. In other words, there was nothing on which the friend could have detrimentally relied. Choice (D) is not the best answer. While it is true that gratuitous assignments are revocable, had this been a valid assignment it would not have been gratuitous, since the friend was paying for the purported assignment by driving the athlete to the training facilities and paying the travel expenses.
133
Q
  1. An inventor developed a prototype of an automobile. engine that she believed was going to revolutionize the automotive industry because of the engine’s fuel-efficient properties. As a wedding present to the inventor’s brother, the inventor wrote on a wedding card she gave to her brother:
    “I hereby assign to you 20 percent of the net profits realized from the production and distribution of my automobile engine within the first year of production.”
    The inventor then borrowed money from creditors and began to promote her automobile engine. She was contacted by a major automobile manufacturer about the possibility of using the engine in a new line of cars the manufacturer intended to release during the next calendar year. While on her way to meet with the representatives of the manufacturer, the inventor was killed in an automobile collision. The manufacturer then contracted with the executor of the inventor’s estate to license the use of the automobile engine design for a 10-year period for $500,000, payable to the estate.
    The creditors from whom the inventor borrowed the money to promote her automobile engine have now attached the proceeds from the licensing agreement, which have so far been paid into the estate. The brother also asserts his assigned rights to a share of the proceeds.
    In subsequent actions by the creditors and the brother, the court will most probably hold in favor of
    (A) the brother, because the rights of an assignee are superior to a lien against the assignor subsequently obtained by legallequitable proceedings.
    (B) the brother, because any proceeds of the assigned right received by the assignor thereafter are held in constructive trust for the assignee.
    (C) the creditors, because the inventor’s assignment to the brother was unenforceable as a defective assignment.
    (D) the creditors, because the rights of creditors/lienors are superior to those of a donee beneficiary.
A
  1. (C) An assignment of future rights under a contract not in existence is unenforceable. Since the inventor had not entered into a valid contract for the use of her automobile engine prototype prior to the time of her assignment to her brother, the assignment was unenforceable. Therefore, the creditors acquired a superior possessory interest to the proceeds from the licensing of the automobile engine. Choices (A) and (B) are incorrect, because the brother never acquired enforceable rights from the inventor that would allow him any recovery. Choice (D) is wrong; the brother was not a donee beneficiary. An intended donee beneficiary is someone who is designated to receive a benefit under a contract. Since the inventor had no contract to license the use of his automobile engine at the time of the wedding, the brother cannot be a beneficiary.
134
Q
  1. A homeowner entered into a contract with an insurance company to purchase a fire insurance policy for her new home. The policy provided that the insurance company promised to pay up to $500,000 if the said house was destroyed by fire or fire-related explosion, while the homeowner promised to pay a quarterly premium of $400.
    The homeowner failed to make the last two quarterly payments before her house burned to the ground. The insurance company refused to pay the homeowner for the loss because of her failure to make the last premium payments.
    In an action by the homeowner against the insurance company to recover for the loss of her house, the homeowner’s best theory of recovery is that
    (A) although she failed to make the last two premium payments, there was a bargained-for exchange.
    (B) the insurance company’s duty to pay was not expressly conditioned on the homeowner’s duty to make the payments.
    (C) the insurance company was under an independent duty to pay for the loss.
    (D) the homeowner did not receive notice of cancellation.
A
  1. (C) In an aleatory contract, it is uniformly held that the duties of performance are independent and that each party may sue the other for breach even though he himself is in default on his own promise. So even though the homeowner is in default on her promise to pay the premium at the time her house burned, she can, nevertheless, recover for the loss on the uncancelled policy. Note that only when the insurer has expressly conditioned its performance on prompt payment of the premium is its duty a dependent one. Choice (B) is not the best answer. While Choice (B) is a correct statement, Choice (C) better, and more specifically states why the homeowner should prevail. The same is true with Choice (D). While it is technically true, it does not address the key issue as specifically as Choice (C). Choice (A) is wrong, because there is no question that a bargained-for exchange existed. The issue is whether the homeowner’s failure to make the two payments discharges the duty of the insurance company. For the reasons stated above, it does not.
135
Q
  1. An employee worked for a hardware company in a city under an at-will-employment arrangement that paid the employee a monthly salary of $2,000. After two years, the owner of the hardware store told the employee it was time for them to enter into a more formalized employment agreement. The owner and the employee then both signed a written agreement drafted by the owner, whereby the employee agreed to work for a salary of $3,000 per month. The agreement further specified that either party had a right to terminate the contract by giving 60- days’ notice to the other. Moreover, the agreement stipulated that if the employee’s employment at the hardware store should be terminated, he would not be able to engage in any aspect of the hardware business for a period of two years within a 50-mile radius of the city. Finally, the agreement stated that should the employee violate this agreement, he would be liable to the hardware company for $1,500.
    The written agreement between the owner and the employee would most likely be held
    (A) enforceable in all respects.
    (B) enforceable only with respect to the salary and termination provisions.
    (C) enforceable in all respects, except with regard to the $1,500 penalty clause.
    (D) unenforceable in all respects.
A
  1. (B) The contract provisions with respect to (a) the covenant not to compete and (b) the penalty and forfeiture clause would be severed from the contract. The restrictive covenant barring the employee from engaging in “any aspect of the hardware business for a period of two years within a 50-mile radius” would be construed as an “unreasonable restraint of trade.” The $1,500 payment clause would be a penalty because it is not a reasonable forecast of what the damages should be in the event of a breach by the employee. Therefore, Choices (A) and (C) are incorrect. Choice (D) is wrong, because the provisions regarding salary and termination notice would not be objectionable as against public policy. It is important to note that the termination provision requiring the 60-day notice is valid. However, the stipulation that (upon termination of the employment contract) the employee would be prohibited from engaging “in any aspect of the hardware business for a period of two years, within a 50-mile radius” would be invalidated as a wrongful restraint of trade.
136
Q
  1. A school hired a senior citizen to act as a crossing guard at a crosswalk located in front of the school. The contract signed by the parties required the senior citizen to be at the crosswalk every weekday afternoon from 2 pm to 4 pm, and to escort children leaving the school across the street. The senior citizen was to be paid $250 per week for his efforts. The contract also stressed the importance of the safety of the children, and stated that if the senior citizen should fail to show up for work without giving notice in advance so a substitute can be located, the senior citizen would be subject to a $200 per incident penalty.
    The contract provision making the senior citizen liable for a $200 per incident penalty may best be described as a (an)
    (A) liquidated damage clause.
    (B) unliquidated damage clause.
    (C) penalty and forfeiture clause.
    (D) aleatory clause.
A
  1. (C) Choice (C) is correct, because a penalty or forfeiture clause is a sum in excess of the value of the contract and fixed to be paid on breach of it. Students should note that if the parties fix upon a certain sum to be paid on breach of the contract, it may be recovered if it was really fixed upon as liquidated damages for non-performance. However, if it was intended in the nature of a penalty in excess of any loss likely to be sustained, the recovery will be limited to the loss actually sustained. Choice (A) is therefore wrong, because this was not an enforceable liquidated damage clause. Choice (B) is incorrect, because an unliquidated damage claim is one that is in dispute. Here, there is no dispute between the parties. Choice (D) is incorrect, since an aleatory or insurance contract is one in which performance is conditioned upon an event (e.g., house burning down) that may never happen.
137
Q
  1. A screenwriter had just finished a screenplay for a romantic comedy. After reading the screenplay, a movie studio decided to option the screenplay for production. A lawyer for the movie studio negotiated an agreement with the screenwriter whereby the movie studio would pay the screenwriter $5,000 for the rights to the screenplay, and 10 percent of the gross receipts earned by the distribution of the movie, should the studio decide to film the screenplay. Both sides signed the agreement.
    Two weeks later, the screenwriter was at a birthday party for a close friend when she discovered she had forgotten to purchase a gift for her friend. She quickly grabbed a piece of paper and wrote on it:
    “I promise that my friend is entitled to 5 percent of the proceeds I will receive for the distribution of the movie to be made from my screenplay.”
    The screenwriter then signed the paper, placed it in an envelope, and handed it to the friend.
    The promise to pay 5 percent of the proceeds to the friend may best be interpreted as a (an)
    (A) equitable lien.
    (B) conditional assignment.
    (C) irrevocable assignment.
    (D) gratuitous assignment of future rights.
A
  1. (D) Accordingto Restatement of Contracts 2d, Section 321, the assignment ofa future right to payment expected to arise out of an existing employment or other continuing business relation is effective in the same way as an assignment of an existing right. Here, the screenwriter’s assignment of a portion of the proceeds she anticipated earning on the distribution of the movie (should it be made) would be effective as a gratuitous assignment, even though the movie did not exist at the time of the assignment. Choice (A) is wrong, because an equitable lien is a court- imposed remedy to prevent unjust enrichment. That would not apply to this situation. Choice (B) is not the best answer. While it is true that there is a condition precedent to the friend’s receiving any money (that the movie is made), Choice CD) is a more specific answer, and a more specific answer is always a more preferred answer. Choice (C) is incorrect, because the assignment was gratuitous, and as a general rule, gratuitous assignments are revocable.
138
Q
  1. On January 1, as part of a New Year’s resolution, a woman signed the following agreement with an exercise facility:
    “I hereby enroll in the exercise facility’s exercise program. A condition of this contract is that all fees are to be paid in advance. If, however, the total enrollment fees exceed $1,250, then one-third shall be paid upon the signing of said agreement, with one-third payable three months later, and one-third six months later. Under no circumstances shall any fees be refundable.”
    The woman was informed that the fee for the exercise program in which she wanted to participate was $1,500, and that figure was written into the contract. Upon signing the contract, the woman made her first payment of $500 and started classes the next day.
    To most accurately describe the installment payment of $500 due on April 1 as it applies to continued use of the exercise facilities, it should be construed as a
    (A) condition precedent.
    (B) condition subsequent.
    (C) concurrent condition.
    (D) express condition.
A
  1. (A) In this situation, the exercise facility will not be obligated to perform its duties under the contract (namely, admit the woman to any further exercise classes) until she makes the installment payment. Thus, the woman’s duty to pay the $500 installment is viewed as a condition precedent to the exercise facility’s being obligated to perform its duties under the contract. In other words, the exercise facility may bar the woman from attending any further classes until such time as she makes the installment payment. Choices (B) and (C) are therefore wrong. Choice (D) is incorrect, because an express condition is one that is stated in the contract as making a performance of a promise expressly conditional upon the occurrence of that condition. That was not the case here, because the contract did not specifically state that the exercise facility’s performance was conditional upon the woman’s making her installment payment.
139
Q
  1. A man who was an avid swimmer moved to a new town and began looking for a facility with a swimming pool that he could use. He signed an agreement with the local swimming club to use their swimming pool four times per week. The agreement, which had a duration of one year, stated that the man could use the poo1 four times per week for $250 per month, due on the first day of each month. For three months, the man paid the $250 on the first and made use of the swimming pool. Toward the middle of the third month, the man was involved in an automobile accident. Shortly thereafter, the man sent the following letter to the swimming club:
    “Please be advised that because of injuries sustained in an automobile accident, my physician has advised me not to engage in any strenuous activities. Since I will not be able to make use of the pool anymore, no further payments will be forthcoming.”
    Which of the following does not accurately state the immediate legal effect of the man’s letter?
    (A) The swimming club has the right to bar the man from any further use of the pooi.
    (B) The swimming club has the right to sue the man immediately for breach of contract.
    (C) The man may retract his repudiation if he does so before the swimming club initiates legal action against him.
    (D) The swimming club must wait until the date of the next payment in order to sue the man for breach of contract.
A
  1. (D) Generally, the promisee has the following remedies available afterthe promisor has repudiated the contract in advance of the time for its performance: (1) sue at once for breach; (2) treat the repudiation as an offer of mutual rescission and accept it in discharge of the contract; (3) treat the repudiation as excusing his own further duty of performance; or (4) ignore the repudiation and urge the promisor to perform. Choice (D) is correct, because it is the only choice that does not accurately reflect one of the immediate legal effects of the man’s letter. The swimming club is not required to wait until the date of the next payment, because the man’s letter made it clear that he will not pay any more of the money owed under the contract. The man’s anticipatory repudiation means that the swimming club does have the right to sue immediately. Since Choice (B) is an accurate legal effect of the man’s letter, it is an incorrect choice. Choice (A) is also an accurate legal effect (and therefore an incorrect choice), because the swimming club may now treat the contract as discharged, thereby giving the swimming club the right to prevent the man from making any further use of the pool (if he so chose to return). Students should also be aware that the promisor may retract his repudiation up to the time the promisee has accepted the repudiation or detrimentally relied upon it. Since the man may retract his repudiation so long as the swimming club has not accepted his repudiation (by bringing legal action as in Choice (C), for example), Choice (C) is another accurate legal effect of his letter, making this choice incorrect.
140
Q
  1. A woman who was three months pregnant enrolled in a pre-natal and post-natal exercise program at the local gymnasium. The contract signed by the woman was to last for one year and called upon the woman to pay $2,000 in quarterly payments of $500. The contract also specified that all fees were non-refundable.
    Upon signing the contract, the woman made her first payment of $500 and started classes the next day. The woman attended classes on a regular basis for the next three months. At the end of the three-month period, the woman was involved in an automobile accident. Although she was not seriously injured, her doctor advised the woman that she needed complete rest. Her doctor told her to avoid strenuous activity and advised her to discontinue her pre-natal exercise classes. One week later, the woman received a billing notice from the gymnasium, indicating that her second installment payment of $500 was past due. She immediately telephoned the manager of the gymnasium and informed him that she would not be attending any further classes because of her accident. Also, the woman said she did not feel obligated to make any additional payments.
    Which of the following most accurately describes the woman’s duty to pay the gymnasium the second installment fee of $500?
    (A) It would be excused, because of impossibility of performance.
    (B) It would be excused, because the essential purpose of the contract was frustrated.
    (C) It would not be excused, because the contract stipulated that no fees would be refundable.
    (D) It would be not excused, because her covenant to make the installment payment is also enforceable as a condition precedent.
A
  1. (B) Choice (B) is correct, because frustration of purpose will excuse the prom isor’s duty where (1) the value of the performance bargained for by the promisor is destroyed by the supervening event, and (2) the frustrating event was not foreseeable at the time the parties contracted The woman’s accident clearly frustrates the purpose behind the contract, since the woman can no longer take advantage of the exercise classes. Choice (A) is wrong, because subjective impossibility will not excuse the promisor’s duty of performance under a contract. The mere personal inability of a promisor to perform is no excuse. This is subjective impossibility. At common law, only objective impossibility (i.e., where because of the supervening event the performance cannot be rendered by anyone) will excuse the (promisor’s) duty of performance. Choice (C) is wrong, because the woman is not seeking a refund of fees paid. Rather, frustration discharges her obligation to make the next payment. Choice (0) is not the best answer. While it is true that her promise to pay is a condition precedent to the gymnasium’s allowing her to attend classes, the promise (and the corresponding condition) are discharged by the frustration of purpose.
141
Q
141. A man decided to enroll in a creative writing course at a local adult education facility. The man signed an agreement with the facility that he would attend weekly classes at the facility for six months and would pay $500 on the first of each month. Upon signing the contract, the man made his first payment of $500 and attended the weekly class sessions. On the first day of the second month, the man once again made his $500 payment and continued to attend the session. At the start of the third month, the man told the director of the facility’s accounting office that he was having financial difficulties, and asked for an extension. The director agreed to permit the man to attend classes while the man tried to get the money to pay the facility. One week later, the man received a letter from the facility informing him that he would be barred from attending any further classes unless the next $500 payment was made.
Is the facility justified in refusing to permit the man to attend classes?
(A) No, because by permitting the man to attend classes without having paid the third monthly installment, the facility waived its right to have that installment fee paid on time.
(B) No, because the facility’s allowing the man to attend classes without paying the third monthly installment created an implied contract, thus permitting him to complete the classes without advance payment of the installments.
(C) Yes, because the man’s failure to make the third monthly installment payment constituted an anticipatory breach.
(D) Yes, because there was no consideration to extinguish the payment of the third monthly installment fee, which was a material part of the contract.
A
  1. (D) Students should be aware that voluntary waiver (of condition) by the promisor is only effective to eliminate a condition that is not a material part of the agreed exchange, one that is onLy an incident to the promisee’s performance. A condition, however, that is all or a substantial part of the agreed exchange can only be waived by a substituted contract on sufficient consideration. To be sure, the (condition) of the payment of the third monthly installment was a substantial or material part of the agreement between the facility and the man. Consequently, that condition could not be waived or extinguished unless supported by consideration or material change of position by the promisee in reliance, thus amounting to an estoppel. Choice (A) is therefore incorrect. Choice (B) is wrong, because no such implied contract would be created. Choice (C) is wrong, because here, there is a present breach (not an anticipatory breach). An anticipatory repudiation occurs when one party to an executory bilateral contract repudiates the contract in advance of the time setfor performance by announcing that he will not perform it. In the present case, the man did not inform the facility prior to the first day of the third month of his intention to repudiate the contract.
142
Q
  1. A widget manufacturer and a widget retailer signed the following agreement:
    “The manufacturer promises to sell and the retailer promises to buy 10,000 widgets at $50 per 100. One thousand widgets are to be delivered by the manufacturer on the first day of each of the next 10 months. EACH MONTHLY DELIVERY ISA SEPARATE CONTRACT. Payment to be made within 10 days of receipt of each shipment.”
    The agreement between the manufacturer and the retailer may best be interpreted as
    (A) a divisible contract.
    (B) an installment contract.
    (C) 10 separate contracts.
    (D) a requirements-output contract.
A
  1. (B) UCC 2-612(1) defines an installment contract as “one which requires or authorizes the delivery of goods in separate lots to be separately accepted, even though the contract contains a clause that each delivery is a separate contract or its equivalent.” Thus, the original agreement would be interpreted as a single installment contract. Choices (A) and (C) are therefore incorrect. Choice (0) is wrong, because a requirements contract is one where the quantity is measured by the requirements of the buyer, and an output contract is one where the quantity is measured by the output of the seller. Since this contract is for a fixed amount (10,000 widgets), it would not be a requirements or an output contract.
143
Q
  1. A lumber supplier and a fence company signed the following agreement on May 1:
    “The supplier promises to sell and the fence company promises to buy 7,000 sections of redwood stockade fence at $30 per section. Each section is to be made of good quality split redwood poles and is to be 7 feet long and 6 feet high; 1,000 sections are to be delivered by seller on or before June 1, and 1,000 sections by the first day in each of the following six months. Payment for the sections to be made within 10 days of delivery.”
    The first shipment of 1,000 sections arrived on May 27, and the fence company sent its payment on June
  2. The second shipment arrived on July 1, and the fence company made payment on July 5. The August shipment arrived on the afternoon of August 1. After the initial inspection, the redwood poles were found to be 7 feet long and 6.25 feet high. The manager of the fence company then called the president of the lumber supplier. During their conversation, the president told the manager that the lumber supplier could not replace the August shipment but would allow a price adjustment. The manager refused the president’s offer. The next day, the manager sent the president a fax stating that he was hereby canceling all future deliveries and returning the last shipment because of nonconformity.
    If the lumber supplier sues the fence company for breach of contract, the court will most likely hold that the lumber company will
    (A) succeed, because all deliveries to date have been timely.
    (B) succeed, because the president offered to adjust the price for the August shipment.
    (C) not succeed, because the president refused to replace the nonconforming poles.
    (D) not succeed, because the deviation impaired the value of the entire contract.
A
  1. (B) When does the breach of part of an installment contract constitute a breach of the whole? According to UCC 2-612(3), “whenever non-conformity or default with respect to one installment substantially impairs the value of the entire contract there is a breach of the whole.” In the official comments, the UCC states that “Cure of non-conformity of an installment…can usually be afforded by an allowance against the price…” Therefore, by offering to make a price adjustment, the non-conformity in the August shipment would not substantially impair the entire contract, so the fence company would have no right to cancel the contract and would be in breach for doing so. Choice (D) is therefore wrong. Choice (A) is incorrect, because the issue is not the timing of the deliveries; it is whether the non-conformity in the August shipment substantially impairs the entire contract. Choice (C) is wrong, because the price adjustment would be a proper attempt to cure the improper delivery; it was not necessary that the poles be replaced.
144
Q
  1. A supermarket signed a contract with a bakery to provide the supermarket with 100 loaves of whole wheat bread per week for 12 consecutive weeks. The loaves were to be delivered on the first day of each week, with payment to be made within four days of delivery. For the first four weeks, the bakery delivered loaves to the supermarket and the supermarket made the appropriate payments. When the fifth delivery arrived, the supermarket discovered that the shipment contained 80 whole wheat loaves and 20 sourdough loaves. The manager of the supermarket immediately called the bakery to complain about the shipment. The operator of the bakery apologized and offered to send 20 loaves of whole wheat bread within 24 hours.
    What is the probable legal effect of the operator’s conversation with the manager with regard to the fifth shipment?
    (A) The supermarket would have the right to reject the fifth shipment and cancel their contract.
    (B) The supermarket would have the right to reject the fifth shipment, but would be held liable for the remaining deliveries.
    (C) The supermarket would not be entitled to reject the operator’s offer to “cure.”
    (D) The supermarket would have a right to “cover” by purchasing substitute loaves of bread.
A
  1. (C) The supermarket would not be entitled to reject the operator’s offer to cure, since the nonconformity in loaves would not constitute a material breach. When dealing with an installment contract, a specific default or nonconformity with respect to one installment does not constitute a breach of the entire contract unless the non-conformity substantially impairs the value of the entire contract. The problem with the 20 loaves would not substantially impair the entire contract, so Choice (A) is wrong. The UCC also holds that if the seller gives adequate assurances that it will cure any non-conformity, such a cure attempt must be accepted. Therefore, the supermarket would have to accept the operator’s promise to cure the problem with the loaves. Choice (B) is therefore wrong, because the supermarket would have no right to reject the fifth shipment. Choice (D) is wrong, because the supermarket would have no right to damages, since the bakery is not in breach, because of the assurance to cure the non-conformity.
145
Q
  1. A lumber mill contracted to deliver one thousand
    8-foot sheets of plywood to a home improvement store on the first of each month for 10 consecutive months starting June 1. The June, July, and August shipments were delivered on time and duly paid for. Then, on August 16, the lumber mill informed the store that the mill would not be able to meet the September 1 delivery date, because its lumber reserve had been destroyed by a forest fire. The mill then asked the store to excuse the mill from further performance. The store refused and demanded that the remaining shipments be delivered on time.
    When the September shipment failed to arrive, the store immediately brought suit for breach of contract. How would the court hold?
    (A) Judgment for the store, because the mill’s duties of performance would not be excused.
    (B) Judgment for the store, because the mill should have foreseen such a contingency occurring.
    (C) Judgment for the mill, because their performance would be discharged by impossibility.
    (D) Judgment for the mill, because their performance would be discharged by frustration of purpose.
A
  1. (A) The forest fire would temporarily suspend the mill’s duty to deliver the August shipment. Note that temporary impossibility suspends contractual duties; it does not discharge them. When performance once more becomes possible, the duty “springs back” into existence. Therefore, the mill’s contractual obligations (with regard to the September, October, November and December shipments) would not be excused by the temporary impossibility (i.e., forest fire). Choices (C) and (D) would therefore be wrong. Choice (B) is not the best answer. The UCC does not state that a party is to be charged with foreseeing a particular contingency; it states that a party cannot use such a contingency as discharging its duty. Therefore, Choice (A) is the more preferred answer.
146
Q
  1. On May 1, a homeowner and a painter entered into a contract whereby the painter was to paint the homeowner’s home in exchange for $10,000. The contract stated:
    “The painting is to be done during the month of June, and payment is to be made within one week after the painting is completed. The rights under this contact are not to be assigned.”
    The painter started the painting on June 1. On June 10, the painter assigned to his nephew the right to receive payment on the painter’s contract with the homeowner.
    Which of the following statements is most accurate with regard to the painter’s assignment to his nephew?
    (A) The homeowner would not be obligated to pay the nephew, since the contract was nonassignable.
    (B) Since personal service contracts of this nature are nonassignable, the homeowner would be under no duty to pay the nephew.
    (C) The assignment would constitute a novation, relieving the homeowner of liability.
    (D) The assignment would be irrevocable if it were reduced to writing and signed by the painter.
A
  1. (D) In accordance with the Restatement of Contracts 2d, Section 332, unless a contrary intention is manifested, a gratuitous assignment is irrevocable if the assignment is in a writing either signed or under seal that is delivered by the assignor. Choice CD) is therefore an accurate statement. Choice (A) is not an accurate statement, because contractual prohibitions on assignments are generally viewed as just a promise not to assign, meaning any attempted assignment will be effective, but the assignor can be sued for breach for breaking the promise not to assign. Therefore, the assignment to the nephew would be effective. Choice (B) is not an accurate statement, because the payment of money does not involve a personal service and can therefore be assigned. Generally, duties under a personal service contract cannot be delegated. Choice (C) is not an accurate statement. A novation is a three- party agreement that grows out of a delegation. Since there was no delegation of duties, novation is not in issue.
147
Q
  1. On February 1, an owner of six vacation cottages in a resort area hired a painter to paint all six cottages for $50,000. The contract stipulated that no money would be due until the owner’s caretaker approved the work done by the painter.
    The painter completed the painting of all six cottages. Two days after the painter had finished the painting, a wildfire destroyed the six cottages. Although the caretaker had inspected four of the cottages to his approval, he had not inspected the other two cottages prior to their destruction.
    Which of the following is the LEAST accurate statement with respect to the caretaker’s approval of the painting of the cottages?
    (A) The owner would only be obligated to pay for the painting of the four cottages that the caretaker approved.
    (B) The caretaker’s approval of the painting of the two cottages destroyed by the wild fire would be excused.
    (C) The owner would be obligated to pay for the painting of all of the cottages.
    (D) The impossibility of securing the caretaker’s approval would render the owner absolutely liable for all of the painting.
A
  1. (A) Under the general rule, impossibility of performance of a condition that is not a material part of the exchange will excuse the condition where otherwise forfeiture would result. Where the promisee (i.e., the painter) has already given the promisor (i.e., the owner) the substantial performance that was bargained for, but because of supervening impossibility has not performed some incidental thing that was made an express condition of the promise, the condition may be excused and the liability of the promisor becomes absolute. Choices (B), (C), and (D) would all therefore be accurate statements. Choice (A) is thus the least accurate and the correct answer.
148
Q
  1. On March 1, a homeowner hired a landscaper to landscape his front yard. On March 25, as the landscaper was almost finished with the work on the front yard, the homeowner said to the landscaper, “I’m very pleased with your work to date. If you are willing, I would like you to landscape my backyard, on the same terms and conditions as our existing contract, the work to be completed by May 1. If you can meet that deadline, I’ll pay you $10,000 at that time.” The landscaper replied, “Agreed. Let’s shake hands on it.” They shook hands.
    What is the probable legal effect of the landscaper’s promise on March 25 to landscape the backyard?
    (A) The landscaper’s promise created an enforceable unilateral contract.
    (B) The landscaper’s promise created an enforceable bilateral contract.
    (C) The landscaper’s promise was voidable, since it was not in writing.
    (D) The landscaper’s promise was illusory, thereby creating an unenforceable contract.
A
  1. (B) On March 25, by agreeing to landscape the backyard, the man’s return promise created an enforceable bilateral contract. A bilateral contract is a contract in which mutual promises are given as the agreed exchange for each other. On the contrary, a unilateral contract is a contract in which a promise is given in exchange for an actual performance by the other party. Here the parties were exchanging promises (the landscaper promising to landscape the backyard by May 1 and the homeowner promising to pay $10,000), so Choice (A) is wrong. Choice (C) is incorrect, because there is no reason why the contract would have to be in writing. It would not fall within any of the sections of the Statute of Frauds. Choice (D) is incorrect, because the landscaper is obligating himself to landscape the back yard by May 1, so his promise was not illusory.
149
Q
  1. A woman owned four beach houses. Each house had star-shaped windows. As the hurricane season was approaching, she hired a carpenter to build storm windows for the star-shaped and deliver them to the four houses. Because of the unique shape of the windows, the carpenter had to do all work by hand. The carpenter and woman signed a contract under which the carpenter promised to build and deliver the storm windows to all four beach houses within 30 days, and the woman promised to pay $8,000 ($2,000 per beach house) within one week of completion of the job.
    Two weeks after the contract was signed, a fire destroyed the carpenter’s workshop, where he was storing all the completed storm windows before delivery. The carpenter then sent a letter to the woman that read:
    “The fire destroyed my equipment and inventory— including the storm windows, which had been completed and which were awaiting delivery. I am dead broke and cannot complete my obligations under the contract.” The woman then telephoned the carpenter and told him, “Unless you fulfill your contractual obligations, I will sue you!”
    In an action for specific performance to compel the carpenter to build and deliver the storm windows, the woman will most likely
    (A) succeed, because the carpenter’s loss of the inventory would not excuse his duty of performance.
    (B) succeed, because the beach houses would suffer severe damage in the event a hurricane struck without the storm windows having been installed.
    (C) not succeed, because the carpenter’s performance would be excused by the unforeseeable act of God.
    (D) not succeed, because the carpenter’s loss of inventory would render his performance impossible.
A
  1. (A) The woman and the carpenter had an enforceable bilateral contract whereby the he was obligated to build and deliver the storm windows. The loss of the inventory will not excuse his duty to perform (under the doctrine of subjective impossibility), because the promisor, by entering into the contract, has assumed the risk of his ability to perform. In short, to constitute an excuse through impossibility of performance, the thing promised must be impossible of performance by anyone; it must be physically and objectively impossible. Choice CD) is therefore incorrect. Choice (B) is not a relevant concern. The woman would have assumed the risk of damage to her beach houses by not contracting earlier for the storm windows. Choice (C) is incorrect for the reason stated above; the carpenter would be charged with assuming the risk of the loss of the inventory.
150
Q
  1. On March 1, a man and a contractor executed a contract that provided that the contractor would construct a two-level redwood deck on each of the eight specified beach houses in a city that the man owned, with all work to be completed by May 1. The contract provided the following:
    “The cost is $2,500 per deck, to be paid upon completion of the decks in each of the eight beach houses.”
    On the morning of March 25, the man went to one of the beach houses as the contractor and his assistants were completing the work on the fourth deck. The man said to the contractor, “I’m very pleased with your progress to date. If you are willing, I would like you to build the same kind of decks on four identical beach houses that I own in a neighboring town, on the same terms and conditions as our existing contract, the work to be completed by May 20. If you can meet that deadline, I’ll pay you $10,000 at that time.” The contractor replied, “Agreed. Let’s shake hands on it.” They shook hands.
    The contractor completed the work on the beach houses located in the city by April 15 and immediately started the construction of the decks on the four beach houses located in the town. On April 22, a hurricane totally destroyed four of the man’s beach houses in the city. In addition, the hurricane demolished a beach house in the town on which the contractor had completed 85 percent of the deck work. Two of the contractor’s assistants were seriously injured, and all of his equipment was washed away.
    The man then received a letter from the contractor, which stated:
    “The hurricane destroyed my equipment, worth $4,000. I am dead broke and cannot complete the work on the three remaining beach houses in the town. Please pay me $20,000 for the work I did in the city, and please send me $2,500 for the deck I built on the beach house in the town.”
    What is the maximum amount that the contractor may recover from the man for the construction that he completed prior to the hurricane?
    (A) $10,000.
    (B) $10,000 plus the amount due for 85 percent of the completed work on the town beach house.
    (C) $20,000.
    (D) $20,000 plus the amount due for 85 percent of the work completed on the town beach house.
A
  1. (C) As per the terms of their agreement, the man was only obligated to pay the contractor $2,500 per deck upon completion. As a result, the contractor will recover $2,500 for each of the eight decks he constructed on the city properties ($20,000). However, the contractor will not be entitled to recover for the partial work he performed on the one beach house located in the town, since it is generally held that a builder in promising the result has accepted the attendant risks inherently involved. Choices (A), (B), and (0) are all therefore incorrect.